You are on page 1of 235

DSAT Reading and Writing

Test Ninjas Publication


Made in California

2024 Edition

www.test-ninjas.com
Contents

I Introduction 7

1 Introduction: About the DSAT 8

1.0.1 Format . . . . . . . . . . . . . . . . . . . . . . . . . . . . . . . . . . . . . 8

1.0.2 Test Duration . . . . . . . . . . . . . . . . . . . . . . . . . . . . . . . . . 8

1.0.3 Adaptive Testing . . . . . . . . . . . . . . . . . . . . . . . . . . . . . . . . 8

1.0.4 Test Experience . . . . . . . . . . . . . . . . . . . . . . . . . . . . . . . . 9

1.0.5 Question Format . . . . . . . . . . . . . . . . . . . . . . . . . . . . . . . . 9

1.0.6 Test Security . . . . . . . . . . . . . . . . . . . . . . . . . . . . . . . . . . 9

1.0.7 Score Delivery . . . . . . . . . . . . . . . . . . . . . . . . . . . . . . . . . 9

1.0.8 Conclusion . . . . . . . . . . . . . . . . . . . . . . . . . . . . . . . . . . . 9

2 Reading Question Types 10

2.1 Introduction . . . . . . . . . . . . . . . . . . . . . . . . . . . . . . . . . . . . . . 10

2.1.1 Domain: Craft and Structure . . . . . . . . . . . . . . . . . . . . . . . . . 10

2.1.2 Domain: Information and Ideas . . . . . . . . . . . . . . . . . . . . . . . . 11

2.1.3 Domain: Standard English Conventions . . . . . . . . . . . . . . . . . . . . 11

2.1.4 Domain: Expression of Ideas . . . . . . . . . . . . . . . . . . . . . . . . . . 11

3 Scoring 1600 13

3.1 The Ultimate Guide to Scoring 1600 on the SAT: Strategies and Tips . . . . . . . . 13

2
3.1.1 Building a Solid Foundation . . . . . . . . . . . . . . . . . . . . . . . . . . 13

3.1.2 Developing a Customized Study Plan . . . . . . . . . . . . . . . . . . . . . 14

3.1.3 If you scored 1550 . . . . . . . . . . . . . . . . . . . . . . . . . . . . . . . 15

3.2 Mastering Time Management Skills for SAT Success . . . . . . . . . . . . . . . . . 15

3.3 Tools for Time Management . . . . . . . . . . . . . . . . . . . . . . . . . . . . . . 15

3.3.1 Using a Computer . . . . . . . . . . . . . . . . . . . . . . . . . . . . . . . 15

3.3.2 Time Tracking Software . . . . . . . . . . . . . . . . . . . . . . . . . . . . 16

3.4 Strategies for Time Management . . . . . . . . . . . . . . . . . . . . . . . . . . . 16

3.4.1 The Skip and Return Strategy . . . . . . . . . . . . . . . . . . . . . . . . . 16

3.4.2 The Section Division Strategy . . . . . . . . . . . . . . . . . . . . . . . . . 16

3.5 Practicing Time Management . . . . . . . . . . . . . . . . . . . . . . . . . . . . . 16

3.5.1 Mock Tests . . . . . . . . . . . . . . . . . . . . . . . . . . . . . . . . . . 16

3.5.2 Gradual Time Reduction . . . . . . . . . . . . . . . . . . . . . . . . . . . . 16

3.5.3 Anxiety Management . . . . . . . . . . . . . . . . . . . . . . . . . . . . . 16

3.6 Non-Traditional Methods for Test Preparation . . . . . . . . . . . . . . . . . . . . 17

3.6.1 Mind Mapping . . . . . . . . . . . . . . . . . . . . . . . . . . . . . . . . . 17

3.6.2 Dual Coding . . . . . . . . . . . . . . . . . . . . . . . . . . . . . . . . . . 17

3.6.3 Interleaved Practice . . . . . . . . . . . . . . . . . . . . . . . . . . . . . . 17

3.6.4 Test-Taking Tips . . . . . . . . . . . . . . . . . . . . . . . . . . . . . . . . 18

3.6.5 Test day preparation . . . . . . . . . . . . . . . . . . . . . . . . . . . . . . 18

3.7 Building Mental Resilience . . . . . . . . . . . . . . . . . . . . . . . . . . . . . . . 19

3.8 Mindfulness Meditation . . . . . . . . . . . . . . . . . . . . . . . . . . . . . . . . 19

3.9 Visualization . . . . . . . . . . . . . . . . . . . . . . . . . . . . . . . . . . . . . . 19

3.10 Mastering the Art of Educated Guessing in Multiple-Choice Questions . . . . . . . . 20

3.10.1 The Process of Elimination . . . . . . . . . . . . . . . . . . . . . . . . . . 20

3.10.2 Beware of Absolute Terms . . . . . . . . . . . . . . . . . . . . . . . . . . . 21

3.10.3 Choose the Longest or Most Detailed Answer . . . . . . . . . . . . . . . . . 21

3
3.10.4 Trust Your First Instinct . . . . . . . . . . . . . . . . . . . . . . . . . . . . 21

3.10.5 When All Else Fails, Guess . . . . . . . . . . . . . . . . . . . . . . . . . . . 21

3.11 Positive Affirmations . . . . . . . . . . . . . . . . . . . . . . . . . . . . . . . . . . 22

3.12 Harnessing the Power of Positive Affirmations for Test Taking . . . . . . . . . . . . 22

3.12.1 Understanding Positive Affirmations . . . . . . . . . . . . . . . . . . . . . . 22

3.12.2 The Role of Positive Affirmations in Test Preparation . . . . . . . . . . . . 22

3.12.3 Incorporating Positive Affirmations into Test Preparation . . . . . . . . . . . 22

3.13 Conclusion . . . . . . . . . . . . . . . . . . . . . . . . . . . . . . . . . . . . . . . 23

II “How to Solve” Tutorials 24

4 How to Solve “Craft and Structure - Word In Context” 25

4.1 Example “Craft and Structure - Word In Context” Question . . . . . . . . . . . . . 25

4.2 How to solve “Craft and Structure - Word In Context” . . . . . . . . . . . . . . . . 26

5 How to Solve “Craft and Structure - Text Structure and Purpose” 29

5.1 Example “Craft and Structure - Text Structure and Purpose” Question . . . . . . . 29

5.2 How to solve “Craft and Structure - Text Structure and Purpose” . . . . . . . . . . 30

6 How to Solve “Craft and Structure - Cross-Text Connections” 34

6.1 Example “Craft and Structure - Cross-Text Connections” Question . . . . . . . . . 34

6.2 How to solve “Craft and Structure - Cross-Text Connections” . . . . . . . . . . . . 35

7 How to Solve “Information and Ideas - Central Ideas and Details” 39

7.1 Example “Craft and Structure - Cross-Text Connections” Question . . . . . . . . . 39

7.2 How to solve “Information and Ideas - Central Ideas and Details” . . . . . . . . . . 40

8 How to Solve “Information and Ideas - Command of Evidence” 44

8.1 Example “Information and Ideas - Command of Evidence” Question . . . . . . . . . 44

8.2 How to solve “Information and Ideas - Command of Evidence” . . . . . . . . . . . . 45

4
9 How to Solve “Information and Ideas - Inferences” 49

9.1 Example “Inferences” Question . . . . . . . . . . . . . . . . . . . . . . . . . . . . 49

9.2 How to solve “Information and Ideas - Inferences” . . . . . . . . . . . . . . . . . . 50

10 How to Solve “Standard English Conventions - Boundaries” 54

10.1 Example “Standard English Conventions - Boundaries” Question . . . . . . . . . . . 54

10.2 How to solve “Standard English Conventions - Boundaries” . . . . . . . . . . . . . 55

11 How to Solve “Standard English Conventions - Form Structure, and Sense” 59

11.1 Example “Standard English Conventions - Form Structure, and Sense” Question . . 59

11.2 How to solve “Standard English Conventions - Form, Structure, and Sense” . . . . . 60

12 How to Solve ”Expression of Ideas - Rhetorical Synthesis” 62

12.1 Example ”Expression of Ideas - Rhetorical Synthesis” Question . . . . . . . . . . . . 63

12.2 How to solve “Expression of Ideas - Rhetorical Synthesis” . . . . . . . . . . . . . . 64

13 How to Solve “Expression of Ideas - Transitions” 68

13.1 Example “Expression of Ideas - Transitions” Question . . . . . . . . . . . . . . . . 68

13.2 How to solve “Expression of Ideas - Transition” . . . . . . . . . . . . . . . . . . . . 69

13.2.1 Additional Transition Word List . . . . . . . . . . . . . . . . . . . . . . . . 72

III Practice Modules 75

14 Module 1 76

15 Module 1 Answers 91

16 Module 2 96

17 Module 2 Answers 110

18 Module 3 115

5
19 Module 3 Answers 130

20 Module 4 135

21 Module 4 Answers 150

22 Module 5 155

23 Module 5 Answers 170

24 Module 6 175

25 Module 6 Answers 190

26 Module 7 195

27 Module 7 Answers 210

28 Module 8 215

29 Module 8 Answers 230

6
Part I

Introduction

7
Chapter 1

Introduction: About the DSAT

The transition from the traditional paper-pencil SAT to the digital SAT brings several key
di↵erences that impact the format, test duration, adaptability, test experience, question format,
test security, and score delivery. Understanding these di↵erences will help students better
prepare for the new digital SAT.

1.0.1 Format

The digital SAT will be taken on a laptop or tablet using a custom exam application, while the
old SAT used paper and pencil. The digital format o↵ers greater flexibility and convenience for
test-takers, as well as streamlined administration and grading for test centers.

1.0.2 Test Duration

The digital SAT will be shorter, approximately 2 hours long, compared to the old SAT’s 3-hour
duration. This change in duration reduces test fatigue, allowing students to maintain focus and
perform optimally throughout the exam.

1.0.3 Adaptive Testing

The digital SAT will be adaptive, adjusting the questions in the second module based on a
student’s performance in the first module. The old SAT did not have adaptive testing and
followed a fixed question sequence. Adaptive testing allows the digital SAT to provide a more
accurate assessment of a student’s abilities by tailoring the test to their specific skill level.

8
1.0.4 Test Experience

The digital SAT will have built-in tools like a countdown clock, flagging questions for review,
a built-in graphing calculator, and a math reference sheet. These tools were not available in
the old SAT format. The additional tools provided in the digital SAT enhance the testing
experience and allow for more efficient test-taking strategies.

1.0.5 Question Format

The digital SAT will feature more direct questions, with the Reading and Writing section
containing shorter texts tied to individual questions. The Math section will have more concise
word problems. The old SAT had longer reading passages with multiple questions per passage
and lengthier word problems. The new question format in the digital SAT aims to streamline
the testing process, focusing on the core skills and knowledge required for college and career
readiness.

1.0.6 Test Security

The digital SAT provides unique test forms for each student, making it almost impossible to
share answers and increasing test security. The old SAT had standardized test forms, which
were less secure. By implementing unique test forms, the digital SAT ensures a fair testing
environment and protects the integrity of the test results.

1.0.7 Score Delivery

Scores for the digital SAT will be delivered in days instead of weeks, as was the case with the
old SAT. The expedited score delivery allows students to receive their results more quickly,
enabling them to make timely decisions regarding college applications and other opportunities.

1.0.8 Conclusion

Despite these di↵erences, both the new digital and old SAT will measure the same core reading,
writing, and math skills, use the same 1600-point scoring scale, and aim to assess college and
career readiness. By understanding the key di↵erences between the digital and paper-pencil
SAT, students can adapt their preparation strategies and maximize their performance on the
new digital SAT format.

9
Chapter 2

Reading Question Types

2.1 Introduction

The SAT Reading section is designed to assess a student’s comprehension, vocabulary, analy-
sis, synthesis, and reasoning skills across various domains. The test focuses on four primary
domains: Craft and Structure, Information and Ideas, Standard English Conventions, and Ex-
pression of Ideas. Within these domains, ten specific skill areas are evaluated. A thorough
understanding of these domains and skills will help you develop e↵ective strategies for tackling
the SAT Reading section. In this chapter, we will provide an in-depth introduction to each
domain and their associated question types.

2.1.1 Domain: Craft and Structure

The Craft and Structure domain evaluates a student’s ability to understand and use high-utility
words and phrases, assess texts rhetorically, and establish connections between related texts.
There are three primary question types associated with this domain:

• Words in Context: These questions require students to determine the meaning of high-
utility academic words or phrases in context or to use them precisely and contextually
appropriate. To excel at these questions, students should expand their vocabulary and
develop skills for understanding words in context.
• Text Structure and Purpose: In this question type, students must analyze the struc-
ture of a text or determine its main rhetorical purpose. Developing a deep understanding
of various text structures and rhetorical purposes will enable students to approach these
questions e↵ectively.
• Cross-Text Connections: These questions assess a student’s ability to draw reasonable
connections between two topically related texts. To perform well on these questions,
students should practice identifying thematic and structural similarities between di↵erent
texts.

10
2.1.2 Domain: Information and Ideas

The Information and Ideas domain measures a student’s comprehension, analysis, and reasoning
skills, as well as their ability to locate, interpret, evaluate, and integrate information and ideas
from texts and informational graphics. This domain features three main question types:

• Central Ideas and Details: In these questions, students are required to determine the
central idea of a text and/or interpret the key details supporting that idea. To excel at
these questions, students should practice identifying central ideas and relevant details in
a variety of texts.

• Command of Evidence: These questions challenge students to use textual or quantita-


tive evidence to support, challenge, or respond to a specified claim or point. To succeed
in this question type, students must develop skills for finding and interpreting relevant
evidence within a text.

• Inferences: These questions assess a student’s ability to draw reasonable inferences


based on explicit and/or implicit information and ideas in a text. To perform well on
these questions, students should practice making logical inferences from various texts.

2.1.3 Domain: Standard English Conventions

The Standard English Conventions domain measures a student’s ability to edit texts to conform
to core conventions of Standard English sentence structure, usage, and punctuation. There are
two primary question types associated with this domain:

• Boundaries: These questions require students to edit text to ensure that sentences
are conventionally complete. To excel at these questions, students should familiarize
themselves with the rules of Standard English sentence structure.

• Form, Structure, and Sense: In this question type, students must edit text to conform
to conventional usage, such as agreement and verb tense/aspect. Developing a thorough
understanding of the rules governing Standard English usage will enable students to ap-
proach these questions e↵ectively.

2.1.4 Domain: Expression of Ideas

The Expression of Ideas domain measures a student’s ability to revise texts to improve

the e↵ectiveness of written expression and to meet specific rhetorical goals. This domain
features two main question types:

11
• Rhetorical Synthesis: These questions challenge students to strategically integrate
provided information and ideas on a topic to form an e↵ective sentence achieving a spec-
ified rhetorical aim. To excel at these questions, students should practice synthesizing
information from various sources to create coherent and persuasive sentences.

• Transitions: In this question type, students must determine the most e↵ective transition
word or phrase to logically connect information and ideas in a text. Developing a deep
understanding of various transition words and phrases and their appropriate usage will
enable students to approach these questions e↵ectively.

In conclusion, a thorough understanding of the four primary domains and their associated
question types is essential for success in the SAT Reading section. By familiarizing yourself
with these domains and question types, you can develop targeted strategies to improve your
performance and increase your overall score.

12
Chapter 3

Scoring 1600

3.1 The Ultimate Guide to Scoring 1600 on the SAT: Strate-


gies and Tips

Achieving a perfect score of 1600 on the SAT is an impressive feat that requires dedication,
strategy, and the right approach. In this comprehensive guide, we will cover step-by-step
strategies, tips, and tricks, as well as non-traditional methods for improving your SAT score,
no matter where you currently stand. We will delve into both the Math and Reading sections,
as well as general test-taking tips, to help you make the leap to that perfect 1600.

3.1.1 Building a Solid Foundation

To begin your SAT preparation, it’s essential to determine your baseline score. This refers to
the score you would achieve if you were to take the SAT without any prior preparation. To
identify your baseline score, take an official SAT practice test under realistic testing conditions,
including adhering to the time limits for each section. Your baseline score serves multiple
purposes:

Goal Setting: Knowing your baseline score will help you set realistic goals for your target
SAT score. This will allow you to gauge the amount of improvement needed and the time
required to reach that goal.

Progress Tracking: As you continue your preparation, taking periodic practice tests will
enable you to compare your scores with your baseline and track your progress. This will help
you adjust your study plan accordingly, focusing on areas that need improvement.

Motivation: Witnessing your progress from your baseline score can be a powerful motivator,
encouraging you to continue your e↵orts and maintain your focus on achieving your target score.
By understanding the SAT format and identifying your baseline score, you’re taking the first
critical steps toward SAT success. This knowledge will serve as a foundation for your study

13
plan and empower you to make the most of your preparation time.

3.1.2 Developing a Customized Study Plan

Developing a study schedule tailored to your current score and the time you have until the
test date is crucial for success. Allocate more time to areas where you struggle the most, and
maintain consistency in following your schedule. Break down your study time into manageable
sessions and ensure you include regular breaks to prevent burnout.

Selecting the best study materials is essential for e↵ective preparation. Some recommended
resources include:

Official SAT practice tests: These tests provide the most accurate representation of the
actual exam, enabling you to gauge your readiness and identify areas for improvement.

Online resources: Websites like Test Ninjas and Khan Academy o↵er a wealth of study
materials, including video lessons, interactive quizzes, and customized study plans.

Strategies for Each Score Range

If you scored 1200

Focus on understanding fundamental concepts and mastering the basics in both Reading and
Math.

Develop your reading comprehension skills by regularly reading diverse texts.

Familiarize yourself with the most common grammar rules.

Begin learning essential techniques.

If you scored 1300

Continue building on your foundational knowledge.

Identify weaknesses and work on specific content areas.

Develop test-taking strategies like time management and process of elimination.

If you scored 1400

Refine your strategies and focus on accuracy.

14
Work on time management and pacing.

Analyze your mistakes and learn from them.

If you scored 1500

Prioritize perfecting your weakest section.

Focus on the most challenging questions.

Develop resilience and a calm mindset for test day.

Make sure you are managing your time for each test.

3.1.3 If you scored 1550

Fine-tune your strategies and focus on eliminating careless errors.

Make sure you are particular careful and identify problem types that are challenging and
you miss easily.

Maintain your skills through regular practice.

Engage in mental preparation and stress management techniques.

3.2 Mastering Time Management Skills for SAT Success

In the quest for SAT success, e↵ective time management is a crucial skill. It’s not just about
answering the questions correctly but doing so within the allotted time. This chapter will delve
deep into the strategies, tools, and methods you can use to develop and practice your time
management skills for the SAT. Let’s take this journey step by step.

3.3 Tools for Time Management

3.3.1 Using a Computer

The digital age provides us with an array of tools to aid in time management. Online SAT
practice platforms are a great start. They can simulate the exact conditions of the test including
the ticking timer. This can help you acclimate to the pressure of the clock.

15
3.3.2 Time Tracking Software

Time tracking software can provide a detailed breakdown of how long you take per question or
per section. This can be crucial in identifying where you are spending too much or too little
time. Some recommended software includes Toggl and RescueTime.

3.4 Strategies for Time Management

3.4.1 The Skip and Return Strategy

Another e↵ective method is the ’skip and return’ strategy. If you encounter a difficult question,
instead of spending excessive time on it, you can skip it and return to it later. This ensures
you have enough time to tackle the easier questions and boosts your confidence.

3.4.2 The Section Division Strategy

Dividing the time for each section by the number of questions can give you a rough estimate of
how much time to spend on each question. For example, the reading section has 54 questions
and is 64 minutes long. This gives you approximately 70 seconds per question.

3.5 Practicing Time Management

3.5.1 Mock Tests

The best way to practice time management is by taking full-length timed mock tests. This will
give you a real sense of the test conditions and help you get used to the time pressure.

3.5.2 Gradual Time Reduction

Another practice method is to gradually reduce the time you allow yourself for each section
during your practice sessions. This creates a sense of urgency and trains you to think and
respond quickly.

3.5.3 Anxiety Management

Lastly, it’s important to manage exam-related anxiety, as it can often skew your perception of
time. Practice deep-breathing exercises, meditation, and other stress management techniques

16
to stay calm and focused during the test.

Remember, e↵ective time management is a skill that can be learned and improved with
practice. Use the strategies and tools discussed in this chapter and you’ll be well on your way
to mastering the clock on your SAT exam.

3.6 Non-Traditional Methods for Test Preparation

In addition to traditional study methods like repetition and rote memorization, there are sev-
eral non-traditional techniques that can enhance your test preparation. These techniques are
supported by cognitive science and can help improve comprehension, retention, and recall.

3.6.1 Mind Mapping

Mind mapping is a powerful tool for visualizing concepts and their relationships. This technique
involves creating a visual diagram that represents ideas and information in a structured way,
starting with a central concept and branching out into related topics.

In the context of Reading and Writing preparation, you can use mind maps to break down
complex passages, illustrate grammatical rules, or map out essay structures. This can help you
see the ”bigger picture” and improve your understanding and retention of the material.

3.6.2 Dual Coding

Dual coding is a learning strategy that involves combining verbal and visual representations of
information. According to the dual-coding theory proposed by Allan Paivio, our brain processes
visual and verbal information separately. Therefore, using both visual and verbal methods to
learn the same content can enhance understanding and recall.

For instance, you can accompany your verbal notes with visual elements such as diagrams,
charts, or infographics. This can be particularly helpful for understanding complex concepts or
relationships between topics.

3.6.3 Interleaved Practice

Interleaved practice involves mixing or alternating between di↵erent subjects or topics during
your study sessions, rather than focusing on one topic at a time (a method known as blocked
practice).

Research has shown that interleaved practice can enhance long-term learning and improve
the ability to transfer learned knowledge to new situations. This is because interleaving en-

17
courages the brain to discriminate between di↵erent types of problems and select the correct
strategy to solve them.

In the context of SAT preparation, you might alternate between Reading, Writing, and
Math during a single study session, rather than dedicating separate sessions to each subject.
This can help keep your mind fresh and avoid mental fatigue from focusing on one subject for
too long.

These non-traditional methods o↵er fresh approaches to test preparation. By visualizing


concepts, combining verbal and visual information, and mixing up your study topics, you can
improve your understanding, retention, and recall of the material, ultimately leading to better
test performance.

3.6.4 Test-Taking Tips

Time management: Allocate a specific amount of time per question, skip difficult questions,
and return to them later to maximize efficiency.

Process of elimination: Eliminate incorrect answer choices to increase your chances of se-
lecting the right answer.

Double-check your work: Revisit questions, especially those you were unsure about, to
ensure you haven’t made any careless mistakes. Review your calculations in the Math section
to confirm accuracy.

Maintain a steady pace: Find a balance between speed and accuracy to ensure you have
enough time to complete the test without sacrificing precision.

Reading strategies: Skim the passage first for a general understanding, then read more
carefully for details. Annotate key points and main ideas to stay engaged and facilitate com-
prehension. Practice active reading to improve focus and retention.

3.6.5 Test day preparation

The days leading up to the SAT, and the test day itself, can be filled with anxiety and stress.
Adequate preparation is essential for achieving the best possible outcome. In this section, we
will explore how to e↵ectively prepare for test day, covering the importance of sleep, nutrition,
gathering essential materials, dressing for comfort, and arriving at the test center early.

Get a Good Night’s Sleep: A good night’s sleep is crucial for optimal cognitive functioning.
Ensuring that you are well-rested before the test can significantly impact your ability to con-
centrate, make decisions, and recall information. Aim to establish a consistent sleep schedule
in the weeks leading up to the SAT, and prioritize sleep the night before the test.

Eat a Nutritious Breakfast: Eating a balanced and nutritious breakfast on test day pro-
vides the fuel your brain needs to function e↵ectively. Choose foods that are rich in complex

18
carbohydrates, proteins, and healthy fats to sustain energy levels throughout the exam. Avoid
sugary or overly processed foods, which can lead to energy crashes.

Dressing for Comfort: Wearing comfortable clothing on test day can help minimize distrac-
tions and allow you to focus on the exam. Dress in layers to accommodate potential fluctuations
in temperature at the testing center. Choose comfortable, loose-fitting clothes and supportive
footwear to maximize your comfort throughout the test.

3.7 Building Mental Resilience

Maintaining a strong and healthy mental state is crucial for achieving success on the SAT. Incor-
porating mindfulness meditation, visualization, positive affirmations, and stress management
techniques into your daily routine can significantly improve focus, reduce stress, and enhance
overall mental well-being. In this chapter, we will delve into each of these practices and discuss
how they can contribute to a more e↵ective SAT preparation experience.

3.8 Mindfulness Meditation

Mindfulness meditation is a practice that involves paying attention to the present moment,
without judgment. Regular practice of mindfulness meditation can lead to numerous benefits,
such as increased focus, reduced stress, and enhanced emotional regulation. Here are some
steps to begin practicing mindfulness meditation:

1. Find a quiet, comfortable space where you can sit or lie down without distractions.

2. Close your eyes and take several deep breaths, inhaling through your nose and exhaling
through your mouth.

3. Begin to focus on your breath, paying attention to the sensation of air entering and leaving
your body.

4. As thoughts arise, acknowledge them without judgment and gently bring your focus back
to your breath.

5. Practice mindfulness meditation for a few minutes each day, gradually increasing the
duration as you become more comfortable with the practice.

3.9 Visualization

Visualization is a powerful technique that involves creating vivid mental images of desired
outcomes or situations. By visualizing yourself achieving your goal of a perfect 1600 on the SAT,

19
as well as successfully tackling challenges during the test, you can boost your self-confidence
and motivation. To practice visualization:

1. Find a quiet, comfortable space where you can sit or lie down without distractions.

2. Close your eyes and take several deep breaths to relax your body and mind.

3. Begin to imagine yourself in the testing environment, feeling confident and prepared.

4. Visualize yourself calmly and accurately answering questions, managing your time e↵ec-
tively, and remaining focused throughout the test.

5. Picture yourself receiving your desired SAT score and experiencing the emotions associ-
ated with this accomplishment.

6. Practice visualization for a few minutes each day, ideally before or after your study ses-
sions.

3.10 Mastering the Art of Educated Guessing in Multiple-


Choice Questions

Multiple-choice questions are a common component of many tests. Although knowing the
correct answer is ideal, there may be times when you are unsure. In such scenarios, the ability
to make an educated guess can make a significant di↵erence. Here are some specific strategies
and tips to help you guess more e↵ectively:

3.10.1 The Process of Elimination

One of the most powerful strategies for guessing on multiple-choice questions is the process of
elimination. By ruling out the obviously incorrect answers, you can significantly increase your
chances of selecting the right one.

1. Identify and eliminate implausible answers: Read the question carefully and cross
out any answers that are clearly incorrect. Often, test makers will include one or two
options that are obviously wrong.

2. Compare remaining options: If there are two answers that seem correct, compare
them to see if they di↵er in a significant way. One may be more comprehensive or
accurate than the other.

20
3.10.2 Beware of Absolute Terms

Options that use words like ”always,” ”never,” ”all,” or ”none” can often be incorrect, as there
are very few absolutes in most subjects. Be cautious with these options and consider whether
the statement they make is truly absolute.

3.10.3 Choose the Longest or Most Detailed Answer

Sometimes, the longest or most detailed answer is the correct one. Test makers often ensure
that the correct answer is indisputably right, which can require more qualifying language or
details.

3.10.4 Trust Your First Instinct

Research has shown that your first answer choice is often the correct one, especially if you have
prepared well for the test. If you’re unsure, don’t change your answer unless you have a strong
reason to do so.

3.10.5 When All Else Fails, Guess

In most cases, leaving a multiple-choice question blank guarantees that you get it wrong. Even
if you have no idea what the correct answer is, it’s generally better to guess. You have a chance
of getting it right, and most testing systems do not penalize for incorrect answers more than
leaving the question blank.

Mastering the art of educated guessing in multiple-choice questions can significantly improve
your test performance. It is a skill that requires practice and strategic thinking, but when
employed properly, it can make the di↵erence between a good score and a great one. And
remember, an educated guess is always better than leaving a question unanswered.

21
3.11 Positive Affirmations

3.12 Harnessing the Power of Positive Affirmations for Test


Taking

3.12.1 Understanding Positive Affirmations

Positive affirmations are short, powerful statements that can change the way we think and
perceive ourselves. They are not just optimistic words, but psychological prompts aimed at
stimulating a positive attitude and self-confidence. The concept is grounded in neuroplasticity,
the idea that our brain has the ability to form new connections and pathways, changing how
we process and react to information.

3.12.2 The Role of Positive Affirmations in Test Preparation

When it comes to test preparation, especially high-stakes tests like SATs, positive affirmations
can be a crucial tool to combat stress, anxiety, and self-doubt. They can help you maintain
focus, boost your confidence, and foster a growth mindset that values e↵ort and learning over
innate ability. This can be especially beneficial in maintaining consistency and positivity during
your SAT preparation.

3.12.3 Incorporating Positive Affirmations into Test Preparation

1. Creating Affirmations: Begin by writing down a list of positive affirmations related to


your test preparation and goals. For example, ”I am capable of achieving a high score,” or
”I am a focused and disciplined learner.” Make sure your affirmations are in the present
tense, positive, and personal.

2. Repetition: Read through your list of affirmations each morning and evening, or any
time you need a boost of motivation. Consistent repetition will help solidify these positive
beliefs in your mind.

3. Visualization: As you repeat your affirmations, visualize yourself successfully reaching


your goals. This adds another layer of reinforcement and makes the affirmations more
e↵ective.

4. Practice: Make this technique a regular part of your test preparation routine. Over time,
these positive affirmations can help strengthen your self-belief and maintain a positive
mindset.

22
3.13 Conclusion

In conclusion, positive affirmations are a powerful tool that can help you achieve your test-
taking goals. By consciously choosing the words and beliefs that guide your preparation, you
can manage stress and build confidence. So start incorporating affirmations into your routine
and take a step towards a more confident and optimistic test preparation journey.

Achieving a perfect 1600 on the SAT is a challenging but attainable goal with dedication,
e↵ective strategies, and consistent e↵ort. By following this comprehensive guide, you will
be well-equipped to tackle the SAT with confidence and ultimately reach your target score.
Remember, improvement takes time, so be patient, stay persistent, and maintain a positive
attitude throughout your journey.

23
Part II

“How to Solve” Tutorials

24
Chapter 4

How to Solve ”Craft and Structure -


Word In Context”

4.1 Example “Craft and Structure - Word In Context” Ques-


tion

In a democratic society, the citizens are expected to participate in the political process.
Voting is the most fundamental form of civic engagement, but it is just one aspect of
being an active citizen. Citizens can also engage in public debates, join political parties,
or even run for public office. The health of a democracy is often gauged by the level of
its citizens’ in these various activities.

Which choice completes the text with the most logical and precise word or phrase?

A) abstention

B) participation

C) indi↵erence

D) rejection

Correct Answer:

B) participation

Explanation:

‘participation’ is the correct answer because it refers to the active involvement of citizens
in the political process, which is the focus of the passage.

25
Distractor Explanation:

A) ‘abstention’ is incorrect because it implies non-involvement or non-participation, which is


contrary to the message of the passage.

C) ‘indi↵erence’ is incorrect because it suggests that citizens do not care about the political
process, which is not the focus of the passage.

D) ‘rejection’ is incorrect because it implies that citizens actively oppose the political process,
which is not the focus of the passage.

4.2 How to solve “Craft and Structure - Word In Context”

“Craft and Structure - Word In Context” DSAT questions are designed to test your compre-
hension, vocabulary, analysis, synthesis, and reasoning skills. These questions require you to
determine the meaning of a high-utility academic word or phrase in context or use such vo-
cabulary in a precise, contextually appropriate way. In this tutorial, we will go through a
step-by-step process to solve these types of questions using the example provided.

Key Steps to Solving Word In Context Questions

1. Read the passage and question carefully with a focus on context

2. Identify the missing word or phrase and the surrounding context

3. Analyze the context of the missing word or phrase

4. Review the choices

5. Evaluate each choice in the context

6. Choose the most logical and precise word or phrase

Step 1: Read the passage and question carefully with a focus on context

Begin by reading the passage and question provided. As you read, pay close attention to the
context and main idea of the passage. Take note of any keywords, phrases, or themes that
might be relevant to the question.

In our example, the passage discusses the importance of citizen participation in a democratic
society and the various ways citizens can engage in the political process. Key themes include
democracy, citizen engagement, voting, and political activities.

26
After reading the passage, read the question and make sure you understand what it is asking.
In this case, the question is: ”Which choice completes the text with the most logical and precise
word or phrase?”

Step 2: Identify the missing word or phrase and the surrounding context

Next, locate the blank space in the passage where the missing word or phrase should be placed.
Carefully read the sentence containing the blank space, as well as the sentences immediately
before and after it, to understand the context in which the missing word or phrase must fit.

In our example, the blank space is located in the sentence: “The health of a democracy
is often gauged by the level of its citizens’ in these various activities.” The
surrounding context includes the previous sentence, which mentions the di↵erent forms of civic
engagement, such as voting, engaging in public debates, joining political parties, and running
for public office.

These extended steps provide you with a deeper understanding of the context in which the
missing word or phrase must fit. By focusing on the passage’s main idea, key themes, and
surrounding context, you’ll be better prepared to evaluate the choices provided and select the
most logical and precise word or phrase to complete the passage.

Step 3: Analyze the context of the missing word or phrase

Analyze the context around the blank space to understand what the missing word or phrase
should convey. In our example, the context suggests that the word or phrase should describe
the citizens’ involvement in the various political activities mentioned in the passage.

Step 4: Review the choices

Review the choices provided for the missing word or phrase. Consider each choice and whether
it fits logically and precisely in the context of the passage. In our example, we have the following
choices:

A) abstention

B) participation

C) indi↵erence

D) rejection

27
Step 5: Evaluate each choice in the context

Evaluate each choice by placing it in the blank space and determining whether it logically and
precisely completes the sentence in the context of the passage.

A) “The health of a democracy is often gauged by the level of its citizens’ abstention in
these various activities.”

This choice suggests that the health of a democracy is measured by citizens not participating
in political activities, which contradicts the passage’s emphasis on participation.

B) “The health of a democracy is often gauged by the level of its citizens’ participation in
these various activities.”

This choice fits the context, as it emphasizes the importance of citizen involvement in
political activities for a healthy democracy.

C) “The health of a democracy is often gauged by the level of its citizens’ indi↵erence in
these various activities.”

This choice suggests that the health of a democracy is measured by citizens’ lack of interest,
which also contradicts the passage’s emphasis on participation.

D) “The health of a democracy is often gauged by the level of its citizens’ rejection in these
various activities.”

This choice implies that the health of a democracy is measured by citizens’ rejection of
political activities, which again contradicts the passage’s emphasis on participation.

Step 6: Choose the most logical and precise word or phrase

Based on your evaluation in Step 5, choose the word or phrase that most logically and precisely
completes the passage. In our example, the correct answer is B) participation, as it emphasizes
the importance of citizen involvement in political activities for a healthy democracy.

By following these steps, you can e↵ectively solve ”Craft and Structure - Word In Context”
DSAT questions. Always remember to read the passage and question carefully, analyze the
context, evaluate the choices, and select the most logical and precise word or phrase to complete
the passage.

28
Chapter 5

How to Solve ”Craft and Structure - Text


Structure and Purpose”

5.1 Example “Craft and Structure - Text Structure and Pur-


pose” Question

In the early days of human civilization, the barter system was a common means of trade.
People exchanged goods and services directly, without any standardized currency. This
system had its limitations, as it relied on the double coincidence of wants and made it
difficult to store wealth. Over time, societies developed currency systems to facilitate
trade and simplify the exchange process.

Which choice best states the main idea of the text?

A) The barter system was the most efficient way for early societies to conduct
trade.

B) The development of currency systems led to the decline of human civilization.

C) The barter system had limitations that led to the development of currency
systems in societies.

D) Storing wealth was impossible in early human civilizations due to a lack of


currency systems.

Correct Answer:

C) The barter system had limitations that led to the development of currency systems in
societies.

Explanation:

29
‘The barter system had limitations that led to the development of currency systems in
societies.’ is the correct answer because the passage discusses the limitations of the barter
system and how societies developed currency systems to overcome these limitations.

Distractor Explanation:

A) ‘The barter system was the most efficient way for early societies to conduct trade.’ is
incorrect because the passage discusses the limitations of the barter system and how it
was eventually replaced by currency systems.

B) ‘The development of currency systems led to the decline of human civilization.’ is incorrect
because the passage does not discuss any negative impact of currency systems on human
civilization.

D) ‘Storing wealth was impossible in early human civilizations due to a lack of currency
systems.’ is incorrect because, while the passage does mention that storing wealth was
difficult with the barter system, it does not claim that it was impossible.

5.2 How to solve “Craft and Structure - Text Structure and


Purpose”

“Craft and Structure - Text Structure and Purpose” questions assess your ability to identify the
main idea, purpose, or structure of a given text. In this tutorial, we will go through a detailed
step-by-step process to solve such questions e↵ectively, using the example provided below.

Key Steps to Solving Text Structure and Purpose Questions

1. Read the passage carefully

2. Understand the question

3. Summarize the passage

4. Analyze each choice

5. Eliminate choices that don’t fit the main idea or purpose

6. Choose the choice that best states the main idea or purpose

7. Double-check your answer

30
Step 1: Read the passage carefully

Read the entire passage carefully to grasp the context, tone, and overall message the author is
trying to convey. Make a mental note of any important details or points made in the text. As you
read, pay close attention to the language used, the organization of ideas, and any arguments
or assertions made by the author. Consider the intended audience and the purpose of the
passage, as this can provide insights into the author’s tone and message. By actively engaging
with the text and analyzing its components, you can develop a comprehensive understanding
of the content and use this knowledge to better interpret, evaluate, or respond to the passage
in various contexts.

In our passage, we can see that it discusses the barter system, its limitations, and the
development of currency systems to overcome these limitations.

Step 2: Understand the question

Make sure you understand what the question is asking. In this case, the question is asking you
to identify the main idea of the text. To do this e↵ectively, carefully read the question and
any associated instructions or prompts. Take note of any keywords or phrases that can help
guide your response. It’s essential to address the specific requirements of the question, rather
than providing a general or unrelated answer. By focusing on the task at hand and aligning
your response with the question’s objectives, you can increase your chances of providing a
comprehensive and accurate answer that successfully demonstrates your understanding of the
text.

Step 3 Summarize the passage

Before analyzing the choices, try to summarize the passage in your own words. This will help
you better understand the main idea and make it easier to identify the correct answer.

The passage explains that the barter system was an early means of trade with limitations,
leading to the development of currency systems in societies.

Step 4 Analyze each choice

Go through the given choices one by one and analyze how well each of them aligns with the
main idea or purpose of the passage.

We have four choices:

A) The barter system was the most efficient way for early societies to conduct trade.

B) The development of currency systems led to the decline of human civilization.

31
C) The barter system had limitations that led to the development of currency systems in
societies.

D) Storing wealth was impossible in early human civilizations due to a lack of currency
systems.

Step 5: Eliminate choices that don’t fit the main idea or purpose

As you analyze each choice, eliminate options that do not fit the main idea or purpose of
the passage. This process of elimination will narrow down your options and make it easier to
identify the correct answer.

In our case, we can eliminate choices A, B, and D because they either misrepresent the
passage or focus on a specific detail instead of the overall main idea.

A) “The barter system was the most efficient way for early societies to conduct trade.”

This choice is incorrect because the passage does not claim that the barter system was the
most efficient way to conduct trade. Instead, it discusses the limitations of the barter system,
such as the reliance on the double coincidence of wants and difficulty in storing wealth.

B) “The development of currency systems led to the decline of human civilization.”

This choice is incorrect because the passage does not mention any negative consequences of
currency systems on human civilization. On the contrary, the passage explains that currency
systems were developed to facilitate trade and simplify the exchange process, suggesting that
their development was a positive advancement for societies.

D) “Storing wealth was impossible in early human civilizations due to a lack of currency
systems.”

This choice is incorrect because it overstates the impact of the lack of currency systems
on storing wealth. The passage mentions that the barter system made it ”difficult” to store
wealth, not “impossible.” Furthermore, this choice focuses on a specific detail about storing
wealth, rather than capturing the main idea of the passage, which is about the limitations of
the barter system and the development of currency systems.

Step 6: Choose the choice that best states the main idea or purpose

After eliminating options that don’t fit the main idea or purpose of the passage, select the
choice that best captures the main idea or purpose of the text.

Choice C (The barter system had limitations that led to the development of currency systems
in societies) best represents the main idea of the passage. Thus, we select it as the correct
answer.

32
Step 7: Double-check your answer

Once you’ve chosen an answer, reread the passage to ensure that your selected choice aligns
with the main idea or purpose of the text.

Re-reading the passage and comparing it with choice C, we can see that it accurately cap-
tures the main idea of the passage, confirming our choice as correct. This answer is correct
because it accurately captures the main idea of the passage. The passage discusses the limi-
tations of the barter system, such as its reliance on the double coincidence of wants and the
difficulty in storing wealth. These limitations are presented as the driving factors behind the
development of currency systems, which were designed to facilitate trade and simplify the ex-
change process. By addressing both the shortcomings of the barter system and the subsequent
adoption of currency systems, this answer choice provides a comprehensive summary of the
passage’s main idea.

By following these steps, you can e↵ectively solve “Craft and Structure - Text Structure
and Purpose” DSAT questions. Remember to read the passage carefully, analyze the choices,
eliminate options that don’t fit, and choose the choice that best captures the main idea or
purpose of the text. Practicing with various examples will help you hone your skills in solving
these types of questions.

33
Chapter 6

How to Solve “Craft and Structure -


Cross-Text Connections”

6.1 Example “Craft and Structure - Cross-Text Connections”


Question

Text 1: In the early 20th century, historian Samuel Freeman argued that the Renaissance
was primarily driven by economic factors, including the growth of trade and the rise of
wealthy merchant families. He posited that these forces led to the creation of a new,
secular culture that prized wealth, materialism, and individualism, ultimately paving the
way for the artistic and scientific innovations of the era.
Text 2: More recent scholarship by historian Lucy Wilson challenges Freeman’s view.
She contends that while economic factors certainly played a role, the Renaissance was
also strongly influenced by a resurgence of interest in classical learning, which inspired
intellectual curiosity, creativity, and the pursuit of knowledge for its own sake.

Based on the texts, how would Lucy Wilson (Text 2) most likely characterize the
conclusion presented in Samuel Freeman’s argument (Text 1)?

A) As a thorough explanation that accurately captures the complex factors behind


the Renaissance

B) As an outdated theory that has been entirely discredited by modern historical


scholarship

C) As an oversimplification that doesn’t fully acknowledge the importance of in-


tellectual factors

D) As a valid perspective but one that overstates the impact of secular culture on
the era’s innovations

34
Correct Answer:

C) As an oversimplification that doesn’t fully acknowledge the importance of intellectual


factors

Explanation:

‘As an oversimplification that doesn’t fully acknowledge the importance of intellectual


factors’ is the correct answer because Lucy Wilson (Text 2) argues that while economic
factors played a role in the Renaissance, the resurgence of interest in classical learning
was also a significant influence, which Freeman’s argument (Text 1) does not address.

Distractor Explanation:

A) ‘As a thorough explanation that accurately captures the complex factors behind the Re-
naissance’ is incorrect because Lucy Wilson (Text 2) challenges Samuel Freeman’s (Text
1) view by arguing that it doesn’t fully acknowledge the importance of intellectual factors.

B) ‘As an outdated theory that has been entirely discredited by modern historical scholarship’
is incorrect because Text 2 does not suggest that Freeman’s argument has been entirely
discredited, but rather that it is an oversimplification.

D) ‘As a valid perspective but one that overstates the impact of secular culture on the era’s
innovations’ is incorrect because Text 2 does not specifically argue that Freeman overstates
the impact of secular culture; it focuses on the role of classical learning.

6.2 How to solve “Craft and Structure - Cross-Text Connec-


tions”

“Craft and Structure - Cross-Text Connections” DSAT questions test your ability to draw
reasonable connections between two topically related texts. These questions require compre-
hension, vocabulary, analysis, synthesis, and reasoning skills. In this tutorial, we will walk
through an extended step-by-step process to solve these types of questions using the example
provided.

Key Steps to Solving Cross-Text Connection Questions

1. Read both texts and the question carefully while noting key themes and perspectives

2. Identify and analyze key points of agreement or disagreement between the texts

3. Review the choices provided and relate them to the texts

35
4. Evaluate each choice in relation to the texts and their points of agreement or disagreement

5. Choose the most appropriate answer based on the texts and your evaluation

Step 1: Read both texts and the question carefully while noting key themes
and perspectives

Begin by reading both texts, ensuring that you understand their main ideas, arguments, and
any sub-points. As you read, take note of any key themes, phrases, or perspectives that might
be relevant to the question. Pay attention to the authors’ tone, the evidence they present, and
the conclusions they draw.

In our example, Text 1 argues that the Renaissance was driven primarily by economic factors,
while Text 2 challenges this view, stating that intellectual factors also played a significant role.

After carefully reading the texts, read the question, ensuring you understand what it is
asking. In this case, the question is: “Based on the texts, how would Lucy Wilson (Text 2)
most likely characterize the conclusion presented in Samuel Freeman’s argument (Text 1)?”

Step 2: Identify and analyze key points of agreement or disagreement


between the texts

Analyze both texts to determine where they agree or disagree on the topic in question. Look
for areas where one author directly addresses the other’s argument or where they present
contrasting perspectives or evidence. Consider the relative weight each author gives to di↵erent
factors or ideas.

In our example, the main disagreement is between the role of economic factors (Text 1) and
the additional importance of intellectual factors (Text 2) in driving the Renaissance. Text 1
emphasizes the growth of trade and the rise of wealthy merchant families, while Text 2 highlights
the resurgence of interest in classical learning.

Step 3: Review the choices provided and relate them to the texts

Review the choices provided for the question. As you consider each choice, think about how it
might relate to the points of agreement or disagreement identified in Step 2. Reflect on whether
each choice accurately captures the authors’ perspectives and the relationship between their
arguments.

In our example, we have the following choices:

A) As a thorough explanation that accurately captures the complex factors behind the
Renaissance

36
B) As an outdated theory that has been entirely discredited by modern historical scholarship

C) As an oversimplification that doesn’t fully acknowledge the importance of intellectual


factors

D) As a valid perspective but one that overstates the impact of secular culture on the era’s
innovations

Step 4: Evaluate each choice in relation to the texts and their points of
agreement or disagreement

Evaluate each choice by considering how it reflects the relationship between the texts, partic-
ularly in terms of agreement or disagreement. Think about whether each choice accurately
represents the authors’ perspectives and the extent to which it captures the nuances of their
arguments.

In our example:

A) This choice implies that Lucy Wilson (Text 2) agrees with Samuel Freeman’s argument
(Text 1), but we know from Step 2 that Wilson challenges Freeman’s view by emphasizing the
importance of intellectual factors in addition to economic ones.

B) This choice suggests that Lucy Wilson (Text 2) completely discredits Samuel Freeman’s
argument (Text 1), but she only challenges it by adding the importance of intellectual factors,
not by entirely dismissing Freeman’s argument.

C) This choice reflects the disagreement identified in Step 2, as Lucy Wilson (Text 2) argues
that Samuel Freeman’s argument (Text 1) oversimplifies the factors behind the Renaissance by
not fully acknowledging the importance of intellectual factors.

D) This choice does not accurately represent the disagreement between the texts, as Text
2 focuses on the role of classical learning and its influence on the Renaissance, not the impact
of secular culture. While it acknowledges that Freeman’s perspective may be valid, it does not
directly address the main point of disagreement identified in Step 2.

Step 5: Choose the most appropriate answer based on the texts and your
evaluation

Based on your evaluation in Step 4, choose the answer that best represents the relationship
between the texts in terms of agreement or disagreement. Consider which choice most accurately
captures the authors’ perspectives and the nuances of their arguments.

In our example, the correct answer is C) As an oversimplification that doesn’t fully acknowl-
edge the importance of intellectual factors, as it accurately reflects Lucy Wilson’s challenge to
Samuel Freeman’s argument.

37
By following these extended steps, you can e↵ectively solve “Craft and Structure - Cross-
Text Connections” DSAT questions. Remember to read both texts and the question carefully,
noting key themes and perspectives; identify and analyze points of agreement or disagreement;
review the choices provided and relate them to the texts; evaluate each choice in relation to the
texts and their points of agreement or disagreement; and select the most appropriate answer
based on the relationship between the texts.

38
Chapter 7

How to Solve “Information and Ideas -


Central Ideas and Details”

7.1 Example “Craft and Structure - Cross-Text Connections”


Question

In the late 19th century, the British Empire experienced rapid expansion, which led to
the development of new technologies for transportation and communication. Among
these innovations was the Penny Black, the world’s first adhesive postage stamp. Issued
in 1840, the Penny Black streamlined the postal system, making it more accessible and
a↵ordable for the public. This stamp facilitated communication across vast distances,
reinforcing the connections within the empire and contributing to its growth.

Which choice best states the main purpose of the text?

A) To explain the role of the Penny Black in improving the postal system and
fostering communication within the British Empire

B) To provide a detailed history of the British Empire’s expansion and the various
technological innovations that resulted from it

C) To argue that the Penny Black was the most significant innovation in the history
of the British Empire and its communication systems

D) To discuss the various means of communication available during the 19th cen-
tury and how they impacted the British Empire’s expansion

Correct Answer:

A) To explain the role of the Penny Black in improving the postal system and fostering
communication within the British Empire

39
Explanation:

‘To explain the role of the Penny Black in improving the postal system and fostering
communication within the British Empire’ is the correct answer because the passage
focuses on the introduction of the Penny Black, its impact on the postal system, and its
role in enhancing communication within the empire.

Distractor Explanation:

B) ‘To provide a detailed history of the British Empire’s expansion and the various tech-
nological innovations that resulted from it’ is incorrect because the passage only briefly
mentions the expansion and focuses primarily on the Penny Black.

C) ‘To argue that the Penny Black was the most significant innovation in the history of the
British Empire and its communication systems’ is incorrect because the passage does not
make comparisons or argue for the superiority of the Penny Black.

D) ‘To discuss the various means of communication available during the 19th century and how
they impacted the British Empire’s expansion’ is incorrect because the passage primarily
discusses the Penny Black, rather than providing a broader overview of 19th-century
communication methods.

7.2 How to solve “Information and Ideas - Central Ideas and


Details”

“Information and Ideas - Central Ideas and Details” DSAT questions test your ability to de-
termine the central idea of a text and interpret the key details supporting that idea. These
questions require comprehension, analysis, and reasoning skills to locate, interpret, evaluate,
and integrate information and ideas from texts. In this tutorial, we will walk through an
extended step-by-step process to solve these types of questions using the example provided.

Key Steps to Solving Cross-Text Connection Questions

1. Read the passage carefully, noting the central idea and supporting details

2. Understand the question and its focus

3. Review the choices provided, relating them to the passage and the question

4. Evaluate each choice in relation to the passage and the focus of the question

5. Select the most appropriate answer based on the passage and your evaluation

40
Step 1: Read the passage carefully, noting the central idea and supporting
details

Begin by reading the passage carefully, ensuring that you understand the main idea and any
supporting details. As you read, take note of the primary topic, the author’s perspective, and
any key points or evidence that support the central idea.

In our example, the passage discusses the introduction of the Penny Black, the world’s first
adhesive postage stamp, and its impact on the postal system and communication within the
British Empire during the 19th century.

Step 2: Understand the question and its focus

After reading the passage, read the question carefully, making sure you understand what it is
asking. Focus on the specific aspect of the text the question addresses, such as the central idea,
a supporting detail, or the author’s purpose.

In this case, the question asks, “Which choice best states the main purpose of the text?”

Step 3: Review the choices provided, relating them to the passage and
the question

Review the choices provided for the question. As you consider each choice, think about how it
might relate to the central idea and supporting details you identified in Step 1, as well as the
focus of the question from Step 2. Keep in mind that some choices may contain elements of
truth but might not be the best answer to the specific question being asked.

In our example, we have the following choices:

A) To explain the role of the Penny Black in improving the postal system and fostering
communication within the British Empire

B) To provide a detailed history of the British Empire’s expansion and the various techno-
logical innovations that resulted from it

C) To argue that the Penny Black was the most significant innovation in the history of the
British Empire and its communication systems

D) To discuss the various means of communication available during the 19th century and
how they impacted the British Empire’s expansion

41
Step 4: Evaluate each choice in relation to the passage and the focus of
the question

Evaluate each choice by considering how it reflects the passage’s central idea, supporting details,
and the specific focus of the question. Keep in mind that the best answer should accurately
capture the main purpose of the text and align with the evidence provided in the passage.

In our example:

A) This choice accurately reflects the passage’s focus on the Penny Black’s role in improving
the postal system and fostering communication within the British Empire.

B) This choice addresses the British Empire’s expansion and technological innovations, but
the passage primarily focuses on the Penny Black and its impact, not a detailed history of the
empire’s expansion.

C) This choice claims that the Penny Black was the most significant innovation, but the
passage does not make comparisons or argue for its superiority over other innovations.

D) This choice discusses various means of communication during the 19th century, while the
passage primarily focuses on the Penny Black and its specific impact on the British Empire’s
postal system and communication.

Step 5: Select the most appropriate answer based on the passage and your
evaluation

Based on your evaluation in Step 4, choose the answer that best addresses the focus of the ques-
tion and accurately reflects the central idea and supporting details of the passage. Remember
to consider the specific context of the question and the evidence provided in the text.

In our example, the correct answer is A) To explain the role of the Penny Black in improving
the postal system and fostering communication within the British Empire. This choice best
captures the main purpose of the text and aligns with the information presented in the passage.

In our example, the question asks, “Which choice best states the main purpose of the text?”
We will analyze each choice and explain why it is correct or incorrect based on the passage.

A) “To explain the role of the Penny Black in improving the postal system and fostering
communication within the British Empire.”

Correct Answer: This choice accurately reflects the passage’s focus on the introduction of
the Penny Black, its impact on the postal system, and its role in enhancing communication
within the British Empire. The passage provides evidence that the Penny Black made the postal
system more accessible and a↵ordable, which facilitated communication across vast distances
and reinforced connections within the empire.

B) “To provide a detailed history of the British Empire’s expansion and the various tech-

42
nological innovations that resulted from it.”

Incorrect Answer: While the passage does mention the rapid expansion of the British Empire
and the development of new technologies, it does not provide a detailed history of this expansion
or discuss various technological innovations in depth. Instead, the passage primarily focuses on
the Penny Black and its impact on the postal system and communication.

C) “To argue that the Penny Black was the most significant innovation in the history of
the British Empire and its communication systems.”

Incorrect Answer: The passage does not argue for the superiority of the Penny Black or
compare it to other innovations. It does highlight the importance of the Penny Black in im-
proving the postal system and fostering communication within the British Empire, but it does
not claim that it was the most significant innovation in the empire’s history.

D) “To discuss the various means of communication available during the 19th century and
how they impacted the British Empire’s expansion.”

Incorrect Answer: This choice incorrectly broadens the scope of the passage. While the
passage does discuss the impact of the Penny Black on the British Empire’s communication
and its role in the empire’s growth, it does not provide an overview of various communication
methods available during the 19th century. The passage’s primary focus is on the Penny Black
and its specific impact on the postal system and communication within the British Empire.

By examining each choice in relation to the passage and the question’s focus, we can de-
termine that choice A is the correct answer, as it best captures the main purpose of the text
and aligns with the information presented in the passage. Remember to read the passage and
question carefully, noting the central idea and supporting details; understand the focus of the
question; review and relate the choices to the passage and the question; evaluate each choice in
relation to the passage and the question’s focus; and select the most appropriate answer based
on your evaluation and the evidence from the text. By practicing these steps, you will build
your skills and confidence in tackling similar questions on the DSAT.

43
Chapter 8

How to Solve “Information and Ideas -


Command of Evidence”

8.1 Example “Information and Ideas - Command of Evi-


dence” Question

A recent study found that a city’s public transport accessibility is linked to its residents’
quality of life. The researchers believe that improved transportation can contribute to
higher levels of happiness and well-being in urban areas.

Which finding, if true, would most directly support the researchers’ belief?

A) Cities with expanded public transportation systems report higher levels of over-
all satisfaction among their residents.

B) Some residents in cities with limited public transportation options choose to


walk or cycle to work.

C) Cities with higher GDP per capita tend to have better public transportation
systems.

D) Tourists prefer cities with well-developed public transportation networks.

Correct Answer:

A) Cities with expanded public transportation systems report higher levels of overall satis-
faction among their residents.

Explanation:

‘Cities with expanded public transportation systems report higher levels of overall sat-

44
isfaction among their residents.’ is the correct answer because it directly supports the
researchers’ belief that improved transportation can contribute to higher levels of happi-
ness and well-being in urban areas.

Distractor Explanation:

B) ‘Some residents in cities with limited public transportation options choose to walk or
cycle to work.’ is incorrect because it doesn’t address the relationship between public
transportation and residents’ happiness.

C) ‘Cities with higher GDP per capita tend to have better public transportation systems.’
is incorrect because it doesn’t show a direct link between improved transportation and
residents’ well-being.

D) ‘Tourists prefer cities with well-developed public transportation networks.’ is incorrect


because it focuses on tourists’ preferences rather than the impact of public transportation
on residents’ happiness.

8.2 How to solve “Information and Ideas - Command of Ev-


idence”

The “Information and Ideas - Command of Evidence” section of the DSAT assesses a student’s
ability to comprehend, analyze, and reason using textual and quantitative evidence. In this
tutorial, we will walk you through a step-by-step process for solving these types of questions
using the example provided above. This guide will help you understand how to locate, inter-
pret, evaluate, and integrate information and ideas from texts and informational graphics to
e↵ectively answer these questions.

Key Steps to Information and Ideas - Command of Evidence

1. Read the passage carefully

2. Understand the question

3. Identify the researchers’ belief

4. Review the answer choices

5. Evaluate each answer choice

6. Eliminate irrelevant answer choices

45
Step 1: Read the passage carefully

Begin by reading the passage provided in the question. It is essential to have a clear under-
standing of the passage’s main idea, as it forms the basis for the question that follows. In our
example, the passage states: “A recent study found that a city’s public transport accessibility
is linked to its residents’ quality of life. The researchers believe that improved transportation
can contribute to higher levels of happiness and well-being in urban areas.”

Step 2: Understand the question

Read the question carefully and make sure you understand what it is asking. In our example,
the question is: “Which finding, if true, would most directly support the researchers’ belief?”

Step 3: Identify the researchers’ belief

To answer the question, you need to identify the researchers’ belief stated in the passage. In
this case, the belief is: ”improved transportation can contribute to higher levels of happiness
and well-being in urban areas.”

Step 4: Review the answer choices

Read through all of the answer choices provided. Take note of each option and consider how it
relates to the researchers’ belief.

In our example, we have the following choices:

A) Cities with expanded public transportation systems report higher levels of overall satis-
faction among their residents.

B) Some residents in cities with limited public transportation options choose to walk or
cycle to work.

C) Cities with higher GDP per capita tend to have better public transportation systems.

D) Tourists prefer cities with well-developed public transportation networks.

Step 5: Evaluate each answer choice

Go through each answer choice and determine whether it directly supports the researchers’
belief or not. To do this, consider the following questions:

46
Does the answer choice provide evidence that improved transportation contributes to higher
levels of happiness and well-being?

Does the answer choice focus on the impact of public transportation on residents’ happiness
and well-being?

Step 6: Eliminate irrelevant answer choices

Based on your evaluation, eliminate answer choices that do not directly support the researchers’
belief. In our example, the following options can be eliminated:

B) This option focuses on alternative transportation methods and does not address the
relationship between public transportation and residents’ happiness.

C) This option discusses the correlation between GDP per capita and public transportation
but does not provide a direct link between improved transportation and residents’ well-being.

D) This option highlights tourists’ preferences rather than the impact of public transporta-
tion on residents’ happiness.

Step 7: Choose the correct answer

After eliminating irrelevant options, you should be left with one answer choice that directly
supports the researchers’ belief. In our example, the correct answer is “A) Cities with expanded
public transportation systems report higher levels of overall satisfaction among their residents.”
This answer directly supports the researchers’ belief by showing a correlation between improved
transportation and higher levels of happiness and well-being in urban areas.

In the given example, the question asks: “Which finding, if true, would most directly support
the researchers’ belief?” The researchers’ belief, as stated in the passage, is that “improved
transportation can contribute to higher levels of happiness and well-being in urban areas.”
Let’s analyze each answer choice in detail:

Correct Answer:

A) ”Cities with expanded public transportation systems report higher levels of overall sat-
isfaction among their residents.”

This answer directly supports the researchers’ belief because it provides evidence that there
is a correlation between improved transportation (expanded public transportation systems)
and higher levels of happiness and well-being (overall satisfaction) in urban areas. By showing
that cities with better public transportation have more satisfied residents, this answer choice
reinforces the researchers’ claim.

Incorrect Answers:

47
B) “Some residents in cities with limited public transportation options choose to walk or
cycle to work.”

This answer choice is incorrect because it does not address the relationship between public
transportation and residents’ happiness. Instead, it focuses on alternative transportation meth-
ods (walking or cycling) chosen by some residents in cities with limited public transportation.
While it may provide information on how people cope with limited transportation options, it
does not provide evidence that supports the researchers’ belief regarding the impact of improved
transportation on happiness and well-being.

C) “Cities with higher GDP per capita tend to have better public transportation systems.”

This answer choice is incorrect because it does not directly link improved transportation to
residents’ well-being. It only establishes a correlation between higher GDP per capita and better
public transportation systems. While it might suggest that wealthier cities can a↵ord better
transportation infrastructure, it does not demonstrate how this improvement in transportation
leads to higher levels of happiness and well-being among residents.

D) “Tourists prefer cities with well-developed public transportation networks.”

This answer choice is incorrect because it focuses on the preferences of tourists rather than
the impact of public transportation on residents’ happiness and well-being. While a well-
developed public transportation network may attract more tourists, it does not provide evidence
to support the researchers’ belief that improved transportation contributes to higher levels of
happiness and well-being for the people living in those cities.

In summary, answer choice A is the correct answer because it directly supports the re-
searchers’ belief by showing a correlation between improved public transportation and higher
levels of happiness and well-being among urban residents. The other answer choices are in-
correct because they either do not address the relationship between public transportation and
residents’ happiness or focus on unrelated aspects like alternative transportation methods, GDP
per capita, or tourists’ preferences.

48
Chapter 9

How to Solve “Information and Ideas -


Inferences”

9.1 Example “Inferences” Question

Abstract Expressionism, an art movement that emerged in the mid-20th century, is


known for its focus on spontaneous, emotional, and gestural art. Artists like Jackson
Pollock and Willem de Kooning gained prominence through their innovative techniques
and unique styles. The movement’s emphasis on individuality and self-expression
suggests that Abstract Expressionist artists may have been seeking to .

Which choice most logically completes the text?

A) break away from traditional artistic norms and conventions

B) emulate the styles of earlier European art movements

C) promote a specific political or social agenda

D) create artwork that is easily understood by the general public

Correct Answer:

A) break away from traditional artistic norms and conventions

Explanation:

‘break away from traditional artistic norms and conventions’ is the correct answer be-
cause it aligns with the Abstract Expressionist movement’s focus on individuality, self-
expression, and innovative techniques.

49
Distractor Explanation:

B) ’emulate the styles of earlier European art movements’ is incorrect because the movement
sought to create new and unique styles rather than imitating past movements.

C) ‘promote a specific political or social agenda’ is incorrect because the movement was
primarily focused on individual expression rather than political or social issues.

D) ‘create artwork that is easily understood by the general public’ is incorrect because the
movement’s emphasis on emotional and gestural art does not necessarily make the artwork
easily understood.

9.2 How to solve “Information and Ideas - Inferences”

“Information and Ideas - Inferences” section of the DSAT assesses a student’s ability to draw
reasonable inferences based on explicit and implicit information and ideas in a text. In this
tutorial, we will guide you through a step-by-step process for solving these types of questions
using the example provided above. This guide will help you develop your skills in locating,
interpreting, evaluating, and integrating information and ideas from texts and informational
graphics to answer these questions e↵ectively.

The “Information and Ideas” domain assesses a student’s ability to comprehend, analyze,
and reason with the information presented in various texts and informational graphics. This
domain measures how well a student can locate, interpret, evaluate, and integrate information
and ideas from written passages or visual representations of data, such as tables, bar graphs,
and line graphs. This skill set is essential for understanding, evaluating, and making connections
between di↵erent pieces of information presented in various formats.

“Inferences” questions require students to make logical conclusions or deductions based on


the information provided in the text. These questions evaluate a student’s ability to under-
stand explicit information (directly stated facts) and implicit information (suggested or hinted
at) within a passage. The purpose of this question type is to measure a student’s capacity
to think critically and draw logical conclusions from the information provided, even when the
answer is not explicitly stated in the text. This skill is crucial for making connections be-
tween di↵erent pieces of information, understanding the author’s intention or viewpoint, and
identifying underlying themes or messages within a passage.

Key Steps to Solving Cross-Text Connection Questions

1. Read the passage carefully

2. Understand the question

3. Identify the main ideas and context

50
4. Review the answer choices

5. Evaluate each answer choice

Step 1: Read the passage carefully

Begin by reading the passage provided in the question. It is crucial to have a clear understanding
of the passage’s main idea, as it forms the basis for the question that follows. In our example,
the passage states: ”Abstract Expressionism, an art movement that emerged in the mid-20th
century, is known for its focus on spontaneous, emotional, and gestural art. Artists like Jackson
Pollock and Willem de Kooning gained prominence through their innovative techniques and
unique styles. The movement’s emphasis on individuality and self-expression suggests that
Abstract Expressionist artists may have been seeking to .”

Step 2: Understand the question

Read the question carefully and make sure you understand what it is asking. In our example,
the question is: ”Which choice most logically completes the text?”

Step 3: Identify the main ideas and context

To answer the question, you need to identify the main ideas and context from the passage. In
this case, the main ideas are the characteristics of the Abstract Expressionist movement, such
as its focus on spontaneous, emotional, and gestural art, as well as its emphasis on individuality
and self-expression.

The context is the emergence of the movement in the mid-20th century and the innovative
techniques and unique styles of its prominent artists.

Step 4: Review the answer choices

Read through all of the answer choices provided. Take note of each option and consider how it
relates to the main ideas and context identified in the passage.

Step 5: Step 5: Evaluate each answer choice

Go through each answer choice and determine whether it logically completes the text by con-
sidering the main ideas and context from the passage.

To do this, ask yourself the following questions:

51
Does the answer choice align with the characteristics of the Abstract Expressionist move-
ment?

Does the answer choice make sense within the context of the passage?

Step 6: Eliminate irrelevant answer choices

Based on your evaluation, eliminate answer choices that do not logically complete the text. In
our example, the following options can be eliminated:

B) This option is irrelevant because the movement sought to create new and unique styles
rather than imitating past movements.

C) This option is not consistent with the passage’s focus on individual expression, as it
suggests that the artists were primarily concerned with political or social issues.

D) This option does not align with the characteristics of the Abstract Expressionist move-
ment, as its emphasis on emotional and gestural art does not necessarily make the artwork
easily understood.

Step 7: Choose the correct answer

After eliminating irrelevant options, you should be left with one answer choice that most logi-
cally completes the text. In our example, the correct answer is “A: break away from traditional
artistic norms and conventions.” This answer aligns with the Abstract Expressionist move-
ment’s focus on individuality, self-expression, and innovative techniques, making it the most
logical choice to complete the text.

In the given example, the question asks: “Which choice most logically completes the text?”
The passage discusses the Abstract Expressionist movement, its characteristics, and the artists
associated with it.

Let’s analyze each answer choice in detail:

Correct Answer:

A) “break away from traditional artistic norms and conventions”

This answer choice is correct because it logically completes the text by aligning with the
characteristics of the Abstract Expressionist movement, such as its focus on spontaneous, emo-
tional, and gestural art, as well as its emphasis on individuality and self-expression. The
movement’s innovative techniques and unique styles suggest that the artists were attempting
to break away from traditional artistic norms and conventions, making this the most logical
choice to complete the text.

Incorrect Answers: B) “emulate the styles of earlier European art movements”

52
This answer choice is incorrect because it contradicts the information presented in the
passage. The passage emphasizes the innovative techniques and unique styles of the Abstract
Expressionist artists, which implies that they were not seeking to emulate the styles of earlier
European art movements. Instead, they aimed to create new and distinctive styles that set
them apart from past movements.

C) “promote a specific political or social agenda”

This answer choice is incorrect because it does not align with the characteristics of the
Abstract Expressionist movement as described in the passage. The movement’s focus on in-
dividuality and self-expression suggests that the artists were more concerned with personal
expression rather than promoting a specific political or social agenda. While some artists
within the movement may have had political or social motivations, this was not the primary
focus of the movement as a whole.

D) “create artwork that is easily understood by the general public”

This answer choice is incorrect because it does not align with the characteristics of the
Abstract Expressionist movement. The passage describes the movement as focusing on spon-
taneous, emotional, and gestural art, which does not necessarily make the artwork easily un-
derstood by the general public. Instead, the emphasis on individuality and self-expression
suggests that the artists were more interested in creating unique and innovative artwork rather
than artwork that was easily understood by a wide audience.

In summary, answer choice A is the correct answer because it most logically completes
the text by aligning with the characteristics of the Abstract Expressionist movement and the
context provided in the passage. The other answer choices are incorrect because they either
contradict the information in the passage, do not align with the movement’s characteristics, or
do not make sense within the context of the passage.

53
Chapter 10

How to Solve “Standard English


Conventions - Boundaries”

10.1 Example “Standard English Conventions - Boundaries”


Question

The rare, brightly colored peacock mantis shrimp can deliver a blow to its prey with a
force more than 2,500 times its own body weight. The shrimp’s powerful strike is due to
the unique structure of its which stores and then releases large amounts
of energy quickly.

Which choice completes the text so that it conforms to the conventions of Standard
English?

A) appendage,

B) appendage;

C) appendage:

D) appendage

Correct Answer:

A) appendage,

Explanation:

‘appendage,’ is the correct answer because it maintains a smooth flow of the sentence,
conforming to Standard English conventions, and provides a proper separation between
the two related ideas.

54
Distractor Explanation:

B) ‘appendage;’ is incorrect because the semicolon is not needed to separate the two related
ideas in this context.

C) ‘appendage:’ is incorrect because the colon is not needed to separate the two related ideas
in this context.

D) ‘appendage’ is incorrect because the lack of punctuation between ’appendage’ and ’which’
disrupts the flow of the sentence and does not conform to Standard English conventions.

10.2 How to solve “Standard English Conventions - Bound-


aries”

The “Standard English Conventions - Boundaries” section of the DSAT assesses a student’s
ability to edit text to ensure that sentences are conventionally complete, conforming to core
conventions of Standard English sentence structure, usage, and punctuation. In this tutorial,
we will guide you through a step-by-step process for solving these types of questions using the
example provided above. This guide will help you develop your editing skills and knowledge
to make text conform to core conventions of Standard English sentence structure, usage, and
punctuation.

The “Standard English Conventions” domain assesses a student’s ability to edit and revise
text to ensure it adheres to the core conventions of Standard English in terms of sentence
structure, usage, and punctuation. This skill is essential for clear and e↵ective written commu-
nication, as it ensures that the text is easily understandable and follows grammatical rules and
conventions.

The “Boundaries” question type focuses on a student’s ability to edit text in a way that
ensures sentences are complete and conform to the conventions of Standard English. This
involves the proper use of punctuation marks, such as commas, semicolons, and colons, as
well as the appropriate separation and connection of clauses and phrases within sentences. By
mastering this skill, students can e↵ectively communicate their ideas in writing and ensure that
their text is easily understandable and grammatically correct.

Key Steps to Standard English Conventions - Boundaries Question

1. Read the passage carefully

2. Understand the question

3. Review the answer choices

4. Evaluate each answer choice

55
5. Eliminate irrelevant answer choices

6. Choose the correct answer

Step 1: Read the passage carefully

Begin by reading the passage provided in the question. It is crucial to have a clear understanding
of the passage’s main idea, as it forms the basis for the question that follows. In our example,
the passage states: “The rare, brightly colored peacock mantis shrimp can deliver a blow to its
prey with a force more than 2,500 times its own body weight. The shrimp’s powerful strike is
due to the unique structure of its which stores and then releases large amounts
of energy quickly.”

Step 2: Understand the question

Read the question carefully and make sure you understand what it is asking. In our example,
the question is: “Which choice completes the text so that it conforms to the conventions of
Standard English?”

Step 3: Review the answer choices

Read through all of the answer choices provided. Take note of each option and consider how it
relates to the main ideas and context identified in the passage.

Step 4: Evaluate each answer choice

Go through each answer choice and determine whether it conforms to the conventions of Stan-
dard English by considering the main ideas and context from the passage. To do this, ask
yourself the following questions:

Does the answer choice fit the sentence structure and maintain a smooth flow?

Does the answer choice use proper punctuation and conform to Standard English conven-
tions?

Step 5: Eliminate irrelevant answer choices

Based on your evaluation, eliminate answer choices that do not conform to the conventions of
Standard English. In our example, the following options can be eliminated:

56
B) This option uses a semicolon, which is not needed to separate the two related ideas in
this context.

C) This option uses a colon, which is not needed to separate the two related ideas in this
context.

D) This option lacks punctuation between “appendage” and “which,” disrupting the flow
of the sentence and not conforming to Standard English conventions.

Step 6: Choose the correct answer

After eliminating irrelevant options, you should be left with one answer choice that most logi-
cally completes the text and conforms to the conventions of Standard English. In our example,
the correct answer is “A) appendage,” because it maintains a smooth flow of the sentence,
conforms to Standard English conventions, and provides proper separation between the two
related ideas.

In the given example, the question asks: “Which choice completes the text so that it con-
forms to the conventions of Standard English?” The passage discusses the peacock mantis
shrimp and the unique structure responsible for its powerful strike. Let’s analyze each answer
choice in detail:

Correct Answer:

A) “appendage,” This answer choice is correct because it uses a comma to maintain a smooth
flow of the sentence and conforms to Standard English conventions. The comma provides a
proper separation between the two related ideas within the sentence, making it the best option
to complete the text in accordance with the conventions of Standard English.

Incorrect Answers:

B) “appendage;” This answer choice is incorrect because it uses a semicolon, which is


not needed to separate the two related ideas in this context. A semicolon is typically used to
separate two independent clauses that are closely related in meaning. However, in this sentence,
the second part of the sentence is a dependent clause, and a semicolon is not the appropriate
punctuation mark to use here.

C) “appendage:” This answer choice is incorrect because it uses a colon, which is not needed
to separate the two related ideas in this context. A colon is generally used to introduce a list,
an explanation, or an elaboration of the preceding clause. In this sentence, a colon is not the
appropriate punctuation mark to use because the second part of the sentence is a dependent
clause that provides additional information rather than an explanation or elaboration.

D) “appendage”

This answer choice is incorrect because it lacks punctuation between “appendage” and
“which.” The absence of punctuation disrupts the flow of the sentence and does not conform

57
to Standard English conventions. A punctuation mark, such as a comma, is needed to separate
the two related ideas and maintain the sentence’s coherence.

In summary, answer choice A is the correct answer because it uses a comma to separate the
two related ideas, maintaining a smooth flow of the sentence and conforming to Standard En-
glish conventions. The other answer choices are incorrect because they either use inappropriate
punctuation marks or lack punctuation altogether, causing disruptions in the sentence’s flow
or structure.

Solving “Standard English Conventions - Boundaries” DSAT questions requires a thorough


understanding of the passage, the question, and the answer choices. By carefully reading the
passage, identifying the main ideas and context, evaluating each answer choice, and eliminat-
ing irrelevant options, you can successfully choose the correct answer that conforms to the
conventions of Standard English sentence structure, usage, and punctuation.

58
Chapter 11

How to Solve “Standard English


Conventions - Form Structure, and Sense”

11.1 Example “Standard English Conventions - Form Struc-


ture, and Sense” Question

In the 19th century, the development of the railroad system a significant


impact on the American economy, leading to increased trade and industrial growth.

Which choice completes the text so that it conforms to the conventions of Standard
English?

A) had

B) having

C) have

D) will have

Correct Answer:

A) had

Explanation:

“had’ is the correct answer because it provides a finite past tense verb that agrees with
the subject and fits the historical context of the sentence.

Distractor Explanation:

59
B) ‘having’ is incorrect because it is a nonfinite verb form (a present participle) that doesn’t
agree with the subject.

C) “have’ is incorrect because it is a present tense verb that does not fit the historical context
of the sentence.

D) ‘will have’ is incorrect because it is a future tense verb that does not fit the historical
context of the sentence.

11.2 How to solve “Standard English Conventions - Form,


Structure, and Sense”

In the “Standard English Conventions” domain, ”Form, Structure, and Sense” questions assess
a student’s ability to edit text to conform to conventional usage, such as subject-verb agreement,
verb tense, and aspect. This skill helps ensure clear and e↵ective written communication by
adhering to the grammatical rules and conventions of Standard English. In this tutorial, we
will go through a step-by-step process to solve “Form, Structure, and Sense” DSAT questions
using the provided example.

The “Standard English Conventions” domain assesses a student’s ability to edit and revise
text to ensure it adheres to the core conventions of Standard English in terms of sentence
structure, usage, and punctuation. This skill is essential for clear and e↵ective written commu-
nication, as it ensures that the text is easily understandable and follows grammatical rules and
conventions.

The “Form, Structure, and Sense” question type assesses a student’s ability to edit text to
ensure it conforms to conventional usage in terms of agreement, verb tense, and aspect. This
involves selecting the correct verb form that fits the context of the sentence, ensuring subject-
verb agreement, and making other grammatical corrections as necessary. By mastering this
skill, students can improve their written communication and ensure their text is grammatically
correct and easily understandable.

Key Steps to Standard English Conventions - Form, Structure, and Sense


Question

1. Read and analyze the passage and question

2. Choose the correct answer based on context and grammatical rules

60
Step 1: Read and analyze the passage and question

Begin by carefully reading the given passage and question. Identify the part of the sentence that
requires editing, and note any contextual clues that may help you choose the correct answer.
In the given example, the passage discusses the development of the railroad system in the 19th
century and its impact on the American economy. The question asks, ”Which choice completes
the text so that it conforms to the conventions of Standard English?”

Step 2: Evaluate each answer choice

Go through each answer choice one by one and consider how it a↵ects the sentence’s form, struc-
ture, and sense. Determine if each choice adheres to conventional usage in terms of agreement,
verb tense, and aspect.

In our example, focus on the verb forms provided in the answer choices:

A) “had” B) “having” C) “have” D) “will have”

Step 3: Choose the correct answer based on context and grammatical


rules

Use the contextual clues from the passage and your understanding of the grammatical rules to
select the answer that best completes the sentence according to Standard English conventions.

In the given example, the passage discusses an event that happened in the 19th century, so
the correct verb form should be in the past tense.

A) “had” is a past tense verb that agrees with the subject and fits the historical context
of the sentence. B) “having” is a nonfinite verb form (a present participle) that doesn’t agree
with the subject. C) “have” is a present tense verb that does not fit the historical context of
the sentence. D) “will have” is a future tense verb that does not fit the historical context of
the sentence.

Based on this analysis, the correct answer is A) “had”.

To solve “Form, Structure, and Sense” DSAT questions in the Standard English Conventions
domain, carefully read and analyze the passage and question, evaluate each answer choice, and
select the correct answer based on context and grammatical rules. By following this step-by-
step process, you can improve your ability to edit text to conform to conventional usage and
enhance your overall written communication skills.

61
62
Chapter 12

How to Solve ”Expression of Ideas -


Rhetorical Synthesis”

12.1 Example ”Expression of Ideas - Rhetorical Synthesis”


Question

While researching a topic, a student has taken the following notes:

• The Doppler e↵ect is the change in frequency or wavelength of a wave in relation


to an observer who is moving relative to the wave source.

• The Doppler e↵ect is observed in both sound and light waves.

• The Doppler e↵ect is named after Austrian physicist Christian Doppler, who first
proposed the phenomenon in 1842.

• The Doppler e↵ect has practical applications in fields such as astronomy, meteorol-
ogy, and medical imaging.

The student wants to explain the importance of the Doppler e↵ect. Which choice most
e↵ectively uses relevant information from the notes to accomplish this goal?

A) The Doppler e↵ect, first proposed by Austrian physicist Christian Doppler in


1842, is a phenomenon observed in both sound and light waves.

B) The Doppler e↵ect is named after Christian Doppler,who first proposed the
phenomenon in 1842, and it has applications in various scientific fields.

C) The Doppler e↵ect, observed in sound and light waves, has significant applica-
tions in diverse fields such as astronomy, meteorology, and medical imaging.

D) The Doppler e↵ect involves the change in frequency or wavelength of a wave


and was first proposed by Christian Doppler in 1842.

63
Correct Answer:

C) The Doppler e↵ect, observed in sound and light waves, has significant applications in
diverse fields such as astronomy, meteorology, and medical imaging.

Explanation:

‘The Doppler e↵ect, observed in sound and light waves, has significant applications in
diverse fields such as astronomy, meteorology, and medical imaging.’ is the correct answer
because it uses relevant information from the notes to emphasize the importance of the
Doppler e↵ect by mentioning its applications in various fields.

Distractor Explanation:

A) ‘The Doppler e↵ect, first proposed by Austrian physicist Christian Doppler in 1842, is
a phenomenon observed in both sound and light waves.’ is incorrect because it doesn’t
emphasize the importance of the Doppler e↵ect.

B) ‘The Doppler e↵ect is named after Christian Doppler, who first proposed the phenomenon
in 1842, and it has applications in various scientific fields.’ is incorrect because it focuses
on Christian Doppler instead of the importance of the Doppler e↵ect itself.

D) ‘The Doppler e↵ect involves the change in frequency or wavelength of a wave and was
first proposed by Christian Doppler in 1842.’ is incorrect because it doesn’t mention the
importance of the Doppler e↵ect and its applications in various fields.

12.2 How to solve “Expression of Ideas - Rhetorical Synthe-


sis”

The “Expression of Ideas” domain assesses a student’s ability to revise texts to improve the
e↵ectiveness of written expression and to meet specific rhetorical goals. Within this domain,
the ”Rhetorical Synthesis” skill focuses on a student’s ability to strategically integrate provided
information and ideas on a topic to form an e↵ective sentence that achieves a specified rhetorical
aim. The ability to synthesize information is essential for e↵ective communication in both
academic and professional settings.

Keys to Solving Rhetorical Synthesis Questions

1. Read the passage carefully and identify the key information provided about the topic.

2. Read the question carefully and identify the rhetorical aim.

64
3. Review each answer choice and evaluate how e↵ectively it uses the relevant information
to accomplish the rhetorical aim.

4. Select the most e↵ective answer choice that synthesizes the information to accomplish the
rhetorical aim.

Step 1: Read the passage carefully and identify the key information pro-
vided about the topic.

To e↵ectively synthesize information and answer the given question, it is essential to carefully
read the passage provided. As you read, try to identify the key points and information provided
about the topic at hand. For example, in this passage, the topic is the Doppler e↵ect, and the
author provides the definition of the phenomenon, its applications in various fields, and its
historical origins.

To make sure you understand the passage, you can annotate or take notes as you read. This
can include underlining or highlighting important phrases, circling key words, or writing notes
in the margin. By engaging with the passage in this way, you can better retain the information
provided and more e↵ectively synthesize it to answer the given question.

Step 2: Read the question carefully and identify the rhetorical aim.

After reading the passage and taking note of the relevant information, it is important to carefully
read the question and identify the rhetorical aim. This aim will guide your answer and ensure
that you e↵ectively synthesize the information provided in the passage to accomplish a specific
purpose. For example, in this question, the aim is to explain the importance of the Doppler
e↵ect.

To identify the rhetorical aim, read the question carefully and take note of key phrases that
indicate the desired outcome. In this case, the phrase ”explain the importance” signals that
the aim is to provide information on the significance of the Doppler e↵ect. It is important to
keep this aim in mind as you review the answer choices and select the most e↵ective option. By
staying focused on the rhetorical aim, you can ensure that your answer e↵ectively synthesizes
the relevant information to accomplish the desired outcome.

Step 3: Review each answer choice and evaluate how e↵ectively it uses
the relevant information to accomplish the rhetorical aim.

Once you have identified the key information in the passage and the rhetorical aim of the
question, it’s time to evaluate each answer choice. Read each option carefully and evaluate
how e↵ectively it synthesizes the relevant information from the passage to accomplish the

65
desired outcome. In this case, you are looking for an answer choice that e↵ectively explains the
importance of the Doppler e↵ect by drawing on the information provided in the passage.

To evaluate each answer choice, consider how well it aligns with the information provided
in the passage and how e↵ectively it accomplishes the desired rhetorical aim. Does the answer
choice provide a clear and concise explanation of the importance of the Doppler e↵ect, drawing
on the information provided in the passage? Does it e↵ectively synthesize the relevant informa-
tion to accomplish the rhetorical aim? Consider these questions as you evaluate each answer
choice to determine which one is the most e↵ective.

Step 4: Select the most e↵ective answer choice that synthesizes the in-
formation to accomplish the rhetorical aim.

After carefully evaluating each answer choice, select the option that most e↵ectively synthesizes
the relevant information to accomplish the rhetorical aim. In this case, the correct answer choice
is C, which emphasizes the importance of the Doppler e↵ect by mentioning its applications in
diverse fields, such as astronomy, meteorology, and medical imaging.

In this DSAT question, the goal is to choose the answer choice that e↵ectively uses the rele-
vant information from the passage to accomplish the rhetorical aim of explaining the importance
of the Doppler e↵ect.

Let’s take a look at each answer choice and evaluate its e↵ectiveness:

A) “The Doppler e↵ect, first proposed by Austrian physicist Christian Doppler in 1842, is
a phenomenon observed in both sound and light waves.”

This answer choice e↵ectively synthesizes the relevant information from the passage by
mentioning Christian Doppler and the fact that the Doppler e↵ect is observed in both sound and
light waves. However, it doesn’t e↵ectively accomplish the desired rhetorical aim of explaining
the importance of the Doppler e↵ect. It simply provides information about what the Doppler
e↵ect is and who discovered it.

B) “The Doppler e↵ect is named after Christian Doppler, who first proposed the phe-
nomenon in 1842, and it has applications in various scientific fields.”

This answer choice is similar to choice A in that it mentions Christian Doppler and the fact
that the Doppler e↵ect has applications in various scientific fields. However, it also doesn’t
e↵ectively accomplish the desired rhetorical aim of explaining the importance of the Doppler
e↵ect. It simply provides information about who discovered the Doppler e↵ect and where it is
used.

C) “The Doppler e↵ect, observed in sound and light waves, has significant applications in
diverse fields such as astronomy, meteorology, and medical imaging.”

This answer choice is the most e↵ective because it synthesizes the relevant information from
the passage to e↵ectively accomplish the desired rhetorical aim of explaining the importance of

66
the Doppler e↵ect. By mentioning that the Doppler e↵ect has significant applications in diverse
fields such as astronomy, meteorology, and medical imaging, it emphasizes the importance of
the Doppler e↵ect and how it is used in various scientific fields.

D: “The Doppler e↵ect involves the change in frequency or wavelength of a wave and was
first proposed by Christian Doppler in 1842.”

This answer choice e↵ectively synthesizes the relevant information from the passage by
mentioning Christian Doppler and what the Doppler e↵ect is. However, it doesn’t e↵ectively
accomplish the desired rhetorical aim of explaining the importance of the Doppler e↵ect. It
simply provides information about what the Doppler e↵ect is and who discovered it, without
emphasizing its importance or applications.

Overall, answer choice C is the most e↵ective because it synthesizes the relevant infor-
mation to accomplish the rhetorical aim of explaining the importance of the Doppler e↵ect.
Answer choices A, B, and D are all incorrect because while they may synthesize some relevant
information, they do not e↵ectively accomplish the rhetorical aim of the question.

It’s important to remember that the correct answer choice should not only align with the in-
formation provided in the passage, but it should also e↵ectively accomplish the desired rhetorical
aim. By staying focused on the rhetorical aim and evaluating each option based on its e↵ec-
tiveness in achieving that aim, you can more e↵ectively synthesize the information to answer
the given question.

67
Chapter 13

How to Solve “Expression of Ideas -


Transitions”

13.1 Example “Expression of Ideas - Transitions” Question

For many years, scientists believed that Thomas Edison’s invention of the incandescent
light bulb in 1879 was the first practical electric light. While Edison’s invention was
a significant breakthrough, British scientist Sir Hiram Maxim had
actually developed a working electric light in 1878, a year before Edison.

Which choice completes the text with the most logical transition?

A) Additionally,

B) Consequently,

C) Moreover,

D) However,

Correct Answer:

D) However

Explanation:

‘However’ is the correct answer because it logically signals a contrast between the belief
that Edison invented the first practical electric light and the fact that Sir Hiram Maxim
developed one a year earlier.

Distractor Explanation:

68
A) ’Additionally’ is incorrect because it illogically implies that Sir Hiram Maxim’s inven-
tion adds to the belief that Edison invented the first practical electric light, rather than
contrasting it.

B) ’Consequently’ is incorrect because it illogically suggests that Maxim’s invention is a result


of the belief that Edison invented the first practical electric light.

C) ’Moreover’ is incorrect because it illogically implies that Maxim’s invention simply adds
more information to the belief that Edison invented the first practical electric light, rather
than contrasting it.

13.2 How to solve “Expression of Ideas - Transition”

In this tutorial, we will be discussing how to solve ”Expression of Ideas - Transitions” DSAT
questions. These types of questions require students to determine the most e↵ective transition
word or phrase to logically connect information and ideas in a text. The correct transition will
help to make the text more cohesive and help the reader to follow the author’s argument.

The Expression of Ideas domain measures the ability to revise texts to improve the e↵ec-
tiveness of written expression and to meet specific rhetorical goals. The domain includes three
skills: Transitions, Rhetorical Synthesis, and Vocabulary.

Transitions is one of the three skills tested in the Expression of Ideas domain. This skill
requires students to determine the most e↵ective transition word or phrase to logically connect
information and ideas in a text. Transitions are words or phrases that connect ideas, sentences,
and paragraphs, helping to guide the reader through the text and make the writing more
coherent and easy to follow. E↵ective transitions help to create a smooth flow of ideas and
prevent the text from becoming disjointed or confusing.

In a Transitions DSAT question, students are presented with a passage and a question that
asks them to choose the most e↵ective transition word or phrase to complete the passage. The
answer choices usually include commonly used transition words and phrases, such as ”however,”
”therefore,” ”in addition,” and ”moreover.” Students must carefully consider the context of the
passage and the relationship between the ideas presented to choose the most e↵ective transition
word or phrase.

Overall, the Transitions skill in the Expression of Ideas domain measures students’ ability to
e↵ectively connect ideas and create a coherent and logical flow of information in their writing.

English transition words are words and phrases that connect and relate ideas and sentences
in a text, making it easier for readers to follow the writer’s train of thought. Here is a list of
common transition words and phrases with explanations of their usage:

• Addition: Used to add information or ideas.

– Additionally: ”Additionally, I would like to thank everyone who helped me.”

69
– Also: ”I am also interested in learning more about the topic.”
– Furthermore: ”Furthermore, the research has shown that the problem is more com-
plex than initially thought.”
– In addition: ”In addition to his academic achievements, he was also an accomplished
musician.”

• Contrast: Used to show di↵erences or opposite ideas.

– However: ”I understand your point of view; however, I disagree with your argument.”
– Nevertheless: ”The team was tired; nevertheless, they continued to work hard.”
– On the other hand: ”The company has a lot of money, but on the other hand, they
have a lot of problems.”
– Yet: ”The problem is serious; yet, no one seems to be taking any action.”

• Cause and E↵ect: Used to show the reason for an event or the result of an action.

– Consequently: ”He missed the train; consequently, he arrived late to the meeting.”
– Due to: ”Due to the weather conditions, the flight was canceled.”
– For this reason: ”For this reason, we decided to postpone the project.”
– Therefore: ”He was absent from work; therefore, he missed an important meeting.”

• Comparison: Used to compare or show similarities.

– Similarly: ”Similarly, many people are concerned about the environmental impact
of the project.”
– Likewise: ”She enjoys cooking; likewise, she likes to experiment with new recipes.”
– In the same way: ”In the same way, the company’s profits have been declining for
the past few years.”
– Just as: ”Just as she was about to leave, the phone rang.”

• Time: Used to show the order of events or time relationships.

– Meanwhile: ”He was at work; meanwhile, his wife was at home taking care of their
children.”
– Next: ”Next, we will discuss the results of the study.”
– Afterward: ”Afterward, he realized that he had made a mistake.”
– Later: ”She left early; later, she regretted not staying longer.”

• Conclusion: Used to summarize or conclude.

– In conclusion: ”In conclusion, I believe that we need to take action to address the
issue.”
– Therefore: ”The data supports our hypothesis; therefore, we can conclude that it is
correct.”

70
– To summarize: ”To summarize, the project was successful due to the e↵orts of
everyone involved.”
– Ultimately: ”Ultimately, the success of the business depends on the satisfaction of
our customers.”

Key Steps to Solving Transition Reading Questions

1. Read the passage carefully and identify the information that needs to be connected with
a transition.

2. Look at the available transition choices and consider which one best conveys the intended
meaning of the passage.

3. Consider the distractor explanations to understand why the incorrect answers are not the
best choice.

4. Double-check your selection and ensure that the transition e↵ectively connects the ideas
in the passage.

Step 1: Read the passage carefully and identify the information that needs
to be connected with a transition.

In this example, the first sentence discusses the commonly held belief that Thomas Edison’s
incandescent light bulb was the first practical electric light. The second sentence provides
information that contrasts with this belief, stating that British scientist Sir Hiram Maxim
actually developed a working electric light a year earlier. To e↵ectively connect these two
ideas, we need to choose a transition that indicates the contrast between the two.

Step 2: Look at the available transition choices and consider which one
best conveys the intended meaning of the passage.

In this case, the correct answer is ”However.” This transition e↵ectively signals a contrast
between the two sentences, indicating that the second sentence contradicts the commonly held
belief presented in the first sentence.

Step 3: Consider the distractor explanations to understand why the incor-


rect answers are not the best choice.

In this example, the distractors ”Additionally,” ”Consequently,” and ”Moreover” are all incor-
rect. ”Additionally” and ”Moreover” suggest that the second sentence adds more information

71
to the first sentence rather than contrasting it, while ”Consequently” suggests that the second
sentence is a result of the first sentence. By understanding why the distractors are incorrect,
you can more easily identify the correct answer.

Step 4: Double-check your selection and ensure that the transition e↵ec-
tively connects the ideas in the passage.

In this example, the transition ”However” e↵ectively contrasts the commonly held belief in the
first sentence with the information provided in the second sentence. By taking the time to
double-check your answer, you can ensure that you have selected the most e↵ective transition
for the passage.

Let’s take a look at each answer choice and evaluate its e↵ectiveness:

A) ”Additionally,” is incorrect because it suggests that Maxim’s invention adds to the idea
that Edison invented the first practical electric light. This is not the case as Maxim’s invention
is a contrast to the belief that Edison was the first.

B) ”Consequently,” is incorrect because it implies that the idea of Edison being the first
led to Maxim’s invention. This is not logical, and there is no evidence to support this.

C) ”Moreover,” is incorrect because it suggests that Maxim’s invention is just another


example of Edison’s work, rather than a contrast to it. This is not the case as it contradicts
the belief that Edison was the first inventor of practical electric light.

D) “However,” is the correct answer because it logically indicates a contrast between the
belief that Edison invented the first practical electric light and the fact that Sir Hiram Maxim
developed one a year earlier. It emphasizes the discrepancy between what scientists believed
and what actually happened, providing a logical transition between the two ideas.

Overall, the correct answer emphasizes the contrast between the commonly held belief and
the historical fact, while the incorrect options either add irrelevant information, imply causation
where there is none, or fail to highlight the contrast between the two ideas.

“Expression of Ideas - Transitions” questions can be tricky, but by carefully considering each
choice and using context clues from the passage, you can identify the most e↵ective transition
word or phrase. Remember to choose the transition that logically connects the two ideas and
makes the passage more cohesive.

13.2.1 Additional Transition Word List

• Additionally - indicating an added element or circumstance

• Also - indicating that something is true in addition to what was already stated

• Furthermore - indicating an additional supporting point

72
• Moreover - indicating that something is true in addition to what was already stated

• Likewise - indicating that something is similar to what was previously stated

• Similarly - indicating that something is similar to what was previously stated

• However - indicating a contrast or exception to what was previously stated

• Nonetheless - indicating a contrast or exception to what was previously stated

• Nevertheless - indicating a contrast or exception to what was previously stated

• On the other hand - indicating a contrast or alternative

• In contrast - indicating a contrast or di↵erence

• Alternatively - indicating an alternative option or possibility

• Alternatively, also, as well - indicating an additional or alternative option or possibility

• In addition - indicating an additional element or circumstance

• Notwithstanding - indicating a contrast or exception to what was previously stated

• In fact - indicating that something is actually true despite what may have been assumed

• Indeed - emphasizing that something is true or significant

• In summary - indicating a brief overview of what was previously discussed

• To summarize - indicating a brief overview of what was previously discussed

• Therefore - indicating a conclusion or consequence based on what was previously stated

• Consequently - indicating a conclusion or consequence based on what was previously


stated

• Thus - indicating a conclusion or consequence based on what was previously stated

• Hence - indicating a conclusion or consequence based on what was previously stated

• As a result - indicating a conclusion or consequence based on what was previously stated

• Accordingly - indicating a conclusion or consequence based on what was previously


stated

• For example - introducing an illustrative example to support a point

• Specifically - indicating a specific example or detail to support a point

• In particular - indicating a specific example or detail to support a point

• To illustrate - introducing an illustrative example to support a point

73
• Namely - introducing a specific or explicit detail or example

• In other words - indicating an alternative or clearer way to express what was previously
stated

• That is - indicating an explanation or clarification of what was previously stated

• Put another way - indicating an alternative or clearer way to express what was previ-
ously stated

• To put it simply - indicating a simplified or clearer way to express what was previously
stated

• As previously stated - referring back to a previously stated point

• As mentioned earlier - referring back to a previously mentioned point

• To conclude - indicating a final statement or conclusion

• Finally - indicating a final point or conclusion

• In conclusion - indicating a final statement or conclusion

• Lastly - indicating a final point or conclusion

74
Part III

Practice Modules

75
Chapter 14

Module 1

Reading and Writing - 27 Questions

DIRECTIONS: The questions in this section are designed to test your reading and writing
abilities. Each question will provide you with one or more passages. It is important to
thoroughly read each passage and question before selecting the best answer based on the
information provided. All of the questions in this section are multiple-choice and will have
four options to choose from. There is only one correct answer for each question.

1. The Congress of Vienna, held from 1814 to 1815, aimed to restore Europe’s balance
of power after the French Revolution and Napoleonic Wars. It was noteworthy for its
diplomatic approach, which prioritized negotiation and consensus-building over military
force. The Congress reestablished the pre-Napoleonic order by restoring monarchies and
redrawing national borders. This reconfiguration of power sought to maintain long-term
stability and prevent future conflicts in Europe.

Which choice best states the main purpose of the text?

A) To emphasize the role of individual diplomats in shaping the decisions made at the
Congress of Vienna
B) To discuss the various military strategies employed during the Congress of Vienna to
achieve lasting peace in Europe
C) To describe the Congress of Vienna’s objectives and its diplomatic approach in restor-
ing Europe’s balance of power
D) To analyze the long-term consequences of the Congress of Vienna on European po-
litical ideologies

76
2. The English Civil War (1642-1651) was a turning point in British history, marked by a
conflict between the Parliamentarians and the Royalists. At the heart of the war was
a struggle for supremacy between King Charles I and the English Parliament, fueled by
religious di↵erences and financial disputes. The war ended in 1651 with the decisive
Parliamentarian victory at the Battle of Worcester, leading to the eventual execution of
Charles I and the establishment of the English Commonwealth under Oliver Cromwell.

Which choice best states the main purpose of the text?

A) To discuss the military tactics used during the English Civil War and their impact
on modern warfare
B) To provide a brief overview of the English Civil War, highlighting the key parties
involved and its outcome
C) To examine the role of religion in the English Civil War and its influence on contem-
porary religious conflicts
D) To analyze the political ramifications of the English Civil War and its e↵ect on the
British monarchy

3. Text 1: Literary critic Hannah Miller examines Robert Browning’s dramatic monologue
’My Last Duchess.’ Miller argues that the poem’s protagonist, the Duke, is an unreliable
narrator who seeks to manipulate the reader’s perception of his dead wife. She contends
that the Duke’s description of his wife’s flirtatious behavior is likely exaggerated to justify
his cruel actions.

Text 2: In contrast, historian Alexander Thompson explores the historical context of ’My
Last Duchess,’ focusing on the real-life figures that may have inspired Browning’s poem.
Thompson suggests that Browning’s portrayal of the Duke and his wife could be more
accurate than previously thought, as it aligns with historical accounts of the figures in
question.

Based on the texts, how would Alexander Thompson (Text 2) most likely respond to
Hannah Miller’s argument about the Duke’s unreliability (Text 1)?

A) By pointing out that the Duke’s portrayal may be more accurate than Miller suggests,
as it aligns with historical accounts of real-life figures
B) By agreeing with Miller’s assertion and suggesting that Browning intentionally created
an unreliable narrator to critique aristocratic power
C) By arguing that Miller’s interpretation is solely based on a close reading of the poem
without considering the historical context

77
D) By suggesting that Miller’s argument is invalid because the poem should be read as
a pure work of fiction without any connection to reality

4. Canoeing, a popular recreational activity, involves paddling a canoe with a single-bladed


paddle. Canoers often enjoy exploring calm waterways, but more experienced canoeists
may challenge themselves by through rougher waters or participating in canoe
racing events.

Which choice completes the text so that it conforms to the conventions of Standard
English?

A) to navigate
B) navigating
C) having navigated
D) navigate

5. Table tennis, a fast-paced sport with origins in England, requires players to have quick
reflexes and strong hand-eye coordination. The International Table Tennis Federation
(ITTF) was established in 1926, and since then, the sport to gain worldwide
popularity and recognition.

Which choice completes the text so that it conforms to the conventions of Standard
English?

A) continue
B) continuing
C) has continued
D) had continued

78
6. Kitesurfing is a thrilling water sport that combines elements of sailing, wakeboarding, and
gymnastics. With the assistance of a large kite, kitesurfers harness the wind’s power to
propel themselves across the water and perform impressive aerial maneuvers. To main-
tain control, kitesurfers must carefully adjust the kite’s angle in relation to the wind by
the control bar.

Which choice completes the text so that it conforms to the conventions of Standard
English?

A) manipulated
B) to manipulate
C) manipulating
D) manipulate

7. J.K. Rowling, the world-renowned author of the Harry Potter series, faced numerous
challenges before achieving success. She was a single mother on welfare and her manuscript
was rejected by many publishers before it a chance with Bloomsbury Publishing.

Which choice completes the text so that it conforms to the conventions of Standard
English?

A) found
B) finding
C) to find
D) had found

79
8. While researching a topic, a student has taken the following notes:

• Mickey Mouse is a fictional character created by Walt Disney and Ub Iwerks in 1928.
• Mickey Mouse made his first appearance in the animated short film Steamboat Willie.
• Mickey Mouse became a popular symbol of the Walt Disney Company and remains
an iconic figure.
• Mickey Mouse has been featured in numerous films, television shows, and comic
strips.

Which choice most e↵ectively uses relevant information from the notes to explain the
significance of Mickey Mouse in popular culture?

A) Mickey Mouse was created by Walt Disney and Ub Iwerks and debuted in the film
Steamboat Willie.
B) Mickey Mouse has been featured in various forms of media since his creation in 1928.
C) Mickey Mouse is a fictional character that has been around for many years.
D) As an iconic figure and symbol of the Walt Disney Company, Mickey Mouse has
played a significant role in popular culture through various films, television shows,
and comic strips.

9. While researching a topic, a student has taken the following notes:

• Behavioral economics combines psychology and economics to study decision-making.


• Prospect Theory, developed by Kahneman and Tversky, is a key concept in behav-
ioral economics.
• Loss aversion, a principle in Prospect Theory, states that people feel losses more
intensely than gains.
• Nudge Theory, proposed by Thaler and Sunstein, suggests using indirect suggestions
to influence decision-making.

Which choice most e↵ectively uses relevant information from the notes to describe the
relationship between Prospect Theory and behavioral economics?

A) Loss aversion, a central idea in Nudge Theory, is essential to understanding behavioral


economics.
B) Kahneman and Tversky developed Nudge Theory, which is a significant aspect of
behavioral economics.
C) Behavioral economics focuses on decision-making, while Prospect Theory deals with
gains and losses.

80
D) Prospect Theory, which includes the principle of loss aversion, is a key concept in the
study of behavioral economics.

10. While researching a topic, a student has taken the following notes:

• The Pomodoro Technique is a time management method developed by Francesco


Cirillo.
• It involves breaking work into 25-minute intervals called ’Pomodoros’ followed by a
5-minute break.
• After completing four Pomodoros, a longer break of 15-30 minutes is taken.
• The technique helps maintain focus and prevent burnout.

Which choice most e↵ectively uses relevant information from the notes to explain the
Pomodoro Technique?

A) The Pomodoro Technique was developed by Francesco Cirillo to manage time by


taking breaks.
B) Francesco Cirillo’s Pomodoro Technique involves 25-minute work intervals and various
breaks.
C) The Pomodoro Technique aims to prevent burnout by taking breaks between work
periods.
D) The Pomodoro Technique divides work into 25-minute intervals with short breaks,
and after four intervals, a longer break is taken.

81
11. Aristotle, a Greek philosopher and polymath, was a student of Plato and later became
the teacher of Alexander the Great. His writings cover a wide range of subjects, including
physics, biology, zoology, metaphysics, logic, ethics, aesthetics, poetry, theater, music,
and rhetoric. Aristotle’s work in the field of logic laid the foundation for that
still exists today.

Which choice completes the text so that it conforms to the conventions of Standard
English?

A) a system of thinking
B) a systems of thinking
C) a system of thinkings
D) a system’s of thinking

12. Walt Whitman, a prominent American poet, is best known for his poetry collection
’Leaves of Grass.’ First published in 1855, the collection underwent several revisions
throughout Whitman’s life, ultimately expanding from twelve poems to over Whit-
man’s work celebrates the individual and the collective human experience, often delving
into themes of democracy, love, and nature.

Which choice completes the text so that it conforms to the conventions of Standard
English?

A) four hundreds
B) four hundred’s
C) four hundred
D) four hundreds’

82
13. The price of gold has always been a popular topic among investors. In times of economic
uncertainty, investors often turn to gold as a safe haven because of its perceived stability.
However, the price of gold can also be influenced by factors such as inflation, interest
rates, and the of other commodities.

Which choice completes the text so that it conforms to the conventions of Standard
English?

A) performance
B) performance’
C) performance’s
D) performances

14. Mahatma Gandhi, a prominent leader in the Indian independence movement, was known
for his practice of nonviolent resistance. His strategy of peaceful protest, called Satya-
graha, was successfully used to the British rule in India and inspired many other
nonviolent movements worldwide.

Which choice completes the text with the most logical and precise word or phrase?

A) endorse
B) ignore
C) challenge
D) emulate

83
15. In infrastructure engineering, the use of prefabricated components has become increas-
ingly popular. These components, which are produced o↵-site and then transported to
the construction location, can provide significant benefits, such as reduced construction
time, increased sustainability, and potentially lower costs. However, the implementation
of prefabricated components can be in certain situations, such as those involving
complex or highly specialized structures.

Which choice completes the text with the most logical and precise word or phrase?

A) negligible
B) inevitable
C) challenging
D) repetitive

16. O↵shoring has become a common business practice for companies seeking to reduce opera-
tional costs. By relocating certain functions to countries with lower labor costs, companies
can achieve significant cost savings. However, it is crucial to consider the impli-
cations of o↵shoring before making a decision.

Which choice completes the text with the most logical and precise word or phrase?

A) ethical
B) aesthetic
C) celestial
D) alliterative

84
17. Jet skiing has become a popular water sport, combining the thrill of speed with the plea-
sure of being on the water. Enthusiasts often find themselves drawn to the adrenaline
rush, quickly improving their skills to navigate challenging courses. However, jet skiing
poses potential dangers, including collisions and environmental harm. Therefore, respon-
sible jet skiing requires proper training, adherence to safety guidelines, and respect for
the environment.

Which choice best states the main idea of the text?

A) The text primarily discusses the history and evolution of jet skiing as a water sport.
B) The text emphasizes the importance of responsible jet skiing, considering both safety
and environmental concerns.
C) The text focuses on the various techniques and methods used by professional jet
skiers.
D) The text describes the di↵erent types of jet skis and their specific features for various
water conditions.

18. The Battle of Stalingrad was a crucial turning point in World War II. From July 1942
to February 1943, the Soviet Union and Nazi Germany fought fiercely for control of the
city. The Soviets ultimately emerged victorious, marking the beginning of the end for the
German army. This grueling conflict resulted in tremendous casualties on both sides, but
it also demonstrated the resilience and determination of the Soviet people in the face of
seemingly insurmountable odds.

Which choice best states the main idea of the text?

A) The Battle of Stalingrad was a relatively minor event in the larger context of World
War II.
B) The Battle of Stalingrad was a pivotal moment in World War II, highlighting the
Soviet Union’s tenacity and ultimate triumph.
C) The Battle of Stalingrad primarily showcased the military prowess of the German
army.
D) The Battle of Stalingrad had little long-term impact on the outcome of World War
II.

85
19. 3D bioprinting is an emerging technology that allows the creation of complex biological
structures by depositing bioinks, which are made of living cells, in a layer-by-layer fashion.
This revolutionary technique has the potential to revolutionize medicine and healthcare,
enabling the fabrication of replacement organs, personalized drug testing, and the study
of complex cellular interactions.

Which choice best states the main idea of the text?

A) The text primarily focuses on the development of the bioinks used in 3D bioprinting
technology.
B) The main idea of the text is to introduce 3D bioprinting as a revolutionary technology
with the potential to transform medicine and healthcare.
C) The purpose of the text is to examine the ethical implications of 3D bioprinting in
medicine.
D) This text highlights the challenges and limitations of 3D bioprinting technology in
the medical field.

20. In network engineering, the implementation of the Border Gateway Protocol (BGP) im-
proves routing between autonomous systems (ASes). However, it’s been observed that
some networks experience increased latency when utilizing BGP. Network engineer Alex
Johnson proposes that using BGP in combination with optimized routing algorithms can
counteract latency issues.

Which finding, if true, would most strongly support Alex Johnson’s proposal?

A) Many networks experiencing increased latency have outdated hardware that is not
optimized for BGP implementation.
B) A majority of network engineers agree that BGP is essential for routing between
autonomous systems.
C) Latency issues in networks using BGP are often the result of misconfigurations by
inexperienced administrators.
D) Networks implementing BGP with optimized routing algorithms experience a signif-
icant reduction in latency compared to using BGP alone.

86
21. John Dalton, an English chemist, developed the atomic theory in the early 19th century.
He proposed that elements are composed of identical atoms with unique masses, and that
chemical reactions involve the rearrangement of these atoms.

Which finding, if true, would most directly challenge the validity of Dalton’s atomic
theory?

A) Some chemical reactions can be explained by a di↵erent model, not involving atoms.
B) Dalton’s atomic theory was initially met with skepticism by the scientific community.
C) A newly discovered element consists of atoms with varying masses, contrary to Dal-
ton’s proposition.
D) The concept of atoms predates Dalton, with mentions in ancient Greek texts.

22. In the Superman comics, many fans believed that Superman’s powers were derived solely
from Earth’s yellow sun. However, a recent article suggests that his Kryptonian physiology
may play a more significant role in his superhuman abilities.

Which finding, if true, would best support the article’s argument?

A) The yellow sun’s radiation has di↵erent e↵ects on Earth-born humans compared to
Kryptonians.
B) Superman’s creators originally intended for his powers to come exclusively from
Earth’s yellow sun.
C) Other superheroes who gained their powers from Earth’s yellow sun have abilities
di↵erent from Superman’s.
D) A study reveals that Kryptonians on other planets with yellow suns do not exhibit
the same powers as Superman.

87
23. In computer engineering, the concept of abstraction plays a crucial role in simplifying
complex systems. This process allows engineers to focus on specific components without
being overwhelmed by intricate details. However, excessive reliance on abstraction can
sometimes lead to .

Which choice most logically completes the text?

A) a significant improvement in the overall performance of the system as abstraction


removes unnecessary elements
B) an increase in productivity and efficiency due to the elimination of all complexities
in the system
C) a loss of understanding of the underlying system and potential issues that may arise
from the hidden complexities
D) a shift in focus from the actual engineering tasks to solely concentrating on the
abstract concepts

24. In modern fencing, there are three weapons used: foil, épée, and sabre. Each weapon has
its unique set of rules and target areas. The foil and épée are point-thrusting weapons,
whereas the sabre includes cutting actions. The primary di↵erence between foil and épée
is that in foil, only the torso is a valid target, while in épée, the entire body is a valid
target. This di↵erence has implications for the way fencers approach each weapon, leading
to .

Which choice most logically completes the text?

A) a significant disparity in the popularity of foil and épée among fencing enthusiasts
B) an increased likelihood of injuries occurring in épée fencing compared to foil fencing
C) distinct strategies and tactics employed by fencers when using foil and épée
D) the development of specialized protective gear exclusively designed for épée fencing

88
25. Neil deGrasse Tyson, a prominent astrophysicist and science communicator, is known
for making complex scientific concepts accessible to the public. He has hosted a number
of educational television shows, such as NOVA ScienceNow and Cosmos: A Spacetime
Odyssey. his engaging public lectures and social media presence have also con-
tributed to his widespread popularity.

Which choice completes the text with the most logical transition?

A) In contrast
B) On the other hand
C) Furthermore
D) Instead

26. Hairspray, a popular styling product, has been in use for decades to provide hold and
volume to various hairstyles. In the 1960s, it was particularly popular for creating the
beehive hairstyle, which required a significant amount of hairspray to maintain its shape.
in recent years, hairspray formulas have become more environmentally friendly,
as they no longer contain chlorofluorocarbons (CFCs) that were once known to contribute
to the depletion of the ozone layer.

Which choice completes the text with the most logical transition?

A) However
B) Moreover
C) Alternatively
D) In contrast

89
27. In a market characterized by perfect competition, there is a large number of buyers and
sellers, and none of them can significantly influence the price of the product or service.
These sellers o↵er identical products, which means that consumers have no preference for
one seller over another. in perfect competition, there are no barriers to entry
and exit, which results in a dynamic market where new businesses can enter and existing
businesses can exit with relative ease.

Which choice completes the text with the most logical transition?

A) In contrast
B) However
C) For example
D) Additionally

90
Chapter 15

Module 1 Answers

1. Correct Answer: C
Explanation: C: ’To describe the Congress of Vienna’s objectives and its diplomatic
approach in restoring Europe’s balance of power’ is the correct answer because the text
focuses on the goals of the Congress and its prioritization of negotiation and consensus-
building.

2. Correct Answer: B
Explanation: B: ’To provide a brief overview of the English Civil War, highlighting the
key parties involved and its outcome’ is the correct answer because the passage presents
a concise summary of the English Civil War, touching on the main participants and the
war’s conclusion.

3. Correct Answer: A
Explanation: A: ’By pointing out that the Duke’s portrayal may be more accurate than
Miller suggests, as it aligns with historical accounts of real-life figures’ is the correct
answer because Text 2 discusses Thompson’s belief that Browning’s portrayal could be
accurate due to its alignment with historical accounts.

4. Correct Answer: B
Explanation: B: ’navigating’ is the correct answer because it is the present participle form
of the verb ’navigate,’ which is grammatically correct and fits the context of the passage.

5. Correct Answer: C
Explanation: C: ’has continued’ is the correct answer because it properly expresses the
present perfect tense in the context, indicating that the sport’s popularity has been in-
creasing since the establishment of the ITTF.

91
6. Correct Answer: C
Explanation: C: ’manipulating’ is the correct answer because it provides a present par-
ticiple verb that fits the context and maintains the appropriate tense.

7. Correct Answer: A
Explanation: A: ’found’ is the correct answer because it provides the main clause with
a finite past tense verb, which indicates the action that took place when the manuscript
was finally accepted by a publisher.

8. Correct Answer: D
Explanation: D: ’As an iconic figure and symbol of the Walt Disney Company, Mickey
Mouse has played a significant role in popular culture through various films, television
shows, and comic strips.’ is the correct answer because it e↵ectively explains the signifi-
cance of Mickey Mouse in popular culture by mentioning his role as a symbol of the Walt
Disney Company and his presence in di↵erent forms of media.

9. Correct Answer: D
Explanation: D: ’Prospect Theory, which includes the principle of loss aversion, is a
key concept in the study of behavioral economics.’ is the correct answer because it
e↵ectively highlights the relationship between Prospect Theory and behavioral economics
by mentioning the principle of loss aversion.

10. Correct Answer: D


Explanation: D: ’The Pomodoro Technique divides work into 25-minute intervals with
short breaks, and after four intervals, a longer break is taken.’ is the correct answer
because it e↵ectively uses information from the notes to explain the Pomodoro Technique,
including the work intervals, breaks, and the structure of the technique.

11. Correct Answer: A


Explanation: A: ’a system of thinking’ is the correct answer because it properly completes
the sentence and conforms to Standard English conventions.

12. Correct Answer: C


Explanation: C: ’four hundred’ is the correct answer because it is the appropriate form
for expressing the number in this context.

92
13. Correct Answer: A
Explanation: A: ’performance’ is the correct answer because it correctly uses the singular
noun ’performance’ to indicate the collective behavior of other commodities a↵ecting the
price of gold.

14. Correct Answer: C


Explanation: C: ’challenge’ is the correct answer because it accurately describes the goal
of Mahatma Gandhi’s Satyagraha strategy in the context of the British rule in India.

15. Correct Answer: C


Explanation: A: ’Challenging’ is the correct answer because it accurately describes the
difficulty that might arise when using prefabricated components in complex or highly
specialized structures.

16. Correct Answer: A


Explanation: A: ’ethical’ is the correct answer because it relates to the moral aspects
and potential consequences of o↵shoring, which is an important factor for companies to
consider when making decisions.

17. Correct Answer: B


Explanation: B: ’The text emphasizes the importance of responsible jet skiing, consider-
ing both safety and environmental concerns.’ is the correct answer because the passage
discusses the popularity and excitement of jet skiing while also highlighting the potential
dangers and the need for responsible behavior.

18. Correct Answer: B


Explanation: B: ’The Battle of Stalingrad was a pivotal moment in World War II, high-
lighting the Soviet Union’s tenacity and ultimate triumph.’ is the correct answer because
the passage emphasizes the significance of the battle as a turning point, the resilience of
the Soviet people, and their victory over the German army.

19. Correct Answer: B


Explanation: B: ’The main idea of the text is to introduce 3D bioprinting as a revolution-
ary technology with the potential to transform medicine and healthcare.’ is the correct
answer because it encapsulates the main idea of the passage, which is about the potential
impact of 3D bioprinting in the medical field.

93
20. Correct Answer: D
Explanation: D: ’Networks implementing BGP with optimized routing algorithms expe-
rience a significant reduction in latency compared to using BGP alone.’ is the correct
answer because it directly supports Alex Johnson’s proposal that the combination of BGP
and optimized routing algorithms can counteract latency issues.

21. Correct Answer: C


Explanation: C: ’A newly discovered element consists of atoms with varying masses,
contrary to Dalton’s proposition.’ is the correct answer because it directly contradicts
one of the key ideas of Dalton’s atomic theory, which claims that elements are composed
of identical atoms with unique masses.

22. Correct Answer: D


Explanation: D: ’A study reveals that Kryptonians on other planets with yellow suns
do not exhibit the same powers as Superman.’ is the correct answer because it provides
evidence that supports the article’s argument that Superman’s Kryptonian physiology
plays a more significant role in his powers than just the presence of a yellow sun.

23. Correct Answer: C


Explanation: C: ’a loss of understanding of the underlying system and potential issues that
may arise from the hidden complexities’ is the correct answer because the passage discusses
the risk of excessive reliance on abstraction, which could lead to a lack of understanding
of the system’s complexities.

24. Correct Answer: C


Explanation: C: ’distinct strategies and tactics employed by fencers when using foil and
épée’ is the correct answer because the passage states that the di↵erence in valid target
areas between foil and épée has implications for how fencers approach each weapon.

25. Correct Answer: C


Explanation: C: ’Furthermore’ is the correct answer because it adds to the information
about Neil deGrasse Tyson’s e↵orts in making complex scientific concepts accessible to
the public.

26. Correct Answer: B


Explanation: B: ’Moreover’ is the correct answer because it shows an additional point
related to the topic of hairspray, indicating that not only has hairspray been popular

94
for styling purposes, but it has also evolved over time to become more environmentally
friendly.

27. Correct Answer: D


Explanation: D: ’Additionally’ is the correct answer because it logically connects the
sentence about no barriers to entry and exit with the previous information about the
characteristics of a perfect competition.

95
Chapter 16

Module 2

Reading and Writing - 27 Questions

DIRECTIONS: The questions in this section are designed to test your reading and writing
abilities. Each question will provide you with one or more passages. It is important to
thoroughly read each passage and question before selecting the best answer based on the
information provided. All of the questions in this section are multiple-choice and will have
four options to choose from. There is only one correct answer for each question.

1. In George Orwell’s novel 1984, he explores the oppressive tactics employed by totalitarian
governments. Orwell’s vision of a dystopian society, controlled by an omnipresent gov-
ernment entity called ’Big Brother,’ highlights the consequences of relinquishing personal
freedom and autonomy. The novel emphasizes the importance of language, thought, and
individualism as powerful tools to resist totalitarianism.

Which choice best states the main purpose of the text?

A) To illustrate the evolution of George Orwell’s writing style throughout his career
B) To provide a detailed analysis of the novel’s linguistic and stylistic elements
C) To argue that Orwell’s novel was a direct critique of contemporary political systems
D) To highlight the oppressive nature of totalitarian governments and the importance
of personal freedom

96
2. Market failure occurs when the allocation of goods and services by a free market is in-
efficient. It is a situation in which the market does not operate as intended and leads
to undesirable outcomes, such as the overproduction of negative externalities or the un-
derproduction of positive externalities. Market failure can arise from various sources,
including externalities, public goods, and imperfect information. To address market fail-
ure, governments may implement policies and regulations, such as taxes and subsidies,
to correct the market’s inefficiencies and promote a more socially optimal allocation of
resources.

Which choice best states the main purpose of the text?

A) To explain the concept of market failure and its causes, and to briefly discuss potential
government interventions
B) To argue that market failure is solely the result of government intervention in the
economy
C) To provide a comprehensive analysis of the di↵erent types of market failure and their
implications for society
D) To describe various economic theories that have been proposed to address and prevent
market failure

3. Text 1: The Organization of American States (OAS) is an international organization


comprising 35 countries from the Americas. Its primary mission is to promote peace,
security, and cooperation among its member states. The OAS has been successful in
resolving conflicts and promoting democracy throughout the region.

Text 2: Critics of the OAS argue that its influence has diminished over the years due to
internal divisions and external interference. They claim that the organization is no longer
as e↵ective in addressing regional issues and that its focus on democracy promotion has
led to interventionist policies, which undermine the sovereignty of its member states.

Based on the texts, how would critics (Text 2) most likely respond to the “primary
mission” of OAS discussed in Text 1?

A) By asserting that the OAS has consistently fulfilled its mission of promoting peace,
security, and cooperation in the Americas
B) By arguing that the OAS’s mission has been compromised due to internal divisions
and interventionist policies, limiting its e↵ectiveness
C) By claiming that the OAS’s primary mission should be focused on economic devel-
opment and integration rather than political issues

97
D) By suggesting that the OAS should expand its membership to include countries from
outside the Americas to strengthen its influence

4. The Statue of Liberty, a gift from the people of France to the United States, has stood
in New York Harbor since 1886. It symbolizes freedom and democracy, and visitors can
access the statue’s crown by climbing a challenging, 377-step spiral staircase. This iconic
monument around 3.2 million visitors annually.

Which choice completes the text so that it conforms to the conventions of Standard
English?

A) to attract
B) attracts
C) attracting
D) will attract

5. Hydrogen fuel cells have the potential to revolutionize the automotive industry. By com-
bining hydrogen and oxygen, these cells generate electricity and emit only water vapor
as a byproduct. However, challenges remain, including the need for a more extensive
refueling infrastructure and concerns about the of hydrogen production.

Which choice completes the text so that it conforms to the conventions of Standard
English?

A) sustainable
B) sustain
C) sustainability
D) sustaining

98
6. Back to the Future, a popular science fiction film series, features a time-traveling DeLorean
car. In the first movie, Marty McFly, the protagonist, accidentally travels back to 1955
and must ensure his parents to secure his own existence.

Which choice completes the text so that it conforms to the conventions of Standard
English?

A) to meet
B) meet
C) meeting
D) met

7. In order to maximize profits, companies often invest in research and development. This
strategy can lead to the creation of new products or services, which in turn ad-
ditional revenue streams for the business.

Which choice completes the text so that it conforms to the conventions of Standard
English?

A) generated
B) generate
C) will be generating
D) to generate

99
8. While researching a topic, a student has taken the following notes:
• Jane Austen’s novel Pride and Prejudice was published in 1813.
• The novel explores themes of class, marriage, and societal expectations.
• Austen used irony and wit to critique the social norms of her time.
• The protagonist, Elizabeth Bennet, defies traditional gender roles through her intel-
ligence and independence.

Which choice most e↵ectively uses relevant information from the notes to explain the
significance of Jane Austen’s Pride and Prejudice?

A) Pride and Prejudice, published in 1813, is a novel by Jane Austen.


B) In Pride and Prejudice, Jane Austen critiques societal norms and gender roles through
the use of irony, wit, and the independent character of Elizabeth Bennet.
C) Elizabeth Bennet is the protagonist of Jane Austen’s Pride and Prejudice.
D) Jane Austen’s Pride and Prejudice explores themes of class and marriage in the early
19th century.

9. While researching a topic, a student has taken the following notes:


• Artificial organs can be used to replace failing human organs.
• 3D printing technology has advanced the field of artificial organ production.
• Artificial hearts and kidneys have been successfully implanted in patients.
• Researchers are working on creating artificial lungs using a combination of synthetic
materials and living cells.

Which choice most e↵ectively uses relevant information from the notes to emphasize the
potential impact of artificial organs on medical treatments?

A) Artificial organs can be used to replace human organs, and researchers are currently
working on developing artificial lungs.
B) 3D printing technology has enabled the creation of artificial organs such as hearts
and kidneys.
C) With the advancements in 3D printing technology, artificial organs like hearts and kid-
neys have been successfully implanted in patients, potentially revolutionizing medical
treatments.
D) The field of artificial organ production has seen advancements, particularly in the
development of artificial hearts and kidneys.

100
10. While researching a topic, a student has taken the following notes:

• Control engineering is a branch of engineering that deals with the design of systems
to maintain desired behaviors.
• Feedback control systems are widely used in control engineering.
• PID (Proportional-Integral-Derivative) controllers are a popular type of feedback
control system.
• Control engineering has applications in various industries, such as manufacturing,
aerospace, and robotics.

Which choice most e↵ectively uses relevant information from the notes to emphasize the
significance of control engineering?

A) Control engineering involves the design of systems and has applications in manufac-
turing.
B) A popular type of feedback control system used in control engineering is the PID
controller.
C) Control engineering plays a crucial role in diverse industries like manufacturing,
aerospace, and robotics by designing systems to maintain desired behaviors.
D) Control engineering focuses on maintaining desired behaviors in systems, such as
feedback control systems.

11. Archimedes of Syracuse, a prominent ancient Greek mathematician, physicist, and engi-
neer, made significant contributions to various fields of science. One notable discovery
was his principle of buoyancy, which states that the upward buoyant force exerted on a
body immersed in a fluid is equal to the weight of the fluid that the body

Which choice completes the text so that it conforms to the conventions of Standard
English?

A) displaces
B) body displaces
C) the body’s displacement
D) body’s displaces

101
12. Ernest Hemingway, a renowned American author, had a unique writing style often char-
acterized by simplicity and minimalism. He referred to this technique as the ’iceberg
theory,’ which focused on revealing only the surface-level information in his narratives,
while the deeper meaning remained to the reader.

Which choice completes the text so that it conforms to the conventions of Standard
English?

A) obscure
B) obscurly
C) hiddenly
D) hiding

13. Reaganomics, a set of economic policies promoted by President Ronald Reagan, focused on
reducing government spending, taxes, and regulation. The theory behind Reaganomics,
supply-side economics, argued that by concentrating on the supply of goods, it would
stimulate economic growth and benefit the entire population, including the

Which choice completes the text so that it conforms to the conventions of Standard
English?

A) poor’s
B) poors
C) poor;
D) poor

102
14. Fixed costs are expenses that remain constant, regardless of the level of goods or services
produced. Examples of fixed costs include rent, insurance, and salaries. These costs
are to the volume of production, and businesses must account for them when
assessing their overall financial health.

Which choice completes the text with the most logical and precise word or phrase?

A) independent
B) proportional
C) attractive
D) imperative

15. The Black Death, which ravaged Europe during the 14th century, was a pandemic caused
by the bacterium Yersinia pestis. This deadly disease was transmitted by fleas that
infested rats, leading to a rapid and decline in population across the continent.

Which choice completes the text with the most logical and precise word or phrase?

A) gradual
B) catastrophic
C) unnoticeable
D) trivial

103
16. Windsurfing, a surface water sport, combines elements of both sailing and surfing. The
windsurfer stands on a board and holds on to a sail attached to a mast, which is to
the board. The sail catches the wind, propelling the board and rider across the water.

Which choice completes the text with the most logical and precise word or phrase?

A) indi↵erent
B) dissimilar
C) connected
D) opposed

17. Natural language processing (NLP) is a subfield of artificial intelligence focused on en-
abling computers to interpret and generate human language. NLP has gained significant
attention due to its potential applications in various domains, including virtual assis-
tants, sentiment analysis, and machine translation. However, the complexity of human
language presents numerous challenges for NLP algorithms, requiring ongoing research
and development to improve their accuracy and efficiency.

Which choice best states the main idea of the text?

A) The text focuses on the challenges faced by NLP algorithms in interpreting human
language.
B) The text provides an overview of the NLP subfield and its potential applications and
challenges.
C) The text highlights the role of virtual assistants in the development of NLP technol-
ogy.
D) The text discusses the importance of machine translation in the advancement of NLP
research.

104
18. In the field of biological engineering, scientists work to develop innovative solutions for a
variety of problems. Recently, a research team successfully designed bacteria capable of
breaking down plastic waste. This breakthrough has the potential to revolutionize waste
management systems around the world, greatly reducing the environmental impact of
plastic pollution.

Which choice best states the main idea of the text?

A) The text focuses on the history of biological engineering and its various applications.
B) The text solely discusses the negative e↵ects of plastic waste on the environment.
C) The text argues that plastic pollution is the most pressing issue facing biological
engineers today.
D) The text highlights a breakthrough in biological engineering that addresses plastic
waste.

19. Henry V was a skilled military leader who led England to several significant victories
during his reign. However, he was also a wise and diplomatic ruler who pursued peaceful
relations when possible. Though known for his triumph at the Battle of Agincourt, Henry
V sought to unify England and France through diplomacy, eventually marrying the French
princess Catherine of Valois.

Which choice best states the main idea of the text?

A) Henry V’s primary goal was to conquer France and expand England’s territories.
B) Henry V was a successful military leader who also valued diplomacy and pursued
peaceful relations.
C) The Battle of Agincourt was the defining moment of Henry V’s reign and overshad-
owed his diplomatic e↵orts.
D) Henry V married Catherine of Valois solely for political gain and did not truly value
diplomacy.

105
20. The Peruvian marinera, a traditional coastal dance, has evolved into various regional
styles. In Northern Peru, the marinera norteña is characterized by its elegance and
intricate footwork, while the marinera Limeña from Lima has a more subdued tempo and
focuses on graceful upper body movements.

Which finding, if true, would best support the claim that the marinera norteña and
marinera Limeña are distinct regional styles of the Peruvian marinera?

A) The marinera norteña has roots in Spanish, African, and indigenous Peruvian dance
traditions, while the marinera Limeña’s origins are unclear.
B) Some Peruvian dancers perform the marinera norteña exclusively, while others spe-
cialize in the marinera Limeña.
C) Both the marinera norteña and marinera Limeña are popular among Peruvian dancers
and audiences, with numerous regional competitions held annually.
D) A comprehensive analysis of the dance techniques reveals significant di↵erences in
footwork and tempo between the two styles.

21. The Falklands War, a 1982 conflict between Argentina and the UK, resulted in a British
victory. It has been suggested that advanced British technology played a pivotal role in
the outcome, while others argue that strategic planning was the key factor.

Which finding, if true, would most directly support the argument that advanced British
technology was crucial to their victory in the Falklands War?

A) British soldiers received advanced training in tactical warfare compared to Argentine


troops.
B) The British government invested more in military research and development than
Argentina.
C) The Royal Navy utilized cutting-edge missile systems that significantly outperformed
Argentine weaponry.
D) The UK’s strategic planning was more e↵ective in anticipating and countering Ar-
gentina’s military moves.

106
22. A recent study found that the installation of o↵shore wind farms positively impacts ma-
rine biodiversity. Researchers observed an increase in fish populations around the wind
turbines, which provided new habitats and attracted more prey species.

Which finding, if true, would most directly support the claim that o↵shore wind farms
positively impact marine biodiversity?

A) O↵shore wind turbines act as artificial reefs, promoting the growth of marine life
around them.
B) The construction of o↵shore wind farms generates noise pollution, which can a↵ect
marine animals.
C) The maintenance of o↵shore wind farms requires frequent human intervention, po-
tentially disturbing marine habitats.
D) O↵shore wind farms can cause changes in ocean currents, a↵ecting the distribution
of marine species.

23. King Henry VIII is famous for his six marriages and his role in the English Reformation.
Though he is often remembered for his turbulent marriages, his reign had a lasting impact
on England’s religious and political landscape. As a result, historians argue that .

Which choice most logically completes the text?

A) Henry VIII’s marriages were the main driving force behind the English Reformation,
with his personal life directly shaping the religious and political changes.
B) Henry VIII’s reign was unremarkable except for his marital life, which overshadows
his other contributions to England’s history.
C) Henry VIII’s reign had no significant impact on England’s religious and political
landscape, as the country reverted to its original state after his death.
D) Henry VIII’s influence on England goes beyond his personal life and should be assessed
in terms of the wider changes he brought about.

107
24. Linus Pauling, a renowned scientist, contributed significantly to the fields of chemistry
and molecular biology. His research on the nature of chemical bonds earned him a Nobel
Prize in Chemistry. Later in his career, Pauling became an advocate for the medical use
of vitamin C, despite facing criticism from the scientific community. This led to

Which choice most logically completes the text?

A) the establishment of a new branch of chemistry focused on vitamins.


B) a decrease in his credibility among his scientific peers.
C) an increase in the number of Nobel Prizes awarded in Chemistry.
D) a shift in public perception regarding the potential benefits of vitamin C.

25. Margaret Thatcher, the first female Prime Minister of the United Kingdom, served from
1979 to 1990. She was known for her strong and uncompromising leadership style, which
earned her the nickname ’The Iron Lady.’ Thatcher implemented a series of economic
policies to combat high inflation and stimulate economic growth. her foreign
policy focused on maintaining Britain’s influence on the world stage and strengthening
relationships with the United States.

Which choice completes the text with the most logical transition?

A) Similarly,
B) In contrast,
C) However,
D) Moreover,

108
26. The English Civil War, which took place from 1642 to 1651, involved a series of armed
conflicts between the Parliamentarians and the Royalists. The Parliamentarians, led by
Oliver Cromwell, sought to limit the power of the monarchy, while the Royalists supported
King Charles I. the conflict came to an end with the execution of Charles I in
1649, and the establishment of the Commonwealth of England, which lasted until 1660.

Which choice completes the text with the most logical transition?

A) In contrast
B) Similarly
C) Additionally
D) Ultimately

27. Market failure is a situation in which the allocation of goods and services by a free market
is not efficient. This often occurs because of the presence of externalities, or costs and
benefits that are not accounted for by market transactions. One common example of
market failure is pollution, as polluting industries often do not bear the full cost of the
damage they cause to the environment. government intervention, such as the
implementation of environmental regulations or a carbon tax, can help to correct these
inefficiencies and improve overall welfare.

Which choice completes the text with the most logical transition?

A) Therefore
B) Alternatively
C) Nonetheless
D) In contrast

109
Chapter 17

Module 2 Answers

1. Correct Answer: D
Explanation: D: ’To highlight the oppressive nature of totalitarian governments and the
importance of personal freedom’ is the correct answer because the passage focuses on the
themes of oppression, personal freedom, and the power of individualism in the novel 1984.

2. Correct Answer: A
Explanation: A: ’To explain the concept of market failure and its causes, and to briefly
discuss potential government interventions’ is the correct answer because the passage
defines market failure, mentions its causes, and briefly discusses possible government
solutions.

3. Correct Answer: B
Explanation: B: ’By arguing that the OAS’s mission has been compromised due to inter-
nal divisions and interventionist policies, limiting its e↵ectiveness’ is the correct answer
because Text 2 states that critics claim the organization’s influence has diminished and
its focus on democracy promotion has led to interventionist policies.

4. Correct Answer: B
Explanation: B: ’attracts’ is the correct answer because it provides a finite present tense
verb that agrees with the subject ’monument’ and conforms to the conventions of Standard
English.

5. Correct Answer: A
Explanation: A: ’sustainable’ is the correct answer because it is an adjective that properly
describes the concerns related to hydrogen production and fits the sentence structure.

110
6. Correct Answer: B
Explanation: B: ’meet’ is the correct answer because it supplies the clause with the finite
present tense verb ’meet’ to indicate the action Marty needs to ensure.

7. Correct Answer: B
Explanation: B: ’generate’ is the correct answer because it is consistent with the present
tense verbs used in the passage and it shows the e↵ect of the strategy on the business.

8. Correct Answer: B
Explanation: B: ’In Pride and Prejudice, Jane Austen critiques societal norms and gender
roles through the use of irony, wit, and the independent character of Elizabeth Bennet.’
is the correct answer because it e↵ectively uses relevant information from the notes to
explain the significance of the novel in terms of its themes, literary techniques, and the
protagonist’s role in challenging societal expectations.

9. Correct Answer: C
Explanation: C: ’With the advancements in 3D printing technology, artificial organs like
hearts and kidneys have been successfully implanted in patients, potentially revolutioniz-
ing medical treatments.’ is the correct answer because it emphasizes the potential impact
of artificial organs on medical treatments by mentioning successful implantations and the
use of advanced technology.

10. Correct Answer: C


Explanation: C: ’Control engineering plays a crucial role in diverse industries like manu-
facturing, aerospace, and robotics by designing systems to maintain desired behaviors.’ is
the correct answer because it e↵ectively emphasizes the importance of control engineering
and its applications in various industries.

11. Correct Answer: A


Explanation: A: ’displaces’ is the correct answer because the subject ’body’ is already
mentioned earlier and the verb ’displaces’ agrees with the singular subject.

12. Correct Answer: A


Explanation: A: ’obscure’ is the correct answer because it appropriately describes the
deeper meaning in Hemingway’s narratives, which was not directly revealed to the readers.

111
13. Correct Answer: D
Explanation: D: ’poor’ is the correct answer because it correctly refers to the entire
population, including the economically disadvantaged people.

14. Correct Answer: A


Explanation: A: ’independent’ is the correct answer because fixed costs do not change
based on the volume of production, making them independent of production levels.

15. Correct Answer: B


Explanation: B: ’catastrophic’ is the correct answer because it accurately describes the
significant and disastrous impact of the Black Death on Europe’s population.

16. Correct Answer: C


Explanation: C: ’connected’ is the correct answer because it accurately describes the
relationship between the mast and the board in windsurfing.

17. Correct Answer: B


Explanation: B: ’The text provides an overview of the NLP subfield and its potential
applications and challenges.’ is the correct answer because it accurately captures the
essence of the passage. The passage introduces NLP as a subfield of artificial intelligence,
mentions its potential applications, and acknowledges the challenges it faces due to the
complexity of human language.

18. Correct Answer: D


Explanation: D: ’The text highlights a breakthrough in biological engineering that ad-
dresses plastic waste.’ is the correct answer because the passage focuses on the devel-
opment of bacteria capable of breaking down plastic waste and its potential impact on
waste management systems.

19. Correct Answer: B


Explanation: B: ’Henry V was a successful military leader who also valued diplomacy
and pursued peaceful relations.’ is the correct answer because it combines the aspects of
Henry V’s military leadership and diplomatic e↵orts mentioned in the passage.

112
20. Correct Answer: D
Explanation: D: ’A comprehensive analysis of the dance techniques reveals significant
di↵erences in footwork and tempo between the two styles.’ is the correct answer because
it directly supports the claim that the marinera norteña and marinera Limeña are distinct
regional styles of the Peruvian marinera, by highlighting the di↵erences in their techniques.

21. Correct Answer: C


Explanation: C: ’The Royal Navy utilized cutting-edge missile systems that significantly
outperformed Argentine weaponry.’ is the correct answer because it directly shows that
advanced British technology played an important role in the Falklands War outcome.

22. Correct Answer: A


Explanation: A: ’O↵shore wind turbines act as artificial reefs, promoting the growth
of marine life around them.’ is the correct answer because it provides direct evidence
supporting the claim that o↵shore wind farms positively impact marine biodiversity by
creating new habitats for marine life.

23. Correct Answer: D


Explanation: D: ’Henry VIII’s influence on England goes beyond his personal life and
should be assessed in terms of the wider changes he brought about.’ is the correct answer
because it acknowledges the lasting impact of his reign on England’s religious and political
landscape.

24. Correct Answer: D


Explanation: D: ’a shift in public perception regarding the potential benefits of vitamin C’
is the correct answer because Pauling’s advocacy likely influenced the way people thought
about the uses of vitamin C in medicine.

25. Correct Answer: A


Explanation: A: ’Similarly’ is the correct answer because it indicates that both her eco-
nomic policies and her foreign policy were focused on achieving specific goals.

26. Correct Answer: D


Explanation: D: ’Ultimately’ is the correct answer because it provides a logical transition
to the final outcome of the English Civil War.

113
27. Correct Answer: A
Explanation: A: ’Therefore’ is the correct answer because it logically connects the cause
of market failure (externalities) to the possible solution (government intervention).

114
Chapter 18

Module 3

Reading and Writing - 27 Questions

DIRECTIONS: The questions in this section are designed to test your reading and writing
abilities. Each question will provide you with one or more passages. It is important to
thoroughly read each passage and question before selecting the best answer based on the
information provided. All of the questions in this section are multiple-choice and will have
four options to choose from. There is only one correct answer for each question.

1. Frida Kahlo, a renowned Mexican painter, was known for her self-portraits and her ability
to communicate her pain and emotions through her artwork. Her paintings often included
vivid colors and a mix of symbolism and surrealism. Kahlo’s work was deeply influenced
by her experiences, which included a near-fatal accident, numerous surgeries, and a tu-
multuous marriage with fellow artist Diego Rivera. Despite her physical and emotional
struggles, Kahlo continued to create art that resonated with people across generations
and cultures.

Which choice best states the main purpose of the text?

A) To analyze the influence of Frida Kahlo’s work on the broader art world and its
reception
B) To discuss the technical aspects of Frida Kahlo’s painting style and use of color
C) To provide a biography of Frida Kahlo, focusing on her marriage to Diego Rivera
D) To emphasize Frida Kahlo’s resilience and the personal nature of her artwork

115
2. Queen Isabella I, who ruled Castile from 1474 to 1504, enacted a series of reforms dur-
ing her reign. Together with her husband, King Ferdinand II of Aragon, she worked to
strengthen the monarchy, centralize power, and consolidate the Catholic Church’s influ-
ence. Among her notable achievements was the sponsorship of Christopher Columbus’s
first voyage in 1492, which led to the discovery of the Americas. Queen Isabella I’s reign
had a significant impact on the world, as her decisions shaped the future of Spain and
the Americas.

Which choice best states the main purpose of the text?

A) To provide an in-depth analysis of Queen Isabella I’s personality traits and how they
contributed to her success as a ruler
B) To emphasize the historical importance of Queen Isabella I’s reign by highlighting
her achievements and their consequences
C) To argue that Queen Isabella I’s reign was marked by a series of controversial decisions
that led to negative outcomes
D) To compare and contrast the leadership styles of Queen Isabella I and King Ferdinand
II during their respective reigns

3. Text 1: Neil Armstrong, the first human to set foot on the moon, went on to become a
professor at the University of Cincinnati. He taught aerospace engineering and was known
for his hands-on teaching approach, often taking students to the field to gain practical
experience.

Text 2: Although Neil Armstrong is celebrated for his role as an astronaut, his contri-
butions to the field of aerospace engineering are also significant. Armstrong’s work as a
professor at the University of Cincinnati, where he developed and implemented innovative
teaching methods, helped shape generations of aerospace engineers.

Based on the texts, how would the author of Text 2 most likely respond to the teaching
approach discussed in Text 1?

A) By asserting that Neil Armstrong’s teaching approach was not as e↵ective as more
traditional methods in the field of aerospace engineering.
B) By suggesting that Neil Armstrong’s hands-on teaching approach was a result of his
experiences as an astronaut and not as relevant to aerospace engineering.
C) By questioning whether Neil Armstrong’s hands-on teaching approach was solely
responsible for the success of his students in the field of aerospace engineering.
D) By agreeing that Neil Armstrong’s hands-on teaching approach was a valuable con-
tribution to the field of aerospace engineering and the education of students.

116
4. Ada Lovelace, often regarded as the world’s first computer programmer, worked closely
with Charles Babbage on the Analytical Engine. Her extensive notes and understanding
of the machine’s potential uses her the title of ’prophet of the computer age’.

Which choice completes the text so that it conforms to the conventions of Standard
English?

A) earning
B) earned
C) to earn
D) will be earning

5. The Archimedes’ screw, a device invented in ancient Greece, was initially used to transfer
water from lower to higher levels. It consists of a helical surface surrounding a cylindrical
shaft, which, when , causes water to move along the helix.

Which choice completes the text so that it conforms to the conventions of Standard
English?

A) rotate
B) rotated
C) rotating
D) to rotate

117
6. The concept of a living wage is gaining traction in many countries, as it seeks to ensure
workers earn enough to maintain a decent quality of life. However, critics argue that the
implementation of a living wage businesses with increased labor costs, potentially
leading to job losses.

Which choice completes the text so that it conforms to the conventions of Standard
English?

A) burdens
B) burdening
C) burden
D) to burden

7. The Iron Curtain, a term coined by Winston Churchill, was a metaphorical barrier divid-
ing Europe into two parts: the democratic Western Europe and the communist Eastern
Europe. The formation of the Iron Curtain with the end of World War II and
the beginning of the Cold War.

Which choice completes the text so that it conforms to the conventions of Standard
English?

A) coincided
B) have coincided
C) coinciding
D) to coincide

118
8. While researching a topic, a student has taken the following notes:

• The first Thanksgiving was celebrated in 1621 by the Pilgrims and the Wampanoag
tribe.
• The feast lasted for three days and included various types of food, such as venison,
fowl, and corn.
• The event was not originally called ’Thanksgiving’ but was a harvest celebration.
• The modern-day Thanksgiving holiday was established by President Abraham Lin-
coln in 1863.

Which choice most e↵ectively uses relevant information from the notes to emphasize the
historical context of the first Thanksgiving celebration?

A) The first Thanksgiving was a three-day feast with a variety of foods, such as venison
and corn.
B) Thanksgiving, a holiday first celebrated by the Pilgrims and the Wampanoag tribe,
now takes place every year in the United States.
C) The first Thanksgiving in 1621 was a harvest celebration between Pilgrims and the
Wampanoag tribe, predating its official establishment as a holiday in 1863.
D) The modern Thanksgiving holiday, established in 1863, was inspired by a feast be-
tween Pilgrims and the Wampanoag tribe.

9. While researching a topic, a student has taken the following notes:

• The Venezuelan refugee crisis has led to millions of people fleeing the country.
• Neighboring countries like Colombia, Peru, and Brazil have taken in the majority of
Venezuelan refugees.
• The crisis has placed significant strain on resources and infrastructure in host coun-
tries.
• International aid agencies are providing assistance to help alleviate the burden on
host countries.

Which choice most e↵ectively uses relevant information from the notes to explain the
impact of the Venezuelan refugee crisis on neighboring countries?

A) The Venezuelan refugee crisis is a pressing issue that a↵ects millions of people.
B) Neighboring countries, such as Colombia, Peru, and Brazil, are experiencing strain
on resources and infrastructure due to the influx of Venezuelan refugees.

119
C) International aid agencies are actively involved in providing assistance to Venezuelan
refugees.
D) The Venezuelan refugee crisis has led to a widespread humanitarian e↵ort to support
displaced individuals.

10. While researching a topic, a student has taken the following notes:

• Elon Musk is the CEO of Tesla, Inc., an electric vehicle company.


• Musk is also the founder of SpaceX, a private space exploration company.
• Tesla’s electric vehicles have contributed to the reduction of greenhouse gas emis-
sions.
• SpaceX has plans to colonize Mars in the future.

Which choice most e↵ectively uses relevant information from the notes to emphasize Elon
Musk’s role in both sustainable transportation and space exploration?

A) Elon Musk is the CEO of Tesla, which produces electric vehicles.


B) Elon Musk, as the CEO of Tesla and founder of SpaceX, plays a crucial role in
advancing sustainable transportation and space exploration.
C) SpaceX is planning to colonize Mars in the future.
D) Elon Musk has contributed to the reduction of greenhouse gas emissions through
Tesla.

120
11. On December 16, 1773, a group of American colonists disguised as Mohawk Indians
boarded three British ships in Boston Harbor. In protest of the British Parliament’s Tea
Act, they proceeded to dump 342 chests of tea into the harbor, an event later known as
the Boston Tea Party. This act of defiance significantly fueled the colonists’ desire for
and ultimately led to the American Revolution.

Which choice completes the text so that it conforms to the conventions of Standard
English?

A) independence’s
B) independences
C) independence
D) independence’

12. Fire protection engineers consider both active and passive fire protection strategies when
designing buildings. Active fire protection systems, such as sprinklers and smoke detec-
tors, respond to the presence of fire or smoke. On the other hand, passive fire protection
methods, like fire-resistant building materials, are meant to the spread of fire.

Which choice completes the text so that it conforms to the conventions of Standard
English?

A) prohibit
B) prohibit’s
C) prohibited
D) prohibits

121
13. Charles Darwin spent years on the HMS Beagle, traveling to various locations, including
the Galápagos Islands. It was during this journey that his observations of the islands’
unique wildlife, such as the Galápagos tortoise and the finches, contributed to his under-
standing of natural selection and the development of his theory of

Which choice completes the text so that it conforms to the conventions of Standard
English?

A) evolution.
B) evolution and;
C) evolution, and
D) evolution; and

14. Leon Trotsky, a key figure in the Russian Revolution, developed a political theory called
that extended Marxist principles. This theory argued for the necessity of con-
tinuous revolution to create a global communist society and urged workers around the
world to support it.

Which choice completes the text with the most logical and precise word or phrase?

A) permanent revolution
B) endless rebellion
C) continuous conflict
D) infinite struggle

122
15. Motocross, a form of o↵-road motorcycle racing, has gained popularity around the world.
The sport is both physically demanding and technically challenging, as riders must navi-
gate various terrains while maintaining their balance and speed. The nature of
the tracks makes motocross an exciting spectator sport.

Which choice completes the text with the most logical and precise word or phrase?

A) gentle
B) diverse
C) ordinary
D) static

16. Ralph Ellison’s ’Invisible Man’ is considered a seminal work in American literature. The
novel explores the theme of invisibility, which is not only about being physically unseen
but also about being in a society that refuses to acknowledge one’s existence.

Which choice completes the text with the most logical and precise word or phrase?

A) socially ignored
B) magnified
C) omnipresent
D) silently observed

123
17. In the Stranger Things series, the town of Hawkins, Indiana is plagued by supernatural
occurrences. The main characters, a group of children and teenagers, face the dangers of
the Upside Down, a parallel world filled with monsters. Their experiences are intertwined
with the lives of the adults in the town, who work to uncover the mysteries and protect
their loved ones from the unknown threats.

Which choice best states the main idea of the text?

A) The text focuses on the supernatural creatures in the Upside Down.


B) The text emphasizes the importance of friendship among the children.
C) The text highlights the role of the adult characters in the series.
D) The text introduces the challenges faced by the characters in Stranger Things.

18. In this passage, Thomas Edison’s invention of the phonograph is discussed. Although
Edison was already well-known for his numerous contributions to technology, the phono-
graph garnered even more attention from the public. It was a groundbreaking invention
in the field of audio recording and playback, allowing users to both record and listen to
sound on the same device. Edison’s invention revolutionized the way people experienced
music, speeches, and other audio content.

Which choice best states the main idea of the text?

A) Thomas Edison’s inventions were not well-known before the phonograph.


B) The phonograph was an insignificant invention in Edison’s portfolio.
C) Thomas Edison’s invention of the phonograph revolutionized the field of audio record-
ing and playback.
D) Edison’s contributions to technology were limited to the field of audio recording and
playback.

124
19. The text discusses Werner Heisenberg’s uncertainty principle, a quantum mechanics the-
ory that states it is impossible to simultaneously measure both the position and velocity
of a particle with absolute precision. This idea revolutionized physics and challenged
previous notions of certainty in scientific observations.

Which choice best states the main idea of the text?

A) The text describes a biography of Werner Heisenberg and his contributions to science.
B) The text focuses on the immediate impact of Heisenberg’s uncertainty principle on
the scientific community.
C) The text introduces Werner Heisenberg’s uncertainty principle and its significance in
quantum mechanics.
D) The text provides a detailed explanation of the mathematical aspects of Heisenberg’s
uncertainty principle.

20. The Mercosur, a regional trade bloc in South America, aims to promote economic coop-
eration among its members. However, critics argue that the bloc has not achieved its full
potential due to internal disputes and protectionist policies.

Which finding, if true, would most strongly support the critics’ argument?

A) The Mercosur has adopted new policies to streamline the trade process among its
members.
B) Countries outside the Mercosur have significantly increased their trade with the bloc.
C) One of the Mercosur members has experienced rapid economic growth in recent years.
D) The overall trade among Mercosur members has stagnated since the bloc’s inception.

125
21. Caving enthusiasts often rely on natural light sources to navigate, but sometimes use
artificial lighting in deep cave systems. A recent study examines the impact of artificial
light on cave-dwelling organisms, exploring potential negative e↵ects on their behavior
and ecology.

Which finding, if true, would most significantly undermine the claim that artificial light
negatively a↵ects cave-dwelling organisms?

A) Cave-dwelling organisms in areas with artificial light exhibit increased reproduction


rates and stronger resilience.
B) Cave systems with artificial light installed are more frequently visited by caving
enthusiasts.
C) Some cave-dwelling organisms have developed bioluminescence as an adaptation to
their dark environment.
D) Cave-dwelling organisms in artificially lit areas show a slight decrease in activity
during daytime hours.

22. C.S. Lewis, a British writer and scholar, is known for his work in Christian apologetics.
However, his academic focus was on medieval and Renaissance literature. He served
as a professor at both Oxford and Cambridge Universities, where he made significant
contributions to the study of these literary periods.

Which finding, if true, would most strongly support the claim that C.S. Lewis had a
profound impact on the study of medieval and Renaissance literature?

A) C.S. Lewis’s close friendship with J.R.R. Tolkien influenced the creation of the Chron-
icles of Narnia.
B) Many of Lewis’s scholarly works on medieval and Renaissance literature became
foundational texts in these fields of study.
C) C.S. Lewis’s conversion to Christianity played a significant role in shaping his apolo-
getic works.
D) Literary critics and scholars continue to debate the religious themes present in the
Chronicles of Narnia.

126
23. Christopher Columbus is often credited with discovering America, despite evidence of
Viking settlements predating his arrival. Some historians argue that Columbus’s actions
led to the colonization and exploitation of Indigenous peoples in the Americas, while
others view his voyages as a turning point in human history, marking the beginning of
globalization and cultural exchange. The debate over Columbus’s legacy continues to this
day, with some calling for the reevaluation of his role in history and others defending his
achievements as an explorer. Ultimately, the perspective on Columbus’s legacy depends
on .

Which choice most logically completes the text?

A) how one weighs the positive aspects of exploration and cultural exchange against the
negative consequences of colonization and exploitation.
B) whether one believes in the historical accuracy of Columbus’s personal diaries and
letters.
C) the extent to which one believes the Indigenous peoples of the Americas would have
eventually made contact with Europeans without Columbus’s voyages.
D) how much emphasis one places on the role of the Viking settlements in shaping the
history of the Americas.

24. Roger Penrose, a renowned physicist, is known for his groundbreaking work involving
black holes and general relativity. Though Penrose has made significant contributions
to the field of physics, he also holds a deep interest in the connection between human
consciousness and quantum mechanics, suggesting that .

Which choice most logically completes the text?

A) Roger Penrose’s contributions to the field of physics are considered less significant
than his work on human consciousness.
B) Roger Penrose’s work in physics is often overshadowed by his fascination with human
consciousness.
C) Roger Penrose explores interdisciplinary topics and is not limited to a single area of
expertise.
D) Roger Penrose’s interest in human consciousness has led him to disprove the existence
of black holes.

127
25. Synthetic biology engineering involves the design and construction of new biological sys-
tems for various applications. For instance, scientists have used synthetic biology tech-
niques to produce biofuels and pharmaceuticals. these advancements, researchers
have also raised concerns about the potential risks and ethical implications of this field.

Which choice completes the text with the most logical transition?

A) Therefore
B) Additionally
C) In contrast
D) Despite

26. During the Elizabethan Era, England experienced a period of significant cultural growth,
with the flourishing of the arts, literature, and theater. Queen Elizabeth I avidly sup-
ported the arts, often commissioning works from prominent playwrights and poets. this
support led to the creation of some of the most famous works in English literature, in-
cluding William Shakespeare’s plays and sonnets.

Which choice completes the text with the most logical transition?

A) Indeed
B) Alternatively
C) However
D) In contrast

128
27. Thomas Je↵erson, the third President of the United States, was a polymath who excelled
in various fields such as architecture, horticulture, and political philosophy. He designed
his own home, Monticello, which showcased his architectural prowess. Je↵erson’s
passion for horticulture was evident in the extensive gardens he created at Monticello,
where he experimented with numerous plant species.

Which choice completes the text with the most logical transition?

A) In contrast,
B) On the other hand,
C) Nevertheless,
D) Similarly,

129
Chapter 19

Module 3 Answers

1. Correct Answer: D
Explanation: D: ’To emphasize Frida Kahlo’s resilience and the personal nature of her
artwork’ is the correct answer because the text highlights her ability to communicate her
pain and emotions through her work, as well as her perseverance despite her struggles.

2. Correct Answer: B
Explanation: B: ’To emphasize the historical importance of Queen Isabella I’s reign by
highlighting her achievements and their consequences’ is the correct answer because the
text mainly focuses on the significant reforms and decisions made during her reign, such
as strengthening the monarchy and sponsoring Columbus’s voyage.

3. Correct Answer: D
Explanation: D: ’By agreeing that Neil Armstrong’s hands-on teaching approach was a
valuable contribution to the field of aerospace engineering and the education of students’
is the correct answer because it aligns with the perspective presented in Text 2, which
highlights Armstrong’s contributions to the field of aerospace engineering through his
innovative teaching methods.

4. Correct Answer: B
Explanation: B: ’earned’ is the correct answer because it provides a finite verb that agrees
with the subject ’her’ and maintains the tense consistency within the sentence.

5. Correct Answer: B
Explanation: B: ’rotated’ is the correct answer because it properly indicates the past
passive tense, conveying the idea that the shaft is being rotated by an external force or
mechanism, which is necessary for the device to function.

130
6. Correct Answer: A
Explanation: A: ’burdens’ is the correct answer because it provides a finite present tense
verb that agrees with the subject ’implementation’ and correctly connects the main clause
with the rest of the sentence.

7. Correct Answer: A
Explanation: A: ’coincided’ is the correct answer because it provides a past tense verb
that agrees with the subject ’formation’ and highlights the relationship between the end
of World War II and the beginning of the Cold War.

8. Correct Answer: C
Explanation: C: ’The first Thanksgiving in 1621 was a harvest celebration between Pil-
grims and the Wampanoag tribe, predating its official establishment as a holiday in 1863.’
is the correct answer because it provides historical context by mentioning the original
event in 1621 and its official establishment as a holiday by President Lincoln in 1863.

9. Correct Answer: B
Explanation: B: ’Neighboring countries, such as Colombia, Peru, and Brazil, are experi-
encing strain on resources and infrastructure due to the influx of Venezuelan refugees.’
is the correct answer because it e↵ectively explains the impact of the Venezuelan refugee
crisis on neighboring countries by mentioning the strain on resources and infrastructure.

10. Correct Answer: B


Explanation: B: ’Elon Musk, as the CEO of Tesla and founder of SpaceX, plays a crucial
role in advancing sustainable transportation and space exploration.’ is the correct answer
because it e↵ectively uses relevant information from the notes to emphasize Elon Musk’s
role in both sustainable transportation and space exploration.

11. Correct Answer: C


Explanation: C: ’independence’ is the correct answer because it properly fits the context
of the passage, and it is a singular noun that doesn’t require any possessive or plural
modifications.

12. Correct Answer: A


Explanation: A: ’prohibit’ is the correct answer because it correctly uses the base form
of the verb to describe the purpose of passive fire protection methods.

131
13. Correct Answer: A
Explanation: A: ’evolution.’ is the correct answer because it provides a grammatically
correct and concise ending to the sentence, indicating the result of Darwin’s observations.

14. Correct Answer: A


Explanation: A: ’permanent revolution’ is the correct answer because it accurately de-
scribes the political theory developed by Leon Trotsky, which involved the extension of
Marxist principles and the necessity of continuous revolution to achieve a global commu-
nist society.

15. Correct Answer: B


Explanation: B: ’diverse’ is the correct answer because it describes the variety of terrains
that motocross riders face, making the sport more challenging and exciting to watch.

16. Correct Answer: A


Explanation: A: ’Socially ignored’ is the correct answer because it best represents the
theme of invisibility in Ralph Ellison’s novel, where the protagonist feels unseen and
unacknowledged by the society around him.

17. Correct Answer: D


Explanation: D: ’The text introduces the challenges faced by the characters in Stranger
Things.’ is the correct answer because the passage provides a general overview of the
series, mentioning the supernatural occurrences, the Upside Down, and the involvement
of both children and adults.

18. Correct Answer: C


Explanation: C: ’Thomas Edison’s invention of the phonograph revolutionized the field of
audio recording and playback.’ is the correct answer because the passage focuses on the
importance of the phonograph in the field of audio recording and its impact on people’s
experiences with music and other audio content.

19. Correct Answer: C


Explanation: C: ’The text introduces Werner Heisenberg’s uncertainty principle and its
significance in quantum mechanics.’ is the correct answer because it accurately reflects
the main idea of the text, which presents Heisenberg’s uncertainty principle and its rev-
olutionary impact on physics.

132
20. Correct Answer: D
Explanation: D: ’The overall trade among Mercosur members has stagnated since the
bloc’s inception.’ is the correct answer because it demonstrates that the Mercosur has not
achieved its goal of promoting economic cooperation among its members, which supports
the critics’ argument.

21. Correct Answer: A


Explanation: A: ’Cave-dwelling organisms in areas with artificial light exhibit increased
reproduction rates and stronger resilience.’ is the correct answer because it directly con-
tradicts the claim that artificial light has negative e↵ects on cave-dwelling organisms, by
showing positive e↵ects instead.

22. Correct Answer: B


Explanation: B: ’Many of Lewis’s scholarly works on medieval and Renaissance literature
became foundational texts in these fields of study.’ is the correct answer because it directly
supports the claim that C.S. Lewis had a profound impact on the study of these literary
periods.

23. Correct Answer: A


Explanation: A: ’how one weighs the positive aspects of exploration and cultural exchange
against the negative consequences of colonization and exploitation’ is the correct answer
because it directly addresses the debate over Columbus’s legacy mentioned in the passage
and highlights the contrasting perspectives on his actions.

24. Correct Answer: C


Explanation: C: ’Roger Penrose explores interdisciplinary topics and is not limited to a
single area of expertise.’ is the correct answer because it highlights that Penrose’s interest
in human consciousness and quantum mechanics demonstrates his ability to delve into
various subjects beyond physics.

25. Correct Answer: D


Explanation: D: ’Despite’ is the correct answer because it indicates that the concerns
raised by researchers are contrasted with the advancements made in synthetic biology
engineering.

133
26. Correct Answer: A
Explanation: A: ’Indeed’ is the correct answer because it emphasizes the significance
of Queen Elizabeth I’s support for the arts, demonstrating how this support led to the
creation of famous works in English literature.

27. Correct Answer: D


Explanation: D: ’Similarly’ is the correct answer because it shows that Je↵erson’s passion
for horticulture is another example of his wide range of interests and abilities, similar to
his architectural prowess.

134
Chapter 20

Module 4

Reading and Writing - 27 Questions

DIRECTIONS: The questions in this section are designed to test your reading and writing
abilities. Each question will provide you with one or more passages. It is important to
thoroughly read each passage and question before selecting the best answer based on the
information provided. All of the questions in this section are multiple-choice and will have
four options to choose from. There is only one correct answer for each question.

1. Energy storage is essential for balancing the fluctuations in supply and demand within the
energy sector. Among the many energy storage solutions, the pumped-storage hydropower
(PSH) system stands out due to its high efficiency and large-scale capacity. The PSH
system works by pumping water to a higher elevation during low-demand periods and
releasing it during high-demand periods to produce electricity through turbines. This
process allows for a more stable grid operation and reduces the need for fossil fuel-based
backup generators, thus contributing to a greener energy infrastructure.

Which choice best states the main purpose of the text?

A) To provide a detailed explanation of how the pumped-storage hydropower system


functions mechanically
B) To discuss the negative environmental impacts associated with using fossil fuel-based
backup generators
C) To emphasize the importance of energy storage and highlight the advantages of
pumped-storage hydropower systems
D) To argue that the energy sector should focus solely on implementing pumped-storage
hydropower solutions

135
2. Seymour Benzer, a renowned physicist and biologist, pioneered the field of behavioral
genetics by studying the fruit fly Drosophila. His groundbreaking experiments demon-
strated that mutations in single genes could significantly impact an organism’s behavior.
Benzer’s work laid the foundation for understanding the genetic basis of behavior, leading
to further research in various species, including humans. Today, his discoveries continue
to influence neuroscience, genetics, and the study of complex behaviors.

Which choice best states the main purpose of the text?

A) To argue that Seymour Benzer’s work on behavioral genetics has largely been over-
looked by modern scientists and researchers
B) To inform readers about the specific experiments that Seymour Benzer conducted on
fruit flies and their behavioral patterns
C) To highlight Seymour Benzer’s pioneering work in behavioral genetics and its lasting
impact on the scientific community
D) To explain the detailed process of how mutations in single genes can have a significant
influence on an organism’s behavior

3. Text 1: In The Phantom of the Opera, the author creates an atmosphere of mystery
surrounding the character of the Phantom, also known as Erik. Erik is a skilled musician,
architect, and inventor, who lives a secluded life beneath the Paris Opera House. His
tragic past and disfigured appearance contribute to his enigmatic character. The author’s
use of vivid descriptions and suspenseful storytelling captivates readers, as they strive to
learn more about the Phantom’s true identity.

Text 2: Critics of The Phantom of the Opera often discuss the novel’s exploration of
unrequited love and obsession. The Phantom’s infatuation with the young singer Christine
Daaé becomes the central focus of the story, as he manipulates events to make her a star.
This unhealthy obsession ultimately leads to the Phantom’s downfall, as he is unable to
win Christine’s love. Some critics argue that the novel serves as a cautionary tale against
the dangers of obsession.

Based on the texts, how would critics (Text 2) most likely respond to the “atmosphere of
mystery” discussed in Text 1?

A) By praising the author’s use of mystery to keep readers engaged and invested in the
story
B) By acknowledging the atmosphere of mystery but placing more emphasis on the
novel’s exploration of unrequited love and obsession

136
C) By criticizing the author for creating an unrealistic and overly dramatic portrayal of
the Phantom’s character
D) By suggesting that the Phantom’s mysterious nature detracts from the story’s overall
themes and message

4. As the demand for renewable energy increases, the need for efficient energy storage also
rises. In response to this, scientists have developed a new type of battery called the
flow battery. Flow batteries can store large amounts of energy by chemical
components dissolved in liquids contained within the system.

Which choice completes the text so that it conforms to the conventions of Standard
English?

A) utilized
B) to utilize
C) utilizing
D) will utilize

5. Emily Dickinson’s unique writing style often includes dashes, unconventional punctuation,
and capitalization. Her poems frequently feature enigmatic language and unusual syntax,
which to their mysterious allure and continue to intrigue readers today.

Which choice completes the text so that it conforms to the conventions of Standard
English?

A) contributing
B) contribute
C) to contribute
D) contributed

137
6. Textile engineering is a diverse field that includes the development of innovative fibers
and materials. In the 20th century, researchers a new class of synthetic textiles,
including polyester and nylon, which revolutionized the fashion and clothing industries.

Which choice completes the text so that it conforms to the conventions of Standard
English?

A) creating
B) to create
C) created
D) has created

7. During Abraham Lincoln’s presidency, the United States faced a critical time as the
nation was divided by the Civil War. Despite the numerous challenges, Lincoln was able
to crucial victories, such as the Emancipation Proclamation and the passage of
the Thirteenth Amendment, which abolished slavery.

Which choice completes the text so that it conforms to the conventions of Standard
English?

A) securing
B) secure
C) to secure
D) secured

138
8. While researching a topic, a student has taken the following notes:

• Brain-machine interfaces (BMIs) are being developed to enable direct communication


between the human brain and external devices.
• BMIs have potential applications in areas such as neuroprosthetics, rehabilitation,
and communication for people with severe disabilities.
• One challenge in BMI development is finding a balance between invasiveness and the
quality of the recorded neural signals.
• Non-invasive techniques, such as electroencephalography (EEG), o↵er lower signal
quality but have fewer risks compared to invasive methods like intracortical micro-
electrode arrays.

The student wants to emphasize the potential applications of brain-machine interfaces.


Which choice most e↵ectively uses relevant information from the notes to accomplish this
goal?

A) Brain-machine interfaces are being developed for direct communication between the
human brain and external devices, but challenges exist in finding a balance between
invasiveness and signal quality.
B) Brain-machine interfaces have the potential to revolutionize neuroprosthetics, reha-
bilitation, and communication for individuals with severe disabilities.
C) Non-invasive techniques such as EEG o↵er lower signal quality but have fewer risks
compared to invasive methods like intracortical microelectrode arrays in the devel-
opment of brain-machine interfaces.
D) The development of brain-machine interfaces involves addressing challenges in record-
ing neural signals, such as finding a balance between invasiveness and signal quality.

139
9. While researching a topic, a student has taken the following notes:

• George W. Bush served as the 43rd President of the United States.


• He pursued a foreign policy based on the concept of preemption.
• Bush’s administration launched the Iraq War in 2003.
• The President’s Emergency Plan for AIDS Relief (PEPFAR) was a significant ini-
tiative during his presidency.

Which choice most e↵ectively uses relevant information from the notes to illustrate a key
aspect of George W. Bush’s foreign policy?

A) George W. Bush was the 43rd President of the United States and focused on foreign
policy.
B) The President’s Emergency Plan for AIDS Relief (PEPFAR) was a major initiative
during George W. Bush’s presidency.
C) During his presidency, George W. Bush focused on both domestic and international
initiatives, such as PEPFAR.
D) George W. Bush’s foreign policy centered on preemption, leading to the launch of
the Iraq War in 2003.

10. While researching a topic, a student has taken the following notes:

• The Internet of Things (IoT) refers to the network of interconnected devices that
communicate and exchange data with each other.
• IoT devices can range from everyday household appliances to industrial machinery.
• IoT technology can improve energy efficiency by monitoring and adjusting energy
consumption in real-time.
• Security and privacy concerns are major challenges in the implementation and de-
velopment of IoT systems.

Which choice most e↵ectively uses relevant information from the notes to highlight a
benefit of IoT technology?

A) IoT technology refers to the network of interconnected devices that communicate and
exchange data, and it includes everyday household appliances.
B) The Internet of Things comprises a wide range of devices, from household appliances
to industrial machinery, and faces challenges in security and privacy.
C) Security and privacy concerns are major challenges that need to be addressed in the
development of IoT systems, which include various devices.

140
D) IoT technology can enhance energy efficiency by monitoring and adjusting energy
consumption in real-time through interconnected devices.

11. The Monroe Doctrine, a significant foreign policy statement issued by President James
Monroe in 1823, aimed to prevent European nations from interfering in the a↵airs of
newly independent Latin American countries. It declared that any further colonization
by European powers in the Americas would be viewed as a by the United States.

Which choice completes the text so that it conforms to the conventions of Standard
English?

A) aggression hostile
B) hostile aggression,
C) act, hostile
D) hostile act

12. Squash is a popular racket sport that requires both physical stamina and mental agility.
The game has its origins in the 19th century, when it was developed at Harrow School in
England. The students would play in a small, confined space, hitting a soft rubber ball
against a wall with their This early version of squash was more about control
and precision than power.

Which choice completes the text so that it conforms to the conventions of Standard
English?

A) racket’s
B) rackets’
C) racket’s’
D) rackets

141
13. Alice Walker, an acclaimed African American author, is known for her contributions to
the literary world. Her most famous work, The Color Purple, won her the Pulitzer Prize
for Fiction in 1983, and is often regarded as a powerful piece of literature addressing the
of African American women in the early 20th century.

Which choice completes the text so that it conforms to the conventions of Standard
English?

A) struggles, particularly
B) struggles particularly,
C) struggles; particularly
D) struggles particularly

14. The Muppet Show, created by Jim Henson, was a groundbreaking television series that
combined puppetry, comedy, and music. Since its debut in 1976, the show has been
by critics for its innovative format and enduring appeal to both children and
adults.

Which choice completes the text with the most logical and precise word or phrase?

A) criticized
B) ignored
C) lauded
D) tamed

142
15. Derivatives are financial contracts whose value is derived from the value of an underlying
asset, such as stocks or commodities. The two most common types of derivatives are op-
tions and futures, which can be used for various purposes, including , speculation,
and managing risk.

Which choice completes the text with the most logical and precise word or phrase?

A) hedging
B) gambling
C) experimenting
D) entertaining

16. During the Gettysburg Address, Abraham Lincoln emphasized the importance of a ’gov-
ernment of the people, by the people, for the people,’ highlighting the democratic princi-
ples that the nation was on.

Which choice completes the text with the most logical and precise word or phrase?

A) safeguarded
B) amused
C) founded
D) deviated

143
17. During the Brazilian Empire (1822-1889), which followed the country’s independence
from Portugal, Brazil experienced significant economic and political growth. Under the
rule of Emperors Pedro I and Pedro II, Brazil expanded its territory, developed a strong
agricultural economy, and became an important global trade partner. However, the empire
faced challenges such as slavery, regional conflicts, and pressure for a more democratic
government.

Which choice best states the main idea of the text?

A) The Brazilian Empire experienced growth and challenges during its rule after gaining
independence.
B) Emperors Pedro I and Pedro II were primarily responsible for Brazil’s political sta-
bility.
C) The Brazilian Empire’s focus on agricultural development led to the abolition of
slavery.
D) Brazil’s economic success during the empire era was solely due to its global trade
partnerships.

18. A financial analyst recently published a report on XYZ Company, examining its stock
performance over the past five years. The report highlighted several variables that po-
tentially influenced the stock’s value, including market trends, industry developments,
and company-specific factors. It concluded that while external factors played a role, the
company’s strong management and innovative strategies were the primary drivers behind
its consistent growth.

Which choice best states the main idea of the text?

A) XYZ Company’s stock value is solely determined by uncontrollable external factors.


B) The financial analyst’s report solely focuses on the stock market trends a↵ecting XYZ
Company.
C) The company’s stock performance is primarily attributed to its strong management
and innovative strategies.
D) The financial analyst’s report is critical of XYZ Company’s management and strate-
gies.

144
19. Abraham Lincoln, the 16th President of the United States, is remembered for his in-
spirational speeches and leadership during the American Civil War. Although he had
humble beginnings as a self-educated lawyer, his eloquent words and determination to
preserve the Union made him a powerful figure. Lincoln’s Emancipation Proclamation
and Gettysburg Address exemplified his devotion to the principles of freedom, equality,
and unity.

Which choice best states the main idea of the text?

A) The text describes the various challenges that Abraham Lincoln faced during his
tenure as the 16th President of the United States.
B) The text focuses on Abraham Lincoln’s self-education and how it played a crucial
role in his development as a lawyer and politician.
C) The text highlights Abraham Lincoln’s humble beginnings and his transformative
impact as a leader through his eloquent speeches and actions.
D) The text provides a detailed analysis of Abraham Lincoln’s Emancipation Proclama-
tion and Gettysburg Address in the context of American history.

20. During the Napoleonic Wars, British naval officer Thomas Cochrane successfully utilized
deception tactics. His innovative use of ruses and false signals helped the British Navy
secure crucial victories, ultimately contributing to the defeat of Napoleon’s forces.

Which finding, if true, would most strongly support the claim that Cochrane’s deception
tactics played a significant role in the British Navy’s success?

A) Cochrane’s ideas on naval tactics were published in a book that became popular
among European military strategists.
B) Cochrane’s deception tactics were later adopted by other naval forces in the 19th
century.
C) Cochrane’s personal achievements as a naval officer were recognized with numerous
awards and promotions.
D) Cochrane’s deceptive tactics led to the capture of multiple French ships, weakening
Napoleon’s naval power.

145
21. Smart lighting systems can detect human presence and adjust the lighting accordingly,
thus conserving energy. A recent study analyzed the energy consumption of buildings
with and without smart lighting technology.

Which finding, if true, would most strongly support the claim that smart lighting systems
contribute to energy conservation?

A) Buildings with smart lighting technology consumed significantly less energy compared
to those without such systems.
B) Some buildings with smart lighting technology had higher energy consumption due
to increased usage of other electronic devices.
C) The installation process of smart lighting systems is more energy-intensive than that
of traditional lighting systems.
D) Many users of smart lighting technology appreciate the convenience and aesthetic
appeal, but do not prioritize energy conservation.

22. Elon Musk recently proposed a futuristic mode of transportation called the Hyperloop.
The concept involves high-speed pods traveling within vacuum tubes, o↵ering a quicker
alternative to conventional transportation methods. Musk believes the Hyperloop could
revolutionize modern travel and contribute to sustainability.

Which finding, if true, would most directly support Musk’s belief that the Hyperloop
could revolutionize modern travel and contribute to sustainability?

A) Some critics argue that the cost and complexity of the Hyperloop may limit its
widespread adoption and impact on the transportation industry.
B) Elon Musk’s other ventures, such as Tesla and SpaceX, have also been focused on
revolutionizing their respective industries.
C) The development of the Hyperloop faces several engineering challenges, including
maintaining a near-vacuum environment in the tubes.
D) A study demonstrates that the Hyperloop’s energy consumption per passenger is
significantly lower than that of traditional transportation methods.

146
23. Environmental Systems Engineering (ESE) is a multidisciplinary field that focuses on
developing sustainable solutions to environmental challenges. One of the key objectives
of ESE is to optimize the use of natural resources while minimizing the environmental
impact. Therefore, ESE professionals often work closely with other disciplines such as
ecology, hydrology, and geophysics, as well as policymakers and stakeholders, to achieve
.

Which choice most logically completes the text?

A) an elimination of the need for collaborations with other disciplines in the field of
environmental engineering.
B) a strict adherence to traditional engineering principles regardless of environmental
concerns.
C) a comprehensive and integrated approach to addressing complex environmental issues.
D) a focus on short-term solutions without considering the long-term environmental
consequences.

24. World War I resulted in the collapse of four major empires and left countless nations in
disarray. In an attempt to prevent future conflicts, the Treaty of Versailles was signed,
which imposed harsh penalties on Germany, ultimately leading to .

Which choice most logically completes the text?

A) Germany becoming a powerful military force that dominated Europe


B) an immediate improvement in international relations and global peace
C) significant economic consequences and the rise of resentment in Germany
D) a strengthened alliance between Germany and the other Central Powers

147
25. In the field of network engineering, the concept of subnetting is crucial for managing IP
address allocation. Subnetting involves dividing a network into smaller, more manageable
subnetworks. , the use of virtual local area networks (VLANs) can further isolate
and segment network traffic for better control and security.

Which choice completes the text with the most logical transition?

A) Nonetheless
B) Alternatively
C) In contrast
D) Similarly

26. Napoleon Bonaparte, a military genius and skilled statesman, was known for his rapid
rise to power during the French Revolution. He successfully conquered much of Europe,
expanding the French Empire in the process. his military campaigns led to
significant changes in European political and social structures.

Which choice completes the text with the most logical transition?

A) For example
B) Alternatively
C) Nevertheless
D) Consequently

148
27. Snorkeling is a popular recreational activity that allows individuals to explore the un-
derwater world. It requires minimal equipment, such as a mask, snorkel, and fins.
scuba diving, on the other hand, requires extensive training and specialized
gear. Snorkeling provides an accessible and cost-e↵ective option for those who wish to
enjoy the beauty of marine life without the complexities of scuba diving.

Which choice completes the text with the most logical transition?

A) In contrast
B) Although
C) Similarly
D) Furthermore

149
Chapter 21

Module 4 Answers

1. Correct Answer: C
Explanation: C: ’To emphasize the importance of energy storage and highlight the ad-
vantages of pumped-storage hydropower systems’ is the correct answer because the text
discusses the necessity of energy storage and the benefits of using PSH systems, such as
high efficiency, large-scale capacity, and contribution to greener energy infrastructure.

2. Correct Answer: C
Explanation: C: ’To highlight Seymour Benzer’s pioneering work in behavioral genetics
and its lasting impact on the scientific community’ is the correct answer because the
passage provides an overview of Benzer’s groundbreaking work, its significance in the
field, and its continuing influence on various areas of research.

3. Correct Answer: B
Explanation: B: ’By acknowledging the atmosphere of mystery but placing more emphasis
on the novel’s exploration of unrequited love and obsession’ is the correct answer because
Text 2 focuses on the critics’ perspective on the novel’s themes of unrequited love and
obsession, rather than the atmosphere of mystery surrounding the Phantom’s character.

4. Correct Answer: C
Explanation: C: ’utilizing’ is the correct answer because it provides the correct present
tense verb that matches the context of the passage and maintains the parallelism with
the other present tense verbs.

5. Correct Answer: B
Explanation: B: ’contribute’ is the correct answer because it provides the main clause

150
with a finite present tense verb that indicates the ongoing e↵ect of Dickinson’s writing
style on readers.

6. Correct Answer: C
Explanation: C: ’created’ is the correct answer because it provides the main clause with
the finite simple past tense verb, showing that the action of creating synthetic textiles
occurred during the 20th century.

7. Correct Answer: B
Explanation: B: ’secure’ is the correct answer because it provides the main clause with
a finite present tense verb to perform the action of the subject, ’Lincoln’, indicating how
he achieved the crucial victories.

8. Correct Answer: B
Explanation: B: ’Brain-machine interfaces have the potential to revolutionize neuropros-
thetics, rehabilitation, and communication for individuals with severe disabilities.’ is
the correct answer because it e↵ectively emphasizes the potential applications of brain-
machine interfaces.

9. Correct Answer: D
Explanation: D: ’George W. Bush’s foreign policy centered on preemption, leading to the
launch of the Iraq War in 2003.’ is the correct answer because it e↵ectively illustrates a
key aspect of George W. Bush’s foreign policy using the information from the notes.

10. Correct Answer: D


Explanation: D: ’IoT technology can enhance energy efficiency by monitoring and ad-
justing energy consumption in real-time through interconnected devices.’ is the correct
answer because it e↵ectively highlights a benefit of IoT technology by mentioning its
ability to improve energy efficiency through real-time monitoring and adjustments.

11. Correct Answer: D


Explanation: D: ’hostile act’ is the correct answer because it completes the sentence with
appropriate noun usage and maintains the conventions of Standard English.

151
12. Correct Answer: D
Explanation: D: ’rackets’ is the correct answer because it accurately describes the plural
form of the noun ’racket’, indicating that multiple students were using rackets to play the
game.

13. Correct Answer: A


Explanation: A: ’struggles, particularly’ is the correct answer because it uses a comma to
separate the main clause from the dependent clause, providing a clearer structure.

14. Correct Answer: C


Explanation: C: ’lauded’ is the correct answer because it indicates that critics have praised
The Muppet Show for its innovative format and enduring appeal.

15. Correct Answer: A


Explanation: A: ’hedging’ is the correct answer because it is a financial strategy that
helps investors protect their investments by reducing the risk of potential losses.

16. Correct Answer: C


Explanation: C: ’founded’ is the correct answer because it shows the establishment of the
nation on democratic principles, which is the focus of the Gettysburg Address.

17. Correct Answer: A


Explanation: A: ’The Brazilian Empire experienced growth and challenges during its rule
after gaining independence.’ is the correct answer because it encapsulates the economic
and political growth, as well as the challenges faced by the empire.

18. Correct Answer: C


Explanation: C: ’The company’s stock performance is primarily attributed to its strong
management and innovative strategies.’ is the correct answer because the passage states
that although external factors played a role, the company’s management and strategies
were the primary drivers behind its consistent growth.

19. Correct Answer: C


Explanation: C: ’The text highlights Abraham Lincoln’s humble beginnings and his trans-
formative impact as a leader through his eloquent speeches and actions.’ is the correct
answer because the passage outlines his early life, his powerful speeches, and his leadership
during the Civil War.

152
20. Correct Answer: D
Explanation: D: ’Cochrane’s deceptive tactics led to the capture of multiple French ships,
weakening Napoleon’s naval power.’ is the correct answer because it directly connects
Cochrane’s deception tactics to the British Navy’s success in the Napoleonic Wars.

21. Correct Answer: A


Explanation: A: ’Buildings with smart lighting technology consumed significantly less
energy compared to those without such systems’ is the correct answer because it directly
supports the claim that smart lighting systems contribute to energy conservation.

22. Correct Answer: D


Explanation: D: ’A study demonstrates that the Hyperloop’s energy consumption per
passenger is significantly lower than that of traditional transportation methods.’ is the
correct answer because it directly supports the idea that the Hyperloop could revolutionize
travel and contribute to sustainability by being more energy efficient.

23. Correct Answer: C


Explanation: C: ’a comprehensive and integrated approach to addressing complex environ-
mental issues.’ is the correct answer because the passage highlights the multidisciplinary
nature of ESE and its focus on sustainable solutions to environmental challenges, which
implies a need for a comprehensive approach.

24. Correct Answer: C


Explanation: C: ’significant economic consequences and the rise of resentment in Ger-
many’ is the correct answer because it logically follows the imposition of harsh penalties
on Germany by the Treaty of Versailles.

25. Correct Answer: D


Explanation: D: ’Similarly’ is the correct answer because it indicates that the use of
VLANs is another technique, like subnetting, that helps manage and control network
traffic.

26. Correct Answer: D


Explanation: D: ’Consequently’ is the correct answer because it indicates that the result
or consequence of Napoleon’s military campaigns was significant changes in European
political and social structures.

153
27. Correct Answer: A
Explanation: A: ’In contrast’ is the correct answer because it highlights the di↵erence be-
tween snorkeling and scuba diving, emphasizing the simplicity and accessibility of snorke-
ling compared to the more complex scuba diving.

154
Chapter 22

Module 5

Reading and Writing - 27 Questions

DIRECTIONS: The questions in this section are designed to test your reading and writing
abilities. Each question will provide you with one or more passages. It is important to
thoroughly read each passage and question before selecting the best answer based on the
information provided. All of the questions in this section are multiple-choice and will have
four options to choose from. There is only one correct answer for each question.

1. Handball is a unique sport that combines elements of basketball, soccer, and rugby. It
originated in Northern Europe in the late 19th century, and over time, the sport has
evolved to become faster-paced and more dynamic. Professional handball players display
incredible athleticism, dexterity, and agility, while the team-oriented nature of the sport
fosters camaraderie and cooperation. Handball has gained global recognition, with the
International Handball Federation hosting championships and the sport being included
in the Olympic Games.

Which choice best states the main purpose of the text?

A) To argue that handball should be considered a more popular sport than basketball,
soccer, or rugby due to its unique qualities
B) To explore the various strategies employed by professional handball players during
high-stakes championship matches
C) To provide an overview of handball, touching on its origins, unique characteristics,
and international recognition
D) To detail the specific training regimens and techniques used by handball players to
improve their athleticism and dexterity

155
2. Biomedical engineering combines engineering principles with medical sciences to design
and create equipment, devices, and software used in healthcare. From artificial organs to
advanced imaging systems, biomedical engineering has revolutionized the medical field.
This interdisciplinary approach has not only improved patient care but also led to signif-
icant discoveries in disease diagnosis and prevention.

Which choice best states the main purpose of the text?

A) To discuss the specific techniques and principles used in biomedical engineering


B) To argue that biomedical engineering is the most important field in modern medicine
C) To highlight the importance of biomedical engineering in revolutionizing healthcare
and medical research
D) To provide a comprehensive history of the development of biomedical engineering

3. Text 1: Environmental economist Dr. Julia Stewart argues that placing a higher value on
ecosystem services, such as clean air and water, will lead to better conservation practices
and economic incentives for individuals and businesses. She believes that recognizing the
true worth of these services can help governments create e↵ective policies to protect the
environment.

Text 2: Dr. Michael Green, another environmental economist, suggests that while valu-
ing ecosystem services is essential, it may not be enough to drive significant change. He
emphasizes the importance of also considering the costs of transitioning to more sustain-
able practices and investing in the development of innovative technologies to reduce the
environmental impact.

Based on the texts, how would Dr. Michael Green (Text 2) most likely respond to the
“placing a higher value on ecosystem services” discussed in Text 1?

A) By arguing that placing higher value on ecosystem services is unnecessary and coun-
terproductive to conservation e↵orts
B) By acknowledging the importance of valuing ecosystem services, but emphasizing the
need to also consider the costs of transitioning to sustainable practices
C) By agreeing with Dr. Julia Stewart’s approach and suggesting that this is the only
way to protect the environment e↵ectively
D) By suggesting that focusing on ecosystem services diverts attention from the more
urgent issue of developing innovative technologies

156
4. In the field of computer engineering, cache memory plays a crucial role in improving the
performance of a computer system. It is a small, high-speed memory block that stores
copies of frequently used data, allowing the computer to the data more quickly
than if it had to access the main memory.

Which choice completes the text so that it conforms to the conventions of Standard
English?

A) retrieve
B) to retrieve
C) retrieving
D) retrieved

5. The assassination of Archduke Franz Ferdinand in 1914 led to a domino e↵ect of events
that eventually World War I. The escalating tensions between the major powers
of Europe culminated in a global conflict that would last for over four years.

Which choice completes the text so that it conforms to the conventions of Standard
English?

A) to trigger
B) triggered
C) triggering
D) having triggered

157
6. Edith Wharton, a Pulitzer Prize-winning author, was known for her keen observation of
social life. Her novel The Age of Innocence, set in the 1870s, explores the challenges
faced by individuals as they navigate the societal norms and expectations of their time.
In the story, the protagonist Newland Archer the love of his life, Countess Ellen
Olenska.

Which choice completes the text so that it conforms to the conventions of Standard
English?

A) encountered
B) encounters
C) to encounter
D) encountering

7. Throughout his political career, Winston Churchill was known for his powerful speeches.
In 1940, during the early years of World War II, he delivered a speech to the House of
Commons, stating that Britain to fight the enemy on multiple fronts.

Which choice completes the text so that it conforms to the conventions of Standard
English?

A) would continue
B) continuing
C) to continue
D) had continued

158
8. While researching a topic, a student has taken the following notes:

• George Eliot, a pseudonym for Mary Ann Evans, was a prominent 19th-century
British novelist.
• Eliot’s works often focused on rural life and explored complex human relationships.
• Her novel Middlemarch is considered one of the greatest works of English literature.
• Middlemarch earned critical acclaim for its intricate character development and nar-
rative structure.

Which choice most e↵ectively uses relevant information from the notes to discuss the
significance of George Eliot’s Middlemarch?

A) George Eliot, also known as Mary Ann Evans, was a British novelist who wrote
Middlemarch.
B) Mary Ann Evans, who used the pseudonym George Eliot, was a 19th-century British
novelist.
C) George Eliot’s works, including Middlemarch, often focused on rural life and human
relationships.
D) Middlemarch, written by George Eliot, is highly regarded for its complex characters
and narrative structure, earning its place as one of the greatest works of English
literature.

9. While researching a topic, a student has taken the following notes:

• Property tax is a primary source of revenue for local governments.


• Property tax rates vary by jurisdiction, often determined by local needs and priori-
ties.
• Some argue that property taxes are regressive, disproportionately a↵ecting lower-
income homeowners.
• Tax relief programs for seniors, disabled individuals, and low-income households exist
in some areas to mitigate the burden.

Which choice most e↵ectively uses relevant information from the notes to discuss the
relationship between property tax and local government funding?

A) Property tax, a primary revenue source for local governments, has rates that vary by
jurisdiction based on local needs and priorities.
B) Property tax rates are regressive, negatively impacting lower-income homeowners.

159
C) Tax relief programs exist for seniors, disabled individuals, and low-income households
to provide financial support.
D) Property taxes are determined by the value of real estate and personal property,
influencing local government revenue.

10. While researching a topic, a student has taken the following notes:

• Ghostbusters is a 1984 American supernatural comedy film directed by Ivan Reit-


man.
• The film stars Bill Murray, Dan Aykroyd, and Harold Ramis as a group of eccentric
parapsychologists who start a ghost-catching business in New York City.
• Ghostbusters was a critical and commercial success, grossing $295.2 million world-
wide.
• The film’s theme song, ’Ghostbusters’ by Ray Parker Jr., became a popular hit and
received an Academy Award nomination for Best Original Song.

Which choice most e↵ectively uses relevant information from the notes to emphasize the
impact of Ghostbusters on the film industry and popular culture?

A) Ghostbusters, directed by Ivan Reitman, is a supernatural comedy film released in


1984.
B) Ghostbusters was a critical and commercial success, grossing $295.2 million world-
wide, and its theme song by Ray Parker Jr. received an Academy Award nomination.
C) Bill Murray, Dan Aykroyd, and Harold Ramis starred as eccentric parapsychologists
in Ghostbusters.
D) The film Ghostbusters focused on a group of parapsychologists who started a ghost-
catching business in New York City.

160
11. Neil deGrasse Tyson, an astrophysicist, author, and science communicator, has inspired
millions through his books, lectures, and television appearances. Despite his tremendous
success, Tyson remains humble and dedicated to his work, focusing on the of
scientific knowledge.

Which choice completes the text so that it conforms to the conventions of Standard
English?

A) dissemination’s
B) disseminations
C) dissemination
D) disseminations’

12. Smart homes can greatly increase energy efficiency and comfort, thanks to their ability
to communicate with appliances and adapt to changes in daily routines. For example, a
smart thermostat can learn a homeowner’s preferences and adjust the home’s temperature
the homeowner’s arrival.

Which choice completes the text so that it conforms to the conventions of Standard
English?

A) prior;
B) prior to
C) prior,
D) prior at

161
13. Pablo Neruda, a renowned Chilean poet, wrote numerous works throughout his lifetime
that captured his love for the natural world. In one of his celebrated collections, ’Elemental
Odes,’ Neruda explores his fascination with everyday objects, such as a lemon, an onion,
and a in a distinctively lyrical style.

Which choice completes the text so that it conforms to the conventions of Standard
English?

A) a pair of shoe
B) pair of shoes
C) pairs of shoes
D) pair of shoe

14. In economics, the concept of marginal revenue refers to the increase in revenue that results
from the sale of one additional unit of a product or service. Marginal revenue is a crucial
factor in determining the most level of production for a firm.

Which choice completes the text with the most logical and precise word or phrase?

A) intuitive
B) consistent
C) profitable
D) enjoyable

162
15. During Barack Obama’s presidency, he emphasized the importance of diplomacy and in-
ternational cooperation, leading to the signing of the Paris Agreement on climate change.
Despite facing significant opposition, Obama’s e↵orts to global relations were
largely successful.

Which choice completes the text with the most logical and precise word or phrase?

A) overlook
B) disregard
C) strengthen
D) disrupt

16. Social robots, designed to assist and interact with humans, have been increasingly used
in various settings, including healthcare and education. Despite their potential benefits,
some critics argue that these robots could human interaction and lead to negative
consequences.

Which choice completes the text with the most logical and precise word or phrase?

A) supplant
B) catalog
C) promote
D) intensify

163
17. Bioremediation, a process that uses microorganisms to break down harmful substances, is
an eco-friendly alternative to traditional cleanup methods. This approach e↵ectively deals
with pollution in soil, water, and air by utilizing microbes that can digest contaminants
and transform them into harmless byproducts. Bioremediation has been successful in
treating oil spills, industrial waste, and agricultural runo↵, demonstrating its versatility
in addressing various environmental issues.

Which choice best states the main idea of the text?

A) Bioremediation’s ability to tackle multiple forms of pollution is limited by its reliance


on microorganisms.
B) Bioremediation primarily focuses on addressing pollution in the air, as it is more
difficult to treat than soil and water.
C) Bioremediation is an eco-friendly method for addressing various environmental issues
by using microbes to break down contaminants.
D) Bioremediation has not yet proven e↵ective in treating major environmental issues,
despite its potential applications.

18. Elizabeth Barrett Browning was a distinguished 19th-century poet who gained notori-
ety for her unique style, which blended traditional and innovative techniques. She was
admired for her use of intricate rhyme schemes, as well as her thematic exploration of
love, political change, and social injustice. Despite facing numerous obstacles, including
ill health and societal expectations for women, Browning persevered and produced a vast
and influential body of work.

Which choice best states the main idea of the text?

A) Elizabeth Barrett Browning’s poetry focuses primarily on the theme of love.


B) Elizabeth Barrett Browning faced many challenges due to societal expectations for
women.
C) Elizabeth Barrett Browning was an influential poet known for her unique style and
thematic exploration.
D) Elizabeth Barrett Browning’s work is characterized solely by its innovative techniques.

164
19. The passage highlights Niels Bohr’s groundbreaking model of the hydrogen atom. Bohr
proposed that electrons orbit the nucleus in discrete energy levels, and they can only
transition between these levels by emitting or absorbing a specific amount of energy. His
model helped explain the previously inexplicable patterns of atomic spectra and laid the
foundation for the development of quantum mechanics.

Which choice best states the main idea of the text?

A) The passage discusses Niels Bohr’s model of the hydrogen atom and its significance
in explaining atomic spectra and paving the way for quantum mechanics.
B) The passage emphasizes the importance of Niels Bohr’s model in addressing the
limitations of classical physics.
C) The passage focuses on Niels Bohr’s contribution to the understanding of atomic
structure and its influence on quantum mechanics.
D) The passage primarily examines Niels Bohr’s discovery of energy levels and the role
of electrons in determining the properties of atoms.

20. In geotechnical engineering, soil classification is essential for determining the suitability
of soil for construction projects. A recent study investigated the accuracy of various
classification methods across diverse soil types and regions.

Which finding, if true, would most strongly support the claim that a specific soil classifi-
cation method is highly accurate?

A) The method consistently produces correct classifications for a wide range of soil types
and regions.
B) The method is widely used by geotechnical engineers and has been endorsed by a
professional organization.
C) The method is based on a combination of physical and chemical properties of soil
samples.
D) The method was developed by a renowned geotechnical engineer with extensive ex-
perience in the field.

165
21. Country X has experienced a significant trade surplus in recent years, primarily due to
the rapid growth of its manufacturing sector. This sector’s expansion has been fueled by
increased foreign investments and advancements in technology.

Which finding, if true, would most directly support the assertion that the growth of
Country X’s manufacturing sector is the primary reason for its trade surplus?

A) Country X has a well-established education system that emphasizes the importance


of technological innovation.
B) The increase in foreign investments and technological advancements in Country X’s
manufacturing sector aligns with the timeline of the trade surplus.
C) Country X’s trade surplus has been consistent over the past decade, with no significant
fluctuations.
D) Other sectors in Country X’s economy, such as agriculture and services, have remained
relatively stable during the trade surplus period.

22. Modern sailboats often feature a keel, a structure that extends into the water to provide
stability and prevent capsizing. Some sailboat designers argue that a heavier keel improves
a sailboat’s overall performance.

Which finding, if true, would most directly undermine the designers’ argument?

A) Sailboats with lighter keels consistently have higher top speeds but lower stability.
B) A sailboat’s keel weight has no significant impact on its ability to sail in rough waters.
C) Sailboats with heavier keels exhibit reduced maneuverability and slower average
speeds.
D) The materials used in constructing a keel can greatly influence the sailboat’s perfor-
mance.

166
23. W.H. Auden was a renowned poet known for his stylistic and technical achievements.
His work ranged from political and social commentary to love poems. Critics argue
that his early work was more innovative and daring, whereas his later work became more
introspective and traditional. This shift in Auden’s poetry may be attributed to .

Which choice most logically completes the text?

A) a lack of inspiration during the later years of his career


B) changes in his personal life and experiences that influenced his writing
C) external pressures to conform to a specific writing style
D) his dissatisfaction with the impact of his earlier works

24. Fidel Castro was a controversial figure in Cuban history. While some people praise him
for his role in the Cuban Revolution and the establishment of socialism in Cuba, others
criticize him for his authoritarian rule and human rights abuses. His legacy remains
.

Which choice most logically completes the text?

A) universally celebrated for his resilience


B) completely disregarded by the international community
C) solely focused on his economic achievements
D) a subject of intense debate and varying perspectives

167
25. The Challenger disaster occurred on January 28, 1986, when the Space Shuttle Chal-
lenger exploded shortly after lifto↵, resulting in the death of all seven crew members. A
major cause of this tragedy was the failure of an O-ring seal in the shuttle’s solid rocket
booster. Engineers had expressed concerns about the performance of these O-rings in
cold temperatures, and the launch occurred on an unusually cold day. an inves-
tigation revealed that communication breakdowns and management issues within NASA
contributed to the decision to proceed with the launch despite these concerns.

Which choice completes the text with the most logical transition?

A) Furthermore
B) In addition
C) However
D) On the other hand

26. Princess Diana was not only known for her humanitarian work and her iconic fashion
sense, but also for her tumultuous relationship with the media. Diana was often hounded
by paparazzi, which led to her feeling overwhelmed and invaded. in 1993, she
decided to withdraw from public life temporarily to protect her own well-being and her
sons’ privacy.

Which choice completes the text with the most logical transition?

A) Similarly,
B) As a result,
C) On the other hand,
D) In contrast,

168
27. A market economy is an economic system in which the production and distribution of
goods and services are determined by supply and demand. This type of economy allows
businesses to thrive and encourages innovation, as competition drives companies to create
new products and improve existing ones. despite the numerous advantages, a
market economy also has its share of drawbacks, such as income inequality and potential
environmental degradation.

Which choice completes the text with the most logical transition?

A) Moreover
B) However
C) In addition
D) On the contrary

169
Chapter 23

Module 5 Answers

1. Correct Answer: C
Explanation: C: ’To provide an overview of handball, touching on its origins, unique
characteristics, and international recognition’ is the correct answer because the passage
o↵ers a brief introduction to handball, discussing its roots, the qualities that set it apart
from other sports, and its global presence.

2. Correct Answer: C
Explanation: C: ’To highlight the importance of biomedical engineering in revolutioniz-
ing healthcare and medical research’ is the correct answer because the text focuses on
the positive impact of biomedical engineering on healthcare, patient care, and medical
discoveries.

3. Correct Answer: B
Explanation: B: ’By acknowledging the importance of valuing ecosystem services, but
emphasizing the need to also consider the costs of transitioning to sustainable practices’
is the correct answer because Text 2 suggests that Dr. Michael Green agrees with valuing
ecosystem services but also emphasizes the importance of considering other factors, such as
the costs of transitioning to sustainable practices and investing in innovative technologies.

4. Correct Answer: A
Explanation: A: ’retrieve’ is the correct answer because it is a finite present tense verb that
indicates the action of the computer in accessing the data, conforming to the conventions
of Standard English.

170
5. Correct Answer: B
Explanation: B: ’triggered’ is the correct answer because it provides a finite past tense
verb, indicating that the assassination led to the start of World War I.

6. Correct Answer: B
Explanation: B: ’encounters’ is the correct answer because it provides the finite present
tense verb to perform the action of the subject (Newland Archer) and is consistent with
the other present tense verbs used to describe the events in the novel.

7. Correct Answer: A
Explanation: A: ’would continue’ is the correct answer because it provides a finite verb
in the future tense that is consistent with the context of the passage.

8. Correct Answer: D
Explanation: D: ’Middlemarch, written by George Eliot, is highly regarded for its com-
plex characters and narrative structure, earning its place as one of the greatest works of
English literature.’ is the correct answer because it e↵ectively discusses the significance
of Middlemarch by highlighting its critical acclaim and the reasons for its praise.

9. Correct Answer: A
Explanation: A: ’Property tax, a primary revenue source for local governments, has rates
that vary by jurisdiction based on local needs and priorities.’ is the correct answer because
it e↵ectively uses information from the notes to discuss the relationship between property
tax and local government funding.

10. Correct Answer: B


Explanation: B: ’Ghostbusters was a critical and commercial success, grossing $295.2
million worldwide, and its theme song by Ray Parker Jr. received an Academy Award
nomination.’ is the correct answer because it highlights both the financial success of the
film and the impact of its theme song on popular culture.

11. Correct Answer: C


Explanation: C: ’dissemination’ is the correct answer because it is a singular noun and
fits the context of the sentence, indicating the act of spreading scientific knowledge.

171
12. Correct Answer: B
Explanation: B: ’prior to’ is the correct answer because it properly connects the two
phrases and maintains the flow of the sentence, while adhering to the conventions of
Standard English.

13. Correct Answer: B


Explanation: B: ’pair of shoes’ is the correct answer because it correctly uses the singular
noun ’pair’ and the plural noun ’shoes’ to describe one set of two shoes.

14. Correct Answer: C


Explanation: C: ’profitable’ is the correct answer because it describes the level of produc-
tion that maximizes a firm’s revenue.

15. Correct Answer: C


Explanation: C: ’strengthen’ is the correct answer because it accurately describes Obama’s
e↵orts to improve global relations through diplomacy and international cooperation.

16. Correct Answer: A


Explanation: A: ’supplant’ is the correct answer because it accurately describes the con-
cern of critics that social robots may replace human interaction, leading to negative
consequences.

17. Correct Answer: C


Explanation: C: ’Bioremediation is an eco-friendly method for addressing various envi-
ronmental issues by using microbes to break down contaminants’ is the correct answer
because the passage explains the process of bioremediation and emphasizes its e↵ective-
ness and versatility in dealing with pollution in soil, water, and air.

18. Correct Answer: C


Explanation: C: ’Elizabeth Barrett Browning was an influential poet known for her unique
style and thematic exploration.’ is the correct answer because it captures the essence of
the passage, mentioning her unique style, thematic exploration, and her influence as a
poet.

172
19. Correct Answer: A
Explanation: A: ’The passage discusses Niels Bohr’s model of the hydrogen atom and its
significance in explaining atomic spectra and paving the way for quantum mechanics.’ is
the correct answer because it accurately captures the main idea of the passage, which
focuses on Bohr’s model and its importance in understanding atomic spectra and the
development of quantum mechanics.

20. Correct Answer: A


Explanation: A: ’The method consistently produces correct classifications for a wide range
of soil types and regions.’ is the correct answer because this finding directly supports the
claim that the specific soil classification method is highly accurate, as it demonstrates its
e↵ectiveness across diverse soil types and regions.

21. Correct Answer: B


Explanation: B: ’The increase in foreign investments and technological advancements in
Country X’s manufacturing sector aligns with the timeline of the trade surplus.’ is the
correct answer because it directly connects the growth of the manufacturing sector with
the trade surplus by showing a correlation in their timelines.

22. Correct Answer: C


Explanation: C: ’Sailboats with heavier keels exhibit reduced maneuverability and slower
average speeds.’ is the correct answer because it directly contradicts the argument that
a heavier keel improves a sailboat’s overall performance.

23. Correct Answer: B


Explanation: B: ’changes in his personal life and experiences that influenced his writing’
is the correct answer because the passage implies that a shift occurred in Auden’s poetry,
and personal experiences can significantly impact an artist’s work.

24. Correct Answer: D


Explanation: D: ’a subject of intense debate and varying perspectives’ is the correct
answer because it acknowledges the di↵ering opinions on Fidel Castro’s role in Cuban
history.

25. Correct Answer: A


Explanation: A: ’Furthermore’ is the correct answer because it logically signals that
the investigation’s findings of communication breakdowns and management issues within

173
NASA are additional contributing factors to the disaster, beyond the O-ring failure and
cold temperatures.

26. Correct Answer: B


Explanation: B: ’As a result’ is the correct answer because it shows the consequence of
Princess Diana’s troubled relationship with the media and why she chose to withdraw
from public life.

27. Correct Answer: B


Explanation: B: ’However’ is the correct answer because it logically signals that the
sentence following it presents a contrast to the previous sentence, which discusses the
advantages of a market economy.

174
Chapter 24

Module 6

Reading and Writing - 27 Questions

DIRECTIONS: The questions in this section are designed to test your reading and writing
abilities. Each question will provide you with one or more passages. It is important to
thoroughly read each passage and question before selecting the best answer based on the
information provided. All of the questions in this section are multiple-choice and will have
four options to choose from. There is only one correct answer for each question.

1. In recent years, international trade agreements have become a point of contention among
various nations. Some argue that these agreements are detrimental to domestic industries,
leading to job losses and wage stagnation. Others contend that such agreements promote
global economic growth and foster international cooperation. By analyzing the pros and
cons of these agreements, policymakers are better equipped to make informed decisions
that benefit their respective countries.

Which choice best states the main purpose of the text?

A) To present the di↵ering perspectives on international trade agreements and their


e↵ects on global economy
B) To argue in favor of international trade agreements as a means of promoting economic
growth
C) To criticize international trade agreements for causing job losses and wage stagnation
D) To provide an in-depth analysis of a specific international trade agreement and its
implications

175
2. Ultimate Frisbee, a team sport played with a flying disc, has gained popularity worldwide
since its inception in the late 1960s. The sport emphasizes fair play and sportsmanship
through its unique feature, the ’Spirit of the Game.’ This principle encourages players to
compete with integrity and respect, relying on self-officiating and mutual trust between
opponents. Ultimate Frisbee’s non-contact nature and self-regulated gameplay foster a
positive environment that transcends winning and losing, making the sport appealing to
diverse communities.

Which choice best states the main purpose of the text?

A) To provide a detailed description of the rules and gameplay elements of Ultimate


Frisbee as a sport
B) To discuss the history of Ultimate Frisbee and its growth in popularity since the late
1960s
C) To compare and contrast Ultimate Frisbee with other team sports and their respective
gameplay features
D) To highlight Ultimate Frisbee’s unique emphasis on sportsmanship and self-regulation
through the ’Spirit of the Game’

3. Text 1: Napoleon Bonaparte, a military genius, was known for his brilliant strategies and
tactics in warfare. His army’s success was largely due to his ability to analyze complex
situations quickly and make e↵ective decisions.

Text 2: Napoleon’s leadership style was also marked by a relentless ambition and a thirst
for personal glory. Some historians argue that his desire for conquest and power often
overshadowed his military acumen, leading to costly mistakes and ultimately his downfall.

Based on the texts, how would the historians mentioned in Text 2 most likely respond to
the military genius discussed in Text 1?

A) By arguing that the historians in Text 2 are biased against Napoleon and are down-
playing his military accomplishments to discredit his legacy
B) By asserting that Napoleon’s military genius was exaggerated, and that his victories
were primarily due to the superior strength of the French army
C) By suggesting that Napoleon’s military genius was a myth and that his successes
were simply a result of luck and favorable circumstances
D) By acknowledging Napoleon’s strategic abilities, but emphasizing that his ambition
and quest for personal glory were detrimental to his long-term success

176
4. The Brazilian economic miracle, a period of rapid economic growth and industrialization,
occurred from the late 1960s to the early 1980s. During this time, Brazil’s GDP at
a remarkable rate, averaging around 8.5% per year, transforming the country into a major
global economic power.

Which choice completes the text so that it conforms to the conventions of Standard
English?

A) will grow
B) has grown
C) grew
D) growing

5. In quality engineering, a process capability index is a statistical measure that evaluates


how well a manufacturing process can produce output within specified tolerance limits.
A process with a high capability index a lower defect rate, ensuring consistent
product quality.

Which choice completes the text so that it conforms to the conventions of Standard
English?

A) has
B) having
C) to have
D) had

177
6. In geological engineering, the study of rock mechanics is essential for analyzing the sta-
bility of tunnels and caverns. Engineers must consider the rock’s initial state, including
factors such as the presence of fractures, before they any excavation projects to
ensure safety and e↵ectiveness.

Which choice completes the text so that it conforms to the conventions of Standard
English?

A) undertaking
B) to undertake
C) undertake
D) undertook

7. The Andean Community, established in 1969, aimed to promote the economic integration
and political cooperation of its member countries. Initially, the organization began with
four countries—Bolivia, Colombia, Ecuador, and Peru—but in 1973, Chile the
community.

Which choice completes the text so that it conforms to the conventions of Standard
English?

A) joining
B) to join
C) joined
D) will join

178
8. While researching a topic, a student has taken the following notes:

• Murray Gell-Mann was an American physicist who won the 1969 Nobel Prize in
Physics.
• Gell-Mann helped develop the theory of quantum chromodynamics.
• He introduced the concept of quarks, which are elementary particles and a funda-
mental constituent of matter.
• Gell-Mann also co-founded the Santa Fe Institute, a research center dedicated to the
study of complex systems.

Which choice most e↵ectively uses relevant information from the notes to emphasize Gell-
Mann’s most notable achievement?

A) Murray Gell-Mann was an American physicist known for his work in quantum chro-
modynamics and co-founding the Santa Fe Institute.
B) Gell-Mann’s work in quantum chromodynamics and his introduction of quarks revo-
lutionized the study of physics.
C) As a physicist and a founder of the Santa Fe Institute, Murray Gell-Mann was influ-
ential in a variety of scientific fields.
D) Murray Gell-Mann, who introduced the concept of quarks and won the 1969 Nobel
Prize in Physics, made groundbreaking contributions to our understanding of matter.

9. While researching a topic, a student has taken the following notes:

• Superman, a fictional character created by Jerry Siegel and Joe Shuster, first ap-
peared in Action Comics #1.
• Superman’s secret identity is Clark Kent, a mild-mannered reporter for the Daily
Planet.
• The character represents the ultimate good, defending humanity and upholding jus-
tice.
• One of Superman’s key abilities is his invulnerability, allowing him to withstand
extreme forces and conditions.

Which choice most e↵ectively uses relevant information from the notes to explain how
Superman embodies the ultimate good?

A) Superman represents the ultimate good by defending humanity and upholding justice
with his incredible abilities, such as invulnerability.
B) As Clark Kent, Superman works as a reporter for the Daily Planet.

179
C) Superman was created by Jerry Siegel and Joe Shuster and made his first appearance
in Action Comics #1.
D) Superman’s invulnerability allows him to withstand extreme forces and conditions,
making him an iconic character.

10. While researching a topic, a student has taken the following notes:

• Plant-based meat substitutes are gaining popularity as an alternative to traditional


meat.
• Companies such as Beyond Meat and Impossible Foods have developed plant-based
burger products.
• These plant-based meats utilize proteins from peas, soy, and other plants.
• Environmental benefits of plant-based meat substitutes include reduced greenhouse
gas emissions and lower water usage.

Which choice most e↵ectively highlights a key advantage of plant-based meat substitutes
compared to traditional meat?

A) Beyond Meat and Impossible Foods are well-known companies producing plant-based
burger products.
B) Plant-based meat substitutes utilize proteins from peas, soy, and other plant sources.
C) Plant-based meat substitutes have environmental benefits, such as reduced green-
house gas emissions and lower water usage.
D) The popularity of plant-based meat substitutes has increased as more people search
for alternatives to traditional meat.

180
11. Alice Walker, a prolific author, is best known for her novel, ’The Color Purple.’ The novel
is set in the rural American South and explores the life of African American women during
the early 20th century. Walker’s writing style combines elements of African American
dialect with , giving her work a unique and authentic voice.

Which choice completes the text so that it conforms to the conventions of Standard
English?

A) Standard English’s conventions


B) Standard English conventions
C) conventions of standard English’s
D) Standard English, conventions

12. During the Battle of Hogwarts, Professor McGonagall, head of Gryffindor House, demon-
strated her powerful Transfiguration abilities by animating the stone statues in the cas-
tle to defend Hogwarts. She cast the spell Piertotum Locomotor, which brought the
to life and commanded them to protect the school.

Which choice completes the text so that it conforms to the conventions of Standard
English?

A) statues to life
B) statues life
C) statues’s life
D) statues’ life

181
13. Bill Gates, the co-founder of Microsoft, has made significant contributions to global health
through the Bill and Melinda Gates Foundation. The foundation’s work includes funding
projects that focus on improving access to clean water in developing countries, as well as
to eradicate diseases such as malaria and polio.

Which choice completes the text so that it conforms to the conventions of Standard
English?

A) e↵orts to
B) e↵ort’s to
C) e↵orts too
D) e↵ort to

14. Variable costs are those that vary directly with the level of production or sales. Examples
include raw materials, labor, and shipping expenses. When companies experience signifi-
cant fluctuations in demand, their total costs can become , making it challenging
to plan and manage their resources e↵ectively.

Which choice completes the text with the most logical and precise word or phrase?

A) negligible
B) unstable
C) sustainable
D) systematic

182
15. During the Napoleonic Wars, the British naval blockade of French ports led to extensive
smuggling operations. Due to the nature of their activities, smugglers would
often use small, fast vessels to avoid detection by the British Royal Navy.

Which choice completes the text with the most logical and precise word or phrase?

A) charitable
B) clandestine
C) ceremonial
D) conventional

16. Geothermal energy, a renewable energy source, is derived from the Earth’s internal heat.
This heat is in underground reservoirs, where it can be extracted and converted
into electricity by geothermal power plants.

Which choice completes the text with the most logical and precise word or phrase?

A) depleted
B) hidden
C) stored
D) ignored

183
17. Excise tax, a tax on specific goods or services, is levied by the government to generate
revenue and discourage harmful consumption. While some argue that it unfairly targets
low-income individuals, others believe that it serves public health by decreasing tobacco
and alcohol consumption. Ultimately, excise tax has both economic and social implica-
tions.

Which choice best states the main idea of the text?

A) Excise tax primarily aims to target low-income individuals in society.


B) Excise tax has both economic and social implications, generating revenue while in-
fluencing consumption behavior.
C) Excise tax generates revenue for the government without any social implications.
D) The main purpose of excise tax is to decrease tobacco and alcohol consumption.

18. Forensic engineering is a specialized field within engineering that involves investigating
and analyzing failures in structures, systems, or components. This process often requires
examining materials, products, and structures to determine the cause of a failure and
identify ways to prevent similar failures in the future. Forensic engineers work closely
with other professionals such as architects, safety experts, and insurance investigators
to provide comprehensive insights into the factors contributing to structural or material
failures.

Which choice best states the main idea of the text?

A) Forensic engineering investigates failures in structures, systems, or components and


works to prevent future failures.
B) Forensic engineers work exclusively with insurance companies to investigate failures.
C) The primary goal of forensic engineering is to understand and improve materials.
D) Forensic engineering solely focuses on the analysis of failed structures.

184
19. In quality engineering, organizations must focus on both process improvement and prod-
uct development to achieve the highest standards. Process improvement involves refining
the methods used to create products, while product development focuses on designing
and building high-quality products. By balancing these two approaches, companies can
efficiently deliver products that meet customer expectations and comply with industry
regulations.

Which choice best states the main idea of the text?

A) The importance of process improvement in quality engineering


B) The necessity of product development for achieving high-quality products
C) The role of customer expectations and industry regulations in quality engineering
D) Balancing process improvement and product development in quality engineering

20. During the Napoleonic Wars, French forces under Napoleon Bonaparte faced difficulties
in maintaining supply lines. The British Royal Navy’s dominance of the seas hindered the
French army’s ability to transport goods and resources, ultimately a↵ecting their military
campaigns.

Which finding, if true, would most strongly support the argument that the British Royal
Navy significantly impacted the French army’s campaigns?

A) The French army had a larger number of soldiers compared to the British army.
B) Napoleon Bonaparte often adopted a scorched-earth policy to deny resources to his
enemies.
C) The British Royal Navy was unable to defeat the French fleet in every naval engage-
ment.
D) The French army experienced consistent shortages of ammunition and food during
major battles.

185
21. A recent study found that runners who incorporated interval training into their routine
improved their overall performance more than those who only focused on distance running.
The researchers concluded that the variety in training routines played a significant role
in the observed improvements.

Which finding, if true, would most directly undermine the researcher’s conclusion?

A) A separate study found that athletes who exclusively practiced interval training did
not show significant performance improvements.
B) Top long-distance runners in the world often incorporate interval training into their
routines.
C) Another study found that runners who focused solely on distance running experienced
fewer injuries than those who incorporated interval training.
D) A follow-up study revealed that participants who improved their performance also
made significant changes to their diet and sleep patterns.

22. Sylvia Townsend Warner, a British novelist and poet, was known for her strong political
beliefs. She joined the Communist Party in 1935 and actively participated in the Spanish
Civil War. Her literary works often reflected her political views, making her a prominent
figure in leftist circles.

Which finding, if true, would most strongly support the claim that Sylvia Townsend
Warner’s political beliefs significantly impacted her literary works?

A) A detailed analysis of Warner’s works reveals frequent themes of social justice, class
struggle, and anti-fascism.
B) Sylvia Townsend Warner had a personal library containing numerous books on poli-
tics, history, and social issues.
C) Several of Warner’s friends and acquaintances were also members of the Communist
Party or held leftist political beliefs.
D) Warner’s works were often criticized for their unconventional narrative style and
unique character development.

186
23. Renewable energy sources, such as solar and wind power, have gained significant traction
in recent years. However, these sources are not without their challenges, including in-
termittency and storage issues. Researchers are exploring innovative solutions to address
these obstacles, with the goal of .

Which choice most logically completes the text?

A) convincing skeptics that renewable energy sources are the only viable options for the
future
B) completely replacing traditional energy sources with renewable ones
C) disproving the notion that renewable energy sources have inherent shortcomings
D) achieving a more reliable, sustainable, and efficient energy system

24. Development economists argue that investing in education is crucial for economic growth.
However, they also acknowledge that the quality of education plays a significant role
in this growth. Thus, a mere increase in the number of schools and students may not
necessarily lead to .

Which choice most logically completes the text?

A) significant positive impacts on a country’s economic growth and development


B) a decrease in the overall quality of education provided in the country
C) an increase in the number of highly skilled workers needed for the economy
D) a direct correlation between the amount of investment in education and economic
growth

187
25. GPS technology has revolutionized the way we navigate and track our movements. It
has enabled accurate positioning and improved the efficiency of transportation systems
worldwide. recent advancements in GPS technology have allowed for even more
precise measurements, sometimes within centimeters of the actual location.

Which choice completes the text with the most logical transition?

A) In contrast
B) Nevertheless
C) Moreover
D) On the other hand

26. Zadie Smith, a renowned British author known for her novels and essays, started her
writing career in her twenties. Smith’s first novel, White Teeth, was published when she
was just 24 years old and quickly gained critical acclaim. throughout her career,
Smith has continued to explore themes such as race, identity, and the complexities of
modern life in her works.

Which choice completes the text with the most logical transition?

A) Nevertheless
B) On the other hand
C) Moreover
D) In contrast

188
27. In the field of process engineering, the design of chemical processes is crucial to ensure
efficiency and safety. One common approach to achieve this is through the use of process
flow diagrams, which visually represent the steps in a process and the flow of materials
between them. another important tool in process engineering is the use of
piping and instrumentation diagrams, which provide more detailed information about the
equipment, piping, and control systems involved in a process.

Which choice completes the text with the most logical transition?

A) As a result,
B) On the other hand,
C) Conversely,
D) Similarly,

189
Chapter 25

Module 6 Answers

1. Correct Answer: A
Explanation: A: ’To present the di↵ering perspectives on international trade agreements
and their e↵ects on global economy’ is the correct answer because the passage mentions
both the positive and negative views on international trade agreements and how they
impact the global economy.

2. Correct Answer: D
Explanation: D: ’To highlight Ultimate Frisbee’s unique emphasis on sportsmanship and
self-regulation through the ’Spirit of the Game” is the correct answer because the text
focuses on the ’Spirit of the Game’ as the defining feature of Ultimate Frisbee, emphasizing
its impact on fair play and positive environment.

3. Correct Answer: D
Explanation: D: ’By acknowledging Napoleon’s strategic abilities, but emphasizing that
his ambition and quest for personal glory were detrimental to his long-term success’ is the
correct answer because Text 2 highlights that some historians believe Napoleon’s ambition
and thirst for personal glory were significant factors in his downfall, despite his military
genius.

4. Correct Answer: C
Explanation: C: ’grew’ is the correct answer because it uses the simple past tense verb to
describe the growth of Brazil’s GDP during the specific period of the Brazilian economic
miracle, which occurred in the past.

190
5. Correct Answer: A
Explanation: A: ’has’ is the correct answer because it provides the appropriate present
tense verb to indicate the relationship between the high capability index and a lower
defect rate.

6. Correct Answer: C
Explanation: C: ’undertake’ is the correct answer because it provides the main clause
with a finite present tense verb to indicate the action performed by engineers.

7. Correct Answer: C
Explanation: C: ’joined’ is the correct answer because it provides the finite past tense
verb needed to properly express the action of Chile becoming a member of the Andean
Community.

8. Correct Answer: D
Explanation: D: ’Murray Gell-Mann, who introduced the concept of quarks and won the
1969 Nobel Prize in Physics, made groundbreaking contributions to our understanding of
matter.’ is the correct answer because it e↵ectively combines the notes about Gell-Mann’s
Nobel Prize and the concept of quarks to emphasize his most notable achievement.

9. Correct Answer: A
Explanation: A: ’Superman represents the ultimate good by defending humanity and
upholding justice with his incredible abilities, such as invulnerability.’ is the correct
answer because it e↵ectively uses information from the notes to explain how Superman
embodies the ultimate good.

10. Correct Answer: C


Explanation: C: ’Plant-based meat substitutes have environmental benefits, such as re-
duced greenhouse gas emissions and lower water usage.’ is the correct answer because it
highlights a key advantage of plant-based meat substitutes compared to traditional meat.

11. Correct Answer: B


Explanation: B: ’Standard English conventions’ is the correct answer because it properly
conveys that the writing style combines elements of African American dialect with the
conventions of Standard English.

191
12. Correct Answer: A
Explanation: A: ’statues to life’ is the correct answer because it properly utilizes the
preposition ’to’ in order to indicate the transition of the statues from being inanimate to
animate.

13. Correct Answer: A


Explanation: A: ’e↵orts to’ is the correct answer because it uses the plural form ’e↵orts’
to match the plural subject of the sentence, and it correctly uses ’to’ to introduce the
infinitive verb ’eradicate’.

14. Correct Answer: B


Explanation: B: ’unstable’ is the correct answer because it accurately describes a situation
where costs are difficult to predict due to fluctuations in demand.

15. Correct Answer: B


Explanation: B: ’Clandestine’ is the correct answer because it accurately describes the
secretive and illicit nature of the smuggling operations during the Napoleonic Wars.

16. Correct Answer: C


Explanation: C: ’stored’ is the correct answer because it accurately describes how the
Earth’s internal heat is contained in underground reservoirs, making it available for ex-
traction and conversion into electricity.

17. Correct Answer: B


Explanation: B: ’Excise tax has both economic and social implications, generating revenue
while influencing consumption behavior.’ is the correct answer because it covers the main
points mentioned in the passage: revenue generation and the impact on consumption
habits.

18. Correct Answer: A


Explanation: A: ’Forensic engineering investigates failures in structures, systems, or com-
ponents and works to prevent future failures.’ is the correct answer because it accurately
summarizes the main idea of the passage, which discusses the role of forensic engineering
in analyzing failures and finding ways to prevent them.

192
19. Correct Answer: D
Explanation: D: ’Balancing process improvement and product development in quality
engineering’ is the correct answer because the passage discusses the significance of both
process improvement and product development in achieving high standards in quality
engineering.

20. Correct Answer: D


Explanation: D: ’The French army experienced consistent shortages of ammunition and
food during major battles.’ is the correct answer because it directly demonstrates the
impact of the British Royal Navy’s dominance on the French army’s ability to maintain
supply lines and conduct successful military campaigns.

21. Correct Answer: D


Explanation: D: ’A follow-up study revealed that participants who improved their per-
formance also made significant changes to their diet and sleep patterns.’ is the correct
answer because it introduces another factor that could be responsible for the observed
performance improvements, thus undermining the researcher’s conclusion that the variety
in training routines played a significant role in the improvements.

22. Correct Answer: A


Explanation: A: ’A detailed analysis of Warner’s works reveals frequent themes of so-
cial justice, class struggle, and anti-fascism.’ is the correct answer because it directly
demonstrates that her political beliefs were significantly reflected in her literary works,
supporting the claim.

23. Correct Answer: D


Explanation: D: ’achieving a more reliable, sustainable, and efficient energy system’ is
the correct answer because it aligns with the researchers’ e↵orts to address the challenges
of renewable energy sources.

24. Correct Answer: A


Explanation: A: ’significant positive impacts on a country’s economic growth and develop-
ment’ is the correct answer because the passage acknowledges that the quality of education
is important for economic growth, not just the quantity of schools and students.

193
25. Correct Answer: C
Explanation: C: ’Moreover’ is the correct answer because it logically signals that the
sentence adds to the information about the benefits of GPS technology by discussing
recent advancements that allow for even more precise measurements.

26. Correct Answer: C


Explanation: C: ’Moreover’ is the correct answer because it indicates that the information
about Smith exploring various themes in her works is an additional point that supports
her successful writing career.

27. Correct Answer: D


Explanation: D: ’Similarly’ is the correct answer because it shows that both process flow
diagrams and piping and instrumentation diagrams are tools used in process engineering
for designing chemical processes.

194
Chapter 26

Module 7

Reading and Writing - 27 Questions

DIRECTIONS: The questions in this section are designed to test your reading and writing
abilities. Each question will provide you with one or more passages. It is important to
thoroughly read each passage and question before selecting the best answer based on the
information provided. All of the questions in this section are multiple-choice and will have
four options to choose from. There is only one correct answer for each question.

1. Capital gains tax is a levy imposed on the profit made from selling an asset, including
stocks, bonds, or real estate. It is designed to encourage long-term investments by tax-
ing short-term gains at a higher rate. This tax structure incentivizes investors to hold
onto their assets for more extended periods, contributing to overall economic stability.
However, critics argue that capital gains tax can hinder economic growth by discouraging
investment and reducing liquidity in the market.

Which choice best states the main purpose of the text?

A) To explain the various assets that can be subjected to capital gains tax, such as
stocks, bonds, and real estate
B) To argue in favor of a higher capital gains tax rate for short-term investments as a
means of promoting economic stability
C) To advocate for the abolishment of capital gains tax to encourage investments and
increase market liquidity
D) To provide an overview of capital gains tax and its intended e↵ects on investment
behavior and the economy

195
2. Synthetic biology engineering combines principles from engineering, biology, and com-
puter science to design and construct biological systems. These systems can have various
applications, such as producing biofuels or developing new medical treatments. One of
the key techniques in synthetic biology is the use of standardized genetic parts, known
as BioBricks, which can be easily assembled to create complex biological systems. This
approach allows researchers to rapidly prototype and test new ideas, similar to assembling
electronic circuits.

Which choice best states the main purpose of the text?

A) To provide a comprehensive history of synthetic biology engineering, including its


origins and major breakthroughs
B) To argue that synthetic biology engineering should prioritize the development of
biofuels over medical treatments
C) To explore the ethical implications of synthetic biology engineering and the potential
consequences of manipulating genetic material
D) To introduce synthetic biology engineering and emphasize its interdisciplinary nature
and the significance of standardized genetic parts

3. Text 1: Bionic technology has made significant advancements in recent years. These
advancements have led to the development of prosthetic limbs that can be controlled by
the user’s thoughts, providing a much more natural movement.

Text 2: In a recent study, researchers found that bionic limbs can sometimes cause phan-
tom limb pain in users. This is because the brain may struggle to adapt to the new limb
and can send mixed signals, resulting in pain or discomfort.

Based on the texts, how would the researchers in Text 2 most likely respond to the
advancements in bionic technology discussed in Text 1?

A) By acknowledging the advancements in bionic technology, but also pointing out the
potential issue of phantom limb pain
B) By suggesting that the advancements in bionic technology should focus more on
aesthetics rather than functionality
C) By asserting that the advancements in bionic technology are not significant enough
to warrant further research
D) By recommending that bionic technology advancements should prioritize the devel-
opment of other types of prosthetics, such as sensory feedback devices

196
4. In paper engineering, the process of folding a flat sheet of paper into a three-dimensional
object is called origami. During the folding process, the paper’s structural integrity is
maintained by the , which support the object’s final shape.

Which choice completes the text so that it conforms to the conventions of Standard
English?

A) creasing
B) crease’s
C) creases’
D) creases

5. Sylvia Plath, a renowned American poet and novelist, struggled with depression through-
out her life. In her semi-autobiographical novel, The Bell Jar, the protagonist, Esther
Greenwood, a similar emotional turmoil, reflecting Plath’s personal experiences.

Which choice completes the text so that it conforms to the conventions of Standard
English?

A) will undergo
B) undergone
C) undergoes
D) had undergone

197
6. Thomas Hardy’s novels often explore the plight of individuals caught in the constraints
of social expectations. In Tess of the d’Urbervilles, for instance, Tess’s life is tragically
shaped by the double standards of her society, which her limited options and
ultimately leads to her downfall.

Which choice completes the text so that it conforms to the conventions of Standard
English?

A) to restrict
B) restricting
C) restricted
D) restrict

7. During the American Revolutionary War, the Continental Army faced numerous chal-
lenges, such as limited resources and inexperienced soldiers. However, despite these set-
backs, the army under the leadership of General George Washington, ultimately
securing independence for the United States.

Which choice completes the text so that it conforms to the conventions of Standard
English?

A) persevered
B) persevered
C) persevering
D) to persevere

198
8. While researching a topic, a student has taken the following notes:

• Diseconomies of scale occur when a company grows too large and experiences dimin-
ishing returns.
• As a firm expands, its per-unit costs may increase due to coordination and commu-
nication challenges.
• The optimal firm size depends on the balance between economies and diseconomies
of scale.
• In some industries, such as software development, diseconomies of scale can appear
more quickly than in others.

Which choice most e↵ectively uses relevant information from the notes to explain the
concept of diseconomies of scale?

A) Diseconomies of scale occur when a firm’s growth leads to increased per-unit costs
due to coordination and communication difficulties.
B) Diseconomies of scale happen when a company grows too large and faces communi-
cation challenges.
C) The optimal firm size depends on the balance between economies and diseconomies
of scale in various industries.
D) In some industries, diseconomies of scale appear more quickly than in others, such
as software development.

9. While researching a topic, a student has taken the following notes:

• The Ecuadorian pasillo is a genre of music originating from Ecuador.


• It is characterized by its waltz-like rhythm and emotional lyrical content.
• The pasillo evolved from European and Indigenous musical influences.
• Notable Ecuadorian pasillo composers include Julio Jaramillo and Francisco Paredes
Herrera.

Which choice most e↵ectively uses relevant information from the notes to explain the
distinct characteristics of the Ecuadorian pasillo?

A) The Ecuadorian pasillo originated from Ecuador and is a form of waltz.


B) The Ecuadorian pasillo is a unique music genre with waltz-like rhythm and emotional
lyrics, influenced by European and Indigenous elements.
C) Julio Jaramillo and Francisco Paredes Herrera are famous for their contributions to
the Ecuadorian pasillo.

199
D) European and Indigenous musical influences shaped the Ecuadorian pasillo, which is
popular in Ecuador.

10. While researching a topic, a student has taken the following notes:

• Nelson Mandela was the first black President of South Africa.


• Mandela spent 27 years in prison for his anti-apartheid activism.
• Mandela’s presidency focused on reconciliation between the country’s racial groups.
• The Truth and Reconciliation Commission was established during Mandela’s term.

Which choice most e↵ectively uses relevant information from the notes to highlight Man-
dela’s approach to addressing racial issues during his presidency?

A) Nelson Mandela was the first black President of South Africa and was imprisoned for
27 years.
B) Mandela focused on reconciliation between racial groups and established the Truth
and Reconciliation Commission.
C) During Mandela’s presidency, the Truth and Reconciliation Commission was estab-
lished.
D) Nelson Mandela was known for his anti-apartheid activism before becoming South
Africa’s president.

200
11. Joseph Conrad, a Polish-British writer, was born in 1857 in present-day Ukraine. Often
classified as a modernist, Conrad is best known for his novels, such as Heart of Darkness
and Lord Jim. His writing style is noted for its intricate, multi-layered narratives and
.

Which choice completes the text so that it conforms to the conventions of Standard
English?

A) its psychological depth


B) its psychological depths
C) its psychological depths’
D) its psychological depth’s

12. Geomatics Engineering involves the use of specialized equipment to collect, analyze, and
interpret spatial data. One such device is the total station, which combines the capabilities
of a theodolite and an electronic distance meter to measure both angles and distances with
high accuracy. These measurements can then be used to calculate the of various
points on a site.

Which choice completes the text so that it conforms to the conventions of Standard
English?

A) coordinates’
B) coordinate’s
C) coordinate
D) coordinates

201
13. During the 19th century, thousands of pioneers embarked on the Oregon Trail in search of
a better life. The journey was treacherous, with the travelers facing numerous challenges
such as crossing rivers, enduring extreme weather conditions, and supplies of
food and water.

Which choice completes the text so that it conforms to the conventions of Standard
English?

A) managing scarce
B) scarce, managing
C) managing, scarce
D) scarce managing

14. Andrew Wiles, a British mathematician, ultimately solved Fermat’s Last Theorem in
1994, nearly 358 years after it was first proposed. His groundbreaking proof, which
involved an unexpected link between elliptic curves and modular forms, was initially
by the mathematical community.

Which choice completes the text with the most logical and precise word or phrase?

A) disregarded
B) embraced
C) astonished
D) repelled

202
15. 3D printing has revolutionized the manufacturing industry by allowing for the rapid pro-
duction of complex objects. However, the technology has also led to increased concerns
about intellectual property theft, as it makes it easier to replicate and distribute copy-
righted designs without the creator’s .

Which choice completes the text with the most logical and precise word or phrase?

A) consent
B) intuition
C) presence
D) knowledge

16. Fiscal transparency is crucial for public sector accountability, as it enables citizens and
markets to assess the government’s financial decisions. However, in some countries, the
lack of in public financial management can lead to misallocations of resources
and corruption.

Which choice completes the text with the most logical and precise word or phrase?

A) uniformity
B) efficiency
C) transparency
D) creativity

203
17. Water skiing is a thrilling sport that not only requires physical strength but also mental
agility. A water skier must maintain balance and control while being pulled by a boat
at high speeds. The skier’s body position, rope handling, and choice of equipment play
crucial roles in their performance. Experienced water skiers often develop their techniques
and preferences, making them better prepared to face the challenges of the sport.

Which choice best states the main idea of the text?

A) The text emphasizes the significance of the boat’s speed in determining the success
of a water skier.
B) The text highlights the importance of both physical and mental aspects in water
skiing and how experience improves performance.
C) The text elaborates on the process of choosing the right equipment for an e↵ective
water skiing experience.
D) The text discusses the negative consequences of not maintaining proper balance and
control during water skiing.

18. Advancements in artificial intelligence have led to the development of highly sophisticated
systems that can perform complex tasks. While these AI-powered machines have the
potential to revolutionize various industries, the ethical implications of such advancements
are often debated. Some argue that the rapid growth of AI might eventually lead to job
displacement and increased reliance on machines, while others believe that AI can enhance
human capabilities and create new opportunities.

Which choice best states the main idea of the text?

A) The text discusses the advancements in AI and the debate surrounding its potential
impact.
B) The ethical implications of AI are universally agreed upon and well-understood.
C) Artificial intelligence is solely responsible for the loss of jobs in the modern world.
D) AI is a detrimental force that will lead to the downfall of human society.

204
19. Hedy Lamarr, a Hollywood actress in the 1940s, was also a brilliant inventor. During
World War II, she co-developed a frequency-hopping communication system to prevent
enemies from jamming radio signals. Although her invention was not used in the war, it
later became the foundation for modern technologies such as Wi-Fi, Bluetooth, and GPS.
Hedy’s intelligence and innovative spirit were often overshadowed by her beauty, but her
contributions have left a lasting impact on the world.

Which choice best states the main idea of the text?

A) Hedy Lamarr’s beauty led to her success as an inventor and actress.


B) Hedy Lamarr’s inventions were widely used during World War II.
C) Hedy Lamarr was a talented inventor whose work laid the groundwork for modern
communication technologies.
D) Hedy Lamarr’s career as an actress was negatively a↵ected by her interest in inven-
tions.

20. The Lion King musical, directed by Julie Taymor, has been praised for its innovative
use of puppetry and costumes. Taymor’s creative direction is said to have contributed
significantly to the show’s success and longevity, making it one of the longest-running
Broadway shows in history.

Which finding, if true, would most strongly support the claim that Julie Taymor’s direc-
tion played a significant role in the success of The Lion King musical?

A) The show’s ticket sales and positive reviews increased dramatically after Taymor’s
involvement was announced.
B) Many other Broadway shows have also used puppetry and costumes in innovative
ways.
C) The Lion King musical has faced criticism for not staying true to the original Disney
film.
D) Some other long-running Broadway shows have had similar creative direction.

205
21. Snowboarding’s popularity has surged since the 1980s. Recently, studies have suggested
that the increase in the number of snowboarders has led to a decline in ski resort profits. A
researcher argues that snowboarders’ spending habits may be contributing to this trend.

Which finding, if true, would most directly support the researcher’s claim?

A) Ski resorts that exclusively cater to skiers report higher profits than resorts that cater
to both skiers and snowboarders.
B) Skiers tend to purchase more expensive snowboarding equipment than snowboarders.
C) Snowboarders tend to spend less on accommodations and amenities at ski resorts
compared to skiers.
D) The number of skiers has remained relatively stable despite the increase in the number
of snowboarders.

22. George Orwell’s novel, 1984, portrays a dystopian society in which the government sup-
presses individualism and free thinking. Critics suggest Orwell wrote the novel as a
cautionary tale about the potential consequences of totalitarianism.

Which finding, if true, would most directly support the claim that Orwell intended 1984
as a cautionary tale about totalitarianism?

A) Many readers consider 1984 to be a compelling and thought-provoking work of fiction.


B) 1984 has frequently been compared to other dystopian novels, such as Aldous Huxley’s
Brave New World.
C) Orwell’s novel has been adapted into several films, television shows, and stage pro-
ductions.
D) Orwell had previously expressed concerns about the dangers of totalitarian regimes
in his essays and letters.

206
23. Confucius, a renowned Chinese philosopher, focused on personal and governmental moral-
ity, proper social relationships, and justice. His teachings have greatly influenced various
aspects of Chinese society, and his ideas have been passed down through generations,
leaving a lasting impact on .

Which choice most logically completes the text?

A) the scientific revolution and the advancements in technology in the western world
B) the development of modern democracy and the promotion of individual rights
C) East Asian culture and the philosophical foundations of many regional governments
D) the establishment of economic systems and trade regulations across the globe

24. Ralph Ellison, an influential African American writer, is best known for his novel In-
visible Man. Ellison’s work often explores themes of identity and invisibility, reflecting
the experiences of Black Americans in a racially divided society. By doing so, Ellison
.

Which choice most logically completes the text?

A) criticizes the African American community for not being more active in fighting for
their rights.
B) argues that invisibility is the only way for Black Americans to survive in a racially
divided society.
C) suggests that identity and invisibility are innate characteristics of all individuals,
regardless of race.
D) e↵ectively raises awareness about the struggles and complexities faced by Black Amer-
icans.

207
25. The Persian Gulf War, which began in 1990, was a conflict between Iraq, led by President
Saddam Hussein, and a coalition of 35 countries led by the United States. The war
was triggered by Iraq’s invasion of Kuwait, which was condemned by the international
community. the coalition forces launched a military o↵ensive, Operation Desert
Storm, to expel Iraqi forces from Kuwait.

Which choice completes the text with the most logical transition?

A) In response,
B) Similarly,
C) However,
D) For example,

26. Many corporations have been known to use loopholes in the tax system to reduce their
overall tax burden. For example, some companies may shift their profits to countries
with lower tax rates. a study conducted by Harvard Business School researchers
found that 15% of US corporations moved their profits to tax havens between 2004 and
2012.

Which choice completes the text with the most logical transition?

A) In contrast
B) On the other hand
C) Alternatively
D) In fact

208
27. Stem cell research has the potential to revolutionize medicine by providing therapies for
a wide range of diseases and conditions. Scientists have been able to create specialized
cells, such as nerve and muscle cells, from stem cells. this breakthrough en-
ables researchers to study diseases in the lab more accurately and develop more targeted
treatments.

Which choice completes the text with the most logical transition?

A) In contrast
B) Nevertheless
C) Consequently
D) On the other hand

209
Chapter 27

Module 7 Answers

1. Correct Answer: D
Explanation: D: ’To provide an overview of capital gains tax and its intended e↵ects on
investment behavior and the economy’ is the correct answer because the passage explains
what capital gains tax is and its purpose in influencing investment behavior and the
economy.

2. Correct Answer: D
Explanation: D: ’To introduce synthetic biology engineering and emphasize its interdis-
ciplinary nature and the significance of standardized genetic parts’ is the correct answer
because the passage mainly focuses on providing an overview of synthetic biology engi-
neering and highlights the importance of BioBricks in the field.

3. Correct Answer: A
Explanation: A: ’By acknowledging the advancements in bionic technology, but also point-
ing out the potential issue of phantom limb pain’ is the correct answer because Text 2
discusses a recent study which found that phantom limb pain can be caused by the brain’s
struggle to adapt to bionic limbs.

4. Correct Answer: D
Explanation: D: ’creases’ is the correct answer because it provides the plural form of the
noun ’crease,’ which appropriately describes the multiple folds that support the object’s
final shape.

5. Correct Answer: C
Explanation: C: ’undergoes’ is the correct answer because it maintains the consistency of
the present tense used throughout the passage to describe the events in the novel.

210
6. Correct Answer: D
Explanation: D: ’restrict’ is the correct answer because it supplies the main clause with
the finite present tense verb that is consistent with the other present tense verbs used to
describe the events in Hardy’s novels. Furthermore, it’s conventional to use the present
tense when discussing a literary work.

7. Correct Answer: B
Explanation: B: ’persevered’ is the correct answer because it provides the finite past tense
verb to indicate the action of the Continental Army during the American Revolutionary
War under General George Washington’s leadership.

8. Correct Answer: A
Explanation: A: ’Diseconomies of scale occur when a firm’s growth leads to increased
per-unit costs due to coordination and communication difficulties.’ is the correct answer
because it e↵ectively uses relevant information from the notes to explain the concept of
diseconomies of scale.

9. Correct Answer: B
Explanation: B: ’The Ecuadorian pasillo is a unique music genre with waltz-like rhythm
and emotional lyrics, influenced by European and Indigenous elements.’ is the correct
answer because it e↵ectively summarizes the distinct characteristics of the Ecuadorian
pasillo mentioned in the notes.

10. Correct Answer: B


Explanation: B: ’Mandela focused on reconciliation between racial groups and estab-
lished the Truth and Reconciliation Commission.’ is the correct answer because it ef-
fectively highlights Mandela’s approach to addressing racial issues during his presidency
by mentioning both his focus on reconciliation and the establishment of the Truth and
Reconciliation Commission.

11. Correct Answer: B


Explanation: B: ’its psychological depths’ is the correct answer because the passage refers
to the multiple layers of psychological complexity in Conrad’s work, requiring the use of
the plural form ’depths’ without the need for possessive forms.

211
12. Correct Answer: D
Explanation: D: ’coordinates’ is the correct answer because it is the plural form of the
noun, indicating that multiple points and their respective positions are being calculated
using the collected data.

13. Correct Answer: C


Explanation: C: ’managing, scarce’ is the correct answer because it properly uses a comma
to separate the two adjectives and maintain the flow of the sentence.

14. Correct Answer: C


Explanation: C: ’astonished’ is the correct answer because it accurately describes the
reaction of the mathematical community to the unexpected link found by Wiles in his
proof.

15. Correct Answer: A


Explanation: A: ’consent’ is the correct answer because it accurately describes the lack
of permission from the creator when copyrighted designs are replicated and distributed
using 3D printing technology.

16. Correct Answer: C


Explanation: C: ’transparency’ is the correct answer because the passage discusses the
importance of fiscal transparency for public sector accountability, and the lack of it can
lead to negative consequences.

17. Correct Answer: B


Explanation: B: ’The text highlights the importance of both physical and mental aspects
in water skiing and how experience improves performance.’ is the correct answer because
the passage emphasizes the need for balance, control, and experience in water skiing.

18. Correct Answer: A


Explanation: A: ’The text discusses the advancements in AI and the debate surrounding
its potential impact.’ is the correct answer because it accurately captures the main idea
of the passage, which presents both the potential benefits and concerns related to AI
advancements.

212
19. Correct Answer: C
Explanation: C: ’Hedy Lamarr was a talented inventor whose work laid the groundwork
for modern communication technologies’ is the correct answer because the passage focuses
on her inventive contributions and their impact on current technology.

20. Correct Answer: A


Explanation: A: ’The show’s ticket sales and positive reviews increased dramatically after
Taymor’s involvement was announced.’ is the correct answer because it demonstrates a
direct correlation between Taymor’s involvement and the success of the musical.

21. Correct Answer: C


Explanation: C: ’Snowboarders tend to spend less on accommodations and amenities at
ski resorts compared to skiers.’ is the correct answer because this finding directly supports
the researcher’s claim that snowboarders’ spending habits are contributing to the decline
in ski resort profits.

22. Correct Answer: D


Explanation: D: ’Orwell had previously expressed concerns about the dangers of totali-
tarian regimes in his essays and letters.’ is the correct answer because it directly points
to the author’s intentions and beliefs on the topic, suggesting that 1984 was written with
the purpose of warning readers about the potential consequences of totalitarianism.

23. Correct Answer: C


Explanation: C: ’East Asian culture and the philosophical foundations of many regional
governments’ is the correct answer because it logically follows the focus of Confucius’
teachings and their influence on Chinese society.

24. Correct Answer: D


Explanation: D: ’e↵ectively raises awareness about the struggles and complexities faced
by Black Americans.’ is the correct answer because it follows logically from the discussion
of Ellison exploring themes of identity and invisibility in a racially divided society.

25. Correct Answer: A


Explanation: A: ’In response’ is the correct answer because it shows the causal relationship
between Iraq’s invasion of Kuwait and the coalition forces’ actions.

213
26. Correct Answer: D
Explanation: D: ’In fact’ is the correct answer because it adds emphasis to the provided
example, indicating that the study by Harvard Business School researchers supports the
claim made in the first sentence about corporations using tax loopholes.

27. Correct Answer: C


Explanation: C: ’Consequently’ is the correct answer because it logically signals that the
development of specialized cells from stem cells leads to the ability to study diseases more
accurately and develop targeted treatments.

214
Chapter 28

Module 8

Reading and Writing - 27 Questions

DIRECTIONS: The questions in this section are designed to test your reading and writing
abilities. Each question will provide you with one or more passages. It is important to
thoroughly read each passage and question before selecting the best answer based on the
information provided. All of the questions in this section are multiple-choice and will have
four options to choose from. There is only one correct answer for each question.

1. In 1997, biotechnologist Dr. Jane Collins developed a groundbreaking technique for edit-
ing plant genomes. By precisely targeting specific genes, Collins’s method enabled sci-
entists to modify crop traits such as pest resistance, drought tolerance, and nutritional
content. This innovation revolutionized agriculture, leading to increased food production
and reduced environmental impact. Today, her technique continues to shape the future
of sustainable farming practices.

Which choice best states the main purpose of the text?

A) To discuss the ethical implications of gene editing in plants and their potential con-
sequences
B) To highlight Dr. Jane Collins’s significant contribution to the field of biotechnology
and its impact on agriculture
C) To argue that other biotechnological advancements are overshadowing Dr. Collins’s
work
D) To describe the process of editing plant genomes in detail and its various applications

215
2. In the 1985 science fiction film ’Back to the Future,’ teenager Marty McFly accidentally
travels back in time to the year 1955 using a time machine created by eccentric scientist
Doc Brown. Marty’s presence in the past jeopardizes his own existence as he interferes
with his parents’ meeting, and he must ensure they fall in love to secure his future.
Throughout the film, Marty and Doc work together to restore the timeline and find a way
for Marty to return to 1985.

Which choice best states the main purpose of the text?

A) To summarize the central plot and conflicts of the film ’Back to the Future’
B) To analyze the scientific accuracy of the time travel concepts in ’Back to the Future’
C) To compare the cultural di↵erences between 1955 and 1985 as portrayed in ’Back to
the Future’
D) To discuss the impact of ’Back to the Future’ on the careers of its lead actors

3. Text 1: Motocross racer Alice Jamison is renowned for her unconventional racing style,
which allows her to gain an edge over opponents. Many racers strictly adhere to traditional
techniques, but Jamison’s innovative approach has opened up possibilities for new racing
strategies.

Text 2: In a recent interview, motocross coach and former racer Tom Harrison discussed
the evolution of racing strategies. Harrison emphasized the importance of adapting to
changing conditions and exploring alternative techniques, arguing that racers who stick
to conventional methods may struggle to keep up with the sport’s advancements.

Based on the texts, how would Tom Harrison (Text 2) most likely respond to the racing
style of Alice Jamison (Text 1)?

A) By asserting that her unconventional style is detrimental to her overall performance


B) By appreciating her willingness to explore new strategies and adapt to the sport’s
evolution
C) By questioning her ability to maintain consistent results with an innovative approach
D) By expressing concerns about the potential risks associated with deviating from
traditional techniques

216
4. Before becoming the 44th President of the United States, Barack Obama worked as a
community organizer, a civil rights attorney, and a law professor. In 1996, he to
the Illinois State Senate, where he served for eight years.

Which choice completes the text so that it conforms to the conventions of Standard
English?

A) was elected
B) being elected
C) to be elected
D) electing

5. Remote sensing is a valuable tool for monitoring the Earth’s surface. Using satellite im-
agery, researchers can gather data on various environmental factors such as temperature,
vegetation, and precipitation. By analyzing this data, scientists can models to
predict climate change and assess the impact of human activities on the environment.

Which choice completes the text so that it conforms to the conventions of Standard
English?

A) utilized
B) having utilized
C) utilizing
D) utilize

217
6. Marie Curie, a physicist and chemist, conducted groundbreaking research on radioactivity.
She the first woman to win a Nobel Prize and remains the only person to have
won Nobel Prizes in two di↵erent scientific fields: physics and chemistry.

Which choice completes the text so that it conforms to the conventions of Standard
English?

A) became
B) becoming
C) to become
D) would become

7. In the 1980s, the Boston Celtics and the Los Angeles Lakers were two of the most dominant
teams in the NBA. Their intense rivalry to a series of memorable championship
clashes, featuring legendary players like Larry Bird and Magic Johnson.

Which choice completes the text so that it conforms to the conventions of Standard
English?

A) to lead
B) leading
C) led
D) leads

218
8. While researching a topic, a student has taken the following notes:

• Alan Turing was a British mathematician and computer scientist.


• Turing developed the concept of the Turing machine, a theoretical device that laid
the foundation for modern computers.
• During World War II, Turing played a crucial role in breaking the German Enigma
code.
• Turing’s work in artificial intelligence led to the creation of the Turing Test, which
evaluates a machine’s ability to exhibit intelligent behavior.

Which choice most e↵ectively uses relevant information from the notes to emphasize
Turing’s contributions to the field of computer science?

A) Alan Turing, a pioneer in computer science, developed the Turing machine concept
and laid the groundwork for artificial intelligence with the Turing Test.
B) Turing was a British scientist whose work in artificial intelligence led to the Turing
Test.
C) Alan Turing, a British mathematician, developed the theoretical concept of the Turing
machine.
D) Alan Turing was a mathematician who helped break the Enigma code during World
War II.

9. While researching a topic, a student has taken the following notes:

• Lacrosse is a fast-paced sport with Native American origins.


• The game is played using a small rubber ball and a long-handled stick called a crosse.
• There are two main versions of lacrosse: field lacrosse and box lacrosse.
• In both versions, the objective is to score goals by shooting the ball into the opposing
team’s net.

Which choice most e↵ectively uses relevant information from the notes to explain the
main objective in Lacrosse?

A) Lacrosse is a fast-paced sport with an objective to run faster than the opposing team.
B) The primary objective in both field and box lacrosse is to score goals by propelling
the ball into the opposing team’s net using a crosse.
C) The objective of lacrosse is to have the most players on the field at the end of the
game.

219
D) The goal of lacrosse is to pass the ball between players using their crosses without
dropping the ball.

10. While researching a topic, a student has taken the following notes:

• Perfect competition is a market structure with many buyers and sellers.


• In perfect competition, firms produce homogeneous products.
• Barriers to entry and exit are minimal in perfect competition.
• Firms in perfect competition are price takers.

Which choice most e↵ectively uses relevant information from the notes to describe the
characteristics of perfect competition?

A) Perfect competition is a market structure with many buyers and sellers, homogeneous
products, and minimal barriers to entry and exit, making firms price takers.
B) Perfect competition involves many buyers and sellers, but the products are not ho-
mogeneous.
C) In perfect competition, firms produce homogeneous products, but there are significant
barriers to entry and exit.
D) Firms in perfect competition are price takers, but the market structure does not
involve many buyers and sellers.

220
11. The Good Friday Agreement, signed in 1998, brought an end to decades of conflict in
Northern Ireland. It created power-sharing arrangements between political parties and
established a between the governments of Ireland and the United Kingdom,
fostering cooperation and dialogue.

Which choice completes the text so that it conforms to the conventions of Standard
English?

A) relation’s
B) relations’
C) relationships’
D) relationship

12. During their time at the university, best friends Jessica and Rebecca became inseparable.
They shared not only their love for studying literature, but also their passion for hiking
and exploring nature. After graduation, they decided to embark on a six-month journey,
where they would visit various national parks and hike through the trails.

Which choice completes the text so that it conforms to the conventions of Standard
English?

A) countrys’
B) country’s
C) countries
D) countries’

221
13. Alexander Graham Bell, the inventor of the telephone, also made significant contributions
to the field of aviation. Bell’s interest in flight led him to establish the Aerial Experiment
Association (AEA) in 1907, which aimed to develop heavier-than-air flying machines.
The AEA created several successful aircraft, with Bell playing a crucial role in their
and engineering.

Which choice completes the text so that it conforms to the conventions of Standard
English?

A) designed
B) designs,
C) design’s
D) design

14. Cleopatra was a skilled diplomat, a brilliant military strategist, and the last Pharaoh of
Ancient Egypt. Her reign was marked by a series of tumultuous events, including wars,
assassinations, and political intrigue. Despite her many accomplishments, Cleopatra’s life
ended in tragedy when she was captured and forced to commit by her Roman
captors.

Which choice completes the text with the most logical and precise word or phrase?

A) celebration
B) resignation
C) espionage
D) suicide

222
15. Hedy Lamarr, an actress and inventor, co-developed a frequency-hopping technology
during World War II. This invention later became the foundation for modern spread-
spectrum communication technologies, such as Bluetooth and Wi-Fi. Lamarr’s work was
for years, until her contributions were finally recognized in the 1990s.

Which choice completes the text with the most logical and precise word or phrase?

A) publicized
B) unrivaled
C) overlooked
D) copied

16. The Uruguayan milonga, a precursor to the tango, is a lively dance characterized by its
syncopated rhythms and intricate footwork. It has its roots in African musical traditions
and was developed in the 19th century by the Afro-Uruguayan community. Over time,
the milonga has evolved into a social event, attracting dancers and musicians
from all walks of life.

Which choice completes the text with the most logical and precise word or phrase?

A) vibrant
B) secluded
C) monotonous
D) predictable

223
17. In the world of powerlifting, three primary exercises are vital for success: the squat, the
bench press, and the deadlift. These exercises involve compound movements that engage
multiple muscle groups, providing a comprehensive strength-building workout. Athletes
often dedicate specific days to each of these exercises, ensuring they have ample time to
perfect their form and build the necessary strength. A well-rounded powerlifting program
incorporates these exercises, along with accessory movements, to maximize the athlete’s
potential.

Which choice best states the main idea of the text?

A) The three primary exercises in powerlifting and their role in a well-rounded program
B) How powerlifting exercises focus on isolating individual muscle groups
C) The importance of perfecting form to prevent injury in powerlifting
D) The benefits of incorporating accessory movements in a powerlifting routine

18. During the French Revolution, the radical Jacobin party, led by Robespierre, sought to
create a new society based on reason and equality. In their pursuit of these ideals, they
instituted the Reign of Terror, executing thousands of perceived enemies. The revolution
ultimately led to the rise of Napoleon Bonaparte, a military leader who seized power and
declared himself Emperor.

Which choice best states the main idea of the text?

A) The French Revolution, driven by radical ideals, led to the Reign of Terror and the
rise of Napoleon.
B) Napoleon Bonaparte’s rise to power was unrelated to the French Revolution.
C) The Reign of Terror was a necessary step for the French Revolution to succeed.
D) Robespierre’s leadership during the French Revolution prevented the establishment
of a democratic government.

224
19. During the Russian Revolution, the power struggle between the Bolsheviks and the Pro-
visional Government intensified. The Bolsheviks, led by Vladimir Lenin, advocated for a
socialist government, while the Provisional Government aimed to restore a constitutional
monarchy. As the revolution unfolded, the Bolsheviks gained popular support, ultimately
leading to their victory and the establishment of the Soviet Union.

Which choice best states the main idea of the text?

A) The Bolsheviks’ rise to power was primarily due to their charismatic leader, Vladimir
Lenin.
B) The Provisional Government’s attempt to restore the monarchy was the main cause
of the Russian Revolution.
C) The Russian Revolution was a minor conflict between various factions, with little
lasting impact on Russia’s political landscape.
D) The power struggle between the Bolsheviks and Provisional Government during the
Russian Revolution resulted in the Bolsheviks’ victory and the Soviet Union’s estab-
lishment.

20. Paper engineers have developed a new technique to enhance the strength of paper products
by adding a specialized polymer. This method not only improves the durability of paper
items but also reduces environmental impact, as the modified paper can be recycled
without compromising quality.

Which finding, if true, would most directly support the e↵ectiveness of the new technique?

A) Some paper engineers have expressed concerns about the long-term viability of the
polymer used in the new technique.
B) Products made with the new technique show a 50% increase in strength and a higher
recycling rate compared to conventional paper products.
C) The new technique has been adopted by a few companies as an alternative to tradi-
tional paper manufacturing processes.
D) A di↵erent technique involving nanocellulose fibers has also been proposed to increase
the strength of paper products.

225
21. In the aftermath of Brexit, many UK-based companies shifted their operations to other
EU countries. A study examined the impact of this relocation on the economies of the
countries where these companies moved.

Which finding, if true, would most strongly support the claim that Brexit has positively
impacted the economies of other EU countries?

A) Several EU countries that did not receive a significant number of UK-based companies
also experienced economic growth post-Brexit.
B) A few EU countries experienced a decline in economic growth following Brexit despite
an increase in UK-based companies relocating there.
C) Countries that received a significant number of relocating UK-based companies ex-
perienced higher economic growth post-Brexit.
D) Some UK-based companies that relocated to EU countries faced challenges in adapt-
ing to new regulations and market conditions.

22. Zadie Smith, a renowned British author, is known for exploring themes of cultural, racial,
and social identity in her novels. Her first novel, White Teeth, was published in 2000 and
has since garnered critical acclaim and numerous awards.

Which finding, if true, would most strongly support the claim that Zadie Smith’s novels
have had a significant impact on contemporary literature?

A) The number of novels exploring cultural, racial, and social identity themes has in-
creased since the release of White Teeth.
B) Zadie Smith’s novels have been adapted into successful film and television produc-
tions.
C) Zadie Smith has received prestigious literary awards for her novels, including the Man
Booker Prize.
D) Zadie Smith’s novels have been translated into multiple languages and are available
worldwide.

226
23. The Organization of American States (OAS) fosters cooperation among its member states,
aiming to promote democracy, human rights, and economic development within the West-
ern Hemisphere. However, critics argue that the OAS has been biased towards certain
political ideologies, which may have resulted in .

Which choice most logically completes the text?

A) a complete disregard for the economic development of the Western Hemisphere


B) the organization’s actions and decisions being influenced by specific political interests
C) an increased focus on improving cultural ties among member states
D) a decline in the overall e↵ectiveness of the OAS in promoting democracy and human
rights

24. A command economy is an economic system in which the government controls the produc-
tion, allocation, and prices of goods and services. While it can lead to a stable economic
environment, critics argue that the lack of market competition may result in inefficient
production and allocation of resources, ultimately leading to .

Which choice most logically completes the text?

A) increased individual freedom for citizens


B) stagnation and limited innovation in the long run
C) higher levels of income inequality among the population
D) greater incentives for entrepreneurship and private investment

227
25. The Trail of Tears was a series of forced relocations of Native American tribes from their
ancestral homelands in the southeastern United States to areas west of the Mississippi
River. Thousands of Native Americans su↵ered and died during the forced marches.
this tragic event had a lasting impact on the tribes involved, causing cultural
disruption and loss of traditional lands.

Which choice completes the text with the most logical transition?

A) On the other hand


B) Similarly
C) Consequently
D) Alternatively

26. Fly fishing is a popular angling method that uses an artificial ’fly’ to catch fish. The
fly is cast using a specialized weighted line, and the method requires a great deal of
skill and practice to master. fly fishing is often considered a more challenging
and rewarding form of angling compared to traditional bait fishing. Enthusiasts enjoy
the artistry involved in casting the fly and the connection with nature that comes from
standing waist-deep in a river or stream.

Which choice completes the text with the most logical transition?

A) On the other hand


B) In contrast
C) Consequently
D) Alternatively

228
27. James Watson, an American molecular biologist, is best known for his discovery of the
structure of DNA, along with Francis Crick and Rosalind Franklin. This groundbreaking
work earned them the Nobel Prize in 1962. Watson went on to write a contro-
versial memoir called ’The Double Helix,’ in which he chronicled the discovery and the
scientific community’s reactions.

Which choice completes the text with the most logical transition?

A) However
B) In addition
C) Subsequently
D) Alternatively

229
Chapter 29

Module 8 Answers

1. Correct Answer: B
Explanation: B: ’To highlight Dr. Jane Collins’s significant contribution to the field of
biotechnology and its impact on agriculture’ is the correct answer because the passage
focuses on Dr. Collins’s development of a gene-editing technique and its influence on
agriculture and sustainable farming practices.

2. Correct Answer: A
Explanation: A: ’To summarize the central plot and conflicts of the film ’Back to the
Future” is the correct answer because the passage provides a brief overview of the storyline,
highlighting Marty’s accidental time travel, the threat to his existence, and the e↵orts to
restore the timeline.

3. Correct Answer: B
Explanation: B: ’By appreciating her willingness to explore new strategies and adapt to
the sport’s evolution’ is the correct answer because Tom Harrison (Text 2) emphasizes
the importance of adapting to changing conditions and exploring alternative techniques,
which aligns with Alice Jamison’s innovative approach (Text 1).

4. Correct Answer: A
Explanation: A: ’was elected’ is the correct answer because it uses the passive voice in
the past tense, which is consistent with the other past tense verbs in the passage and
correctly describes the event that took place in 1996.

5. Correct Answer: D
Explanation: D: ’utilize’ is the correct answer because it provides a finite verb that agrees
with the subject ’scientists’ and is consistent with the tense used in the rest of the passage.

230
6. Correct Answer: A
Explanation: A: ’became’ is the correct answer because it provides a finite past tense
verb that is consistent with the other verbs in the passage and correctly describes Marie
Curie’s accomplishment.

7. Correct Answer: C
Explanation: C: ’led’ is the correct answer because it provides the appropriate past tense
verb to describe the rivalry that occurred in the 1980s.

8. Correct Answer: A
Explanation: A: ’Alan Turing, a pioneer in computer science, developed the Turing ma-
chine concept and laid the groundwork for artificial intelligence with the Turing Test.’ is
the correct answer because it e↵ectively emphasizes Turing’s contributions to the field of
computer science by mentioning both the Turing machine and the Turing Test.

9. Correct Answer: B
Explanation: B: ’The primary objective in both field and box lacrosse is to score goals
by propelling the ball into the opposing team’s net using a crosse.’ is the correct answer
because it e↵ectively summarizes the main objective of the game using information from
the notes.

10. Correct Answer: A


Explanation: A: ’Perfect competition is a market structure with many buyers and sellers,
homogeneous products, and minimal barriers to entry and exit, making firms price tak-
ers.’ is the correct answer because it e↵ectively summarizes the characteristics of perfect
competition from the notes.

11. Correct Answer: D


Explanation: D: ’relationship’ is the correct answer because it correctly completes the
sentence and maintains the singular form of the noun, indicating a single relationship
between the two governments.

12. Correct Answer: D


Explanation: D: ’countries” is the correct answer because it indicates that they will be
visiting national parks and hiking trails in multiple countries, which requires the plural
possessive form ’countries” to show that the trails belong to more than one country.

231
13. Correct Answer: D
Explanation: D: ’design’ is the correct answer because it is the most appropriate term
to describe Bell’s role in the creation of the aircraft, making the sentence grammatically
correct and coherent.

14. Correct Answer: D


Explanation: D: ’suicide’ is the correct answer because it accurately describes the tragic
end of Cleopatra’s life after she was captured by the Romans.

15. Correct Answer: C


Explanation: C: ’overlooked’ is the correct answer because it shows that Hedy Lamarr’s
work was not recognized for a long time, despite its significance.

16. Correct Answer: A


Explanation: A: ’vibrant’ is the correct answer because it accurately describes the lively
and engaging atmosphere associated with the milonga, which is a social event that attracts
diverse participants.

17. Correct Answer: A


Explanation: A: ’The three primary exercises in powerlifting and their role in a well-
rounded program’ is the correct answer because the passage discusses the squat, bench
press, and deadlift as essential exercises and how a comprehensive powerlifting program
includes these exercises.

18. Correct Answer: A


Explanation: A: ’The French Revolution, driven by radical ideals, led to the Reign of
Terror and the rise of Napoleon.’ is the correct answer because it summarizes the main
ideas of the passage, which are the radical Jacobin party’s goals, the Reign of Terror, and
the eventual rise of Napoleon Bonaparte.

19. Correct Answer: D


Explanation: D: ’The power struggle between the Bolsheviks and Provisional Government
during the Russian Revolution resulted in the Bolsheviks’ victory and the Soviet Union’s
establishment.’ is the correct answer because it accurately summarizes the main idea of
the text, which focuses on the conflict between the two factions and its outcome.

232
20. Correct Answer: B
Explanation: B: ’Products made with the new technique show a 50% increase in strength
and a higher recycling rate compared to conventional paper products.’ is the correct
answer because this finding directly supports the e↵ectiveness of the new technique by
showing a significant improvement in strength and recyclability.

21. Correct Answer: C


Explanation: C: ’Countries that received a significant number of relocating UK-based
companies experienced higher economic growth post-Brexit.’ is the correct answer because
it directly supports the claim that Brexit has positively impacted the economies of other
EU countries by showing a correlation between the relocation of UK-based companies and
economic growth in those countries.

22. Correct Answer: A


Explanation: A: ’The number of novels exploring cultural, racial, and social identity
themes has increased since the release of White Teeth.’ is the correct answer because it
directly shows that her novels have influenced other writers and the themes they choose
to explore, thereby impacting contemporary literature.

23. Correct Answer: B


Explanation: B: ’the organization’s actions and decisions being influenced by specific
political interests’ is the correct answer because it directly relates to the criticism of the
OAS being biased towards certain political ideologies.

24. Correct Answer: B


Explanation: B: ’stagnation and limited innovation in the long run’ is the correct answer
because the passage indicates that the lack of market competition in a command economy
may lead to inefficient production and resource allocation, which could stifle growth and
innovation.

25. Correct Answer: C


Explanation: C: ’Consequently’ is the correct answer because it logically signals that
the lasting impact on the tribes is a result or consequence of the forced relocations and
su↵ering during the Trail of Tears.

233
26. Correct Answer: C
Explanation: C: ’Consequently’ is the correct answer because it logically signals a con-
clusion or consequence based on the previous statement, emphasizing the challenge and
reward associated with fly fishing as a result of the skill and practice required.

27. Correct Answer: C


Explanation: C: ’Subsequently’ is the correct answer because it logically signals that the
event of Watson writing the memoir followed the event of earning the Nobel Prize.

234

You might also like